Abdominsl Surgery Merged

You might also like

Download as pdf or txt
Download as pdf or txt
You are on page 1of 141

Назва наукового напрямку (модуля): Семестр: 11

General questions in surgery (situational task) 6 course


Опис:

Перелік питань:
1. Complaints of cough with purulent sputum, increased body temperature to 39°C, pain in the left
half of the chest. Has been ill for 2 weeks, the onset is caused by undercooling. The lag of the left
half of the chest during breathing, lung sounds by percussion, by auscultation crackling rales over
the lower lobe of the left lung. On X-ray of the chest expressed infiltration of lung tissue with areas
of enlightenment in the center. What is the primary diagnosis?
A. * Abscessing pneumonia.
B. Pleural empyema.
C. Acute lung abscess.
D. Pyopneumothorax.
E. Bronchiectatic disease.
2. Complaints of cough with purulent sputum, increased body temperature to 39°C, pain in the left
half of the chest. Has been ill for 2 weeks, the onset is caused by undercooling. The lag of the left
half of the chest during breathing, the shortening of percussion sound over the lower lobe, by
auscultation weakened breathing with amphoric sound, crackling rales. On X-ray of the chest
expressed infiltration of lung tissue with enlightenment in the center with fluid level. What is the
primary diagnosis?
A. * Acute lung abscess.
B. Pleural empyema.
C. Abscessing pneumonia.
D. Pyopneumothorax.
E. Bronchiectatic disease.
3. Complaints of cough with purulent sputum, increased body temperature to 39°C, pain in the left
half of the chest. Has been ill for 2 months, the onset is caused by undercooling. The lag of the left
half of the chest during breathing, the shortening of percussion sound over the lower lobe, by
auscultation weakened breathing with amphoric sound. On X-ray of the chest the destruction cavity
with the fibrous capsule in the projection of lower lobe of the left lung, infiltration of lung tissue is
not determined. What is the primary diagnosis?
A. * Chronic lung abscess.
B. Pleural empyema.
C. Acute lung abscess
D. Abscessing pneumonia.
E. Pyopneumothorax.
4. Complaints of cough with purulent sputum, increased body temperature to 39°C, pain in the left
half of the chest. Has been ill for 2 weeks, the onset is caused by undercooling. The lag of the left
half of the chest during breathing, the shortening of percussion sound over the lower lobe, by
auscultation weakened breathing with amphoric sound. On the X-ray of chest the destruction cavity
with the fluid level, with a clear thin-walled capsule in the projection of the lower lobe of the left
lung. Infiltration of lung tissue is not determined. What is the primary diagnosis?
A. * Suppurative cyst of the lung.
B. Pleural empyema.
C. Acute lung abscess
D. Abscessing pneumonia.
E. Pyopneumothorax.
5. Complaints of cough with purulent sputum, increased body temperature to 39°C, pain in the left
half of the chest. Has been ill for 2 weeks, the onset is caused by undercooling. The lag of the left
half of the chest during breathing, over the left lobe a dull percussion sound, by auscultation the
breathing is absent. On X-ray of the chest the shadow in the basal parts of the left lung with an
oblique upper level along Damuazo's line. What is the primary diagnosis?
A. * Pleural empyema.
B. Acute lung abscess
C. Chronic lung abscess.
D. Suppurative cyst of the lung.
E. Pyopneumothorax.
6. Complaints of cough with purulent sputum, increased body temperature to 39°C, pain in the left
half of the chest, dyspnea. Has been ill for 2 weeks, the onset is caused by undercooling. The lag of
the left half of the chest during breathing, the shortening of percussion sound over the lower lobe,
by auscultation the breathing is absent. On X-ray of the chest the shadow in the basal parts of the
left lung with a horizontal fluid level and enlightenment over it. Is visible the edge of collapsed
lung. What is the primary diagnosis?
A. * Limited pyopneumothorax.
B. Acute lung abscess
C. Chronic lung abscess.
D. Pleural empyema.
E. Total pyopneumothorax.
7. Complaints of cough with foul-smelling purulent sputum with streaks of blood, increased body
temperature to 40°C, pain in the left half of the chest, dyspnea at rest. Has been ill for 2 weeks, the
onset is caused by undercooling. The lag of the left half of the chest during breathing, with a
shortening of the pulmonary percussion sound, by auscultation moist rales over the left lung. On
X-ray of the chest expressed infiltration of the left lung with multiple sites of destruction. What is
the primary diagnosis?
A. * Gangrene of the lung.
B. Pleural empyema.
C. Acute lung abscess
D. Abscessing pneumonia.
E. Pyopneumothorax.
8. Complaints of cough with foul-smelling purulent sputum with streaks of blood, increased body
temperature to 40°C, pain in the left half of the chest, dyspnea at rest. Has been ill for 2 weeks, the
onset is caused by undercooling. The lag of the left half of the chest during breathing, with a
shortening of the pulmonary percussion sound, by auscultation moist rales over the lower lobe of
the left lung. On X-ray of the chest expressed infiltration of left lung tissue with a giant cavity in
the lower lobe with the level of the fluid. What is the primary diagnosis?
A. * Gangrenous abscess of lung.
B. Pleural empyema.
C. Acute lung abscess
D. Abscessing pneumonia.
E. Gangrene of the lung.
9. Complaints of cough with purulent sputum, increased body temperature to 39°C, pain in the left
half of the chest. Has been ill for 2 weeks, the onset is caused by undercooling. The lag of the left
half of the chest during breathing, the shortening of percussion sound over the lower lobe, by
auscultation weakened breathing. On X-ray of the chest paracostal fusiform shadow in the
projection of the left lower lobe of the lung. What is the primary diagnosis?
A. * Limited empyema.
B. Wide-spread pleural empyema.
C. Acute lung abscess
D. Chronic lung abscess.
E. Pyopneumothorax.
10. In the patient on the fourth day after the chest trauma on X-ray - heterogeneous shadow in the
lower lobe. By puncture received a small amount of light yellow fluid with blood clots. What
treatment are the best for the patient?
A. * Drainage of the pleural cavity
B. Operational - lung decortication
C. Daily puncture
D. Resorbed therapy
E. Antibacterial therapy
11. In the patient, 48 years old, on the seventh day after the onset of a moderate pain in the chest,
severe cough, fever to 39°C appeared the bad-smell sputum. The patient's condition remains grave,
with expectoration more than 600 ml of gray-green sputum, and hectic fever. On X-ray – on the
background of the heterogeneous shadow of the lower lobe of right lung revealed a cavity with a
horizontal level of fluid. What is the primary diagnosis?
A. * Abscess of the right lung
B. Gangrene of the right lung
C. Acute abscess of the right lung
D. Suppuration cyst of the right lung
E. Suppuration tuberculous cavern
12. In the patient, 35 years old, during the physical exertion appeared severe pain in the left half of the
chest. Objectively: the patient is covered with cold sweat, dyspnea, pain during inspiration. By
auscultation: vesicular breathing on the right side, on the left - is absent. Tachycardia, pulse 100
beats/min. What is the primary diagnosis?
A. * Spontaneous pneumothorax
B. Angina pectoris
C. Acute myocardial infarction
D. Left-sided pleurisy
E. Pneumonia
13. The patient A., age 37, entered with complaints of cough with purulent sputum to 150 ml per day,
pain in the right half of the chest, fever to 38°C. Has been ill for two weeks. The day before the
entrance to the clinic during cough attack expectorated to 300 ml of purulent bad-smell sputum. On
examination: a shortening of the pulmonary percussion sound under the right scapula, and the
weakening of vesicular breathing. What is the primary diagnosis?
A. * Acute lung abscess
B. Acute bronchitis
C. Exacerbation of chronic abscess
D. Exacerbation of bronchoectatic disease
E. Pleural empyema
14. Patient A., aged 42, had been treated for two months for an acute abscess of the upper lobe of right
lung without improvement. The treatment: intramuscular injection of antibiotics, sulfanilamidns
drugs. Remains the cough with purulent sputum to 80-100 ml per day, fever (37,6°C). What is the
primary diagnosis?
A. * Chronic lung abscess
B. Acute abscess of the right lung
C. Tuberculous cavern
D. Peripheral lung cancer
E. Suppurative cyst of lung

15. The patient, 78 years old, entered with complaints of pain in the left half of the chest, coughing,
with daily 80 ml of mucopurulent sputum, fever to 37,2°C. The X-rays of the lower lobe of right
lung revealed a cavity with irregular internal border and outside spicules with minor infiltration
around. What is the primary diagnosis?
A. * Hollow form of lung cancer
B. Chronic lung abscess
C. Suppuration cyst of lung
D. Fibro-cavernous tuberculosis
E. Limited empyema
16. The patient has a pyogenic lung abscess, which was complicated by repeated bleeding., The patient
is undergoing the operative treatment. What antibiotics are the most suitable for preoperative
prophylaxis?
A. * Cephalosporins.
B. Penicillin.
C. Macrolides.
D. Aminoglycosides.
E. Fluorohinolones.
17. The patient with bilateral hydrothorax has undergone the repeat pleural puncture of both sides.
After the last puncture felt the deterioration, fever, pain in the chest. Therapeutist on the next day
during pleural puncture on the right obtained the pus. What is the mechanism of acute right-side
A. * empyema?
Contact-aspirating.
B. Lymphogenous.
C. Hematogenous.
D. Implantation.
E. Airborne.
18. The patient has the pyogenic lung abscess, which was complicated by bleeding. What medicines
are the most suitable to stop the bleeding?
A. * Vitamin K.
B. Anticoagulants.
C. Antibiotics.
D. Antiaggregants.
E. Prostaglandins.
19. The patient has the lung abscess, which was complicated by bleeding. What medicines are the most
suitable to stop the bleeding?
A. * Dicynon.
B. Heparin.
C. Penicillin.
D. Courantil.
E. Alprostan.
20. The patient has the lung abscess, which was complicated by bleeding to 200 ml. How this bleeding
is classified?
A. * I degree
B. 0 degree
C. II degree
D. III degree
E. IV degree

21. The patient received chest trauma 2 hours ago. Complains of the severe pain in the right half of
the chest, dyspnea at rest. On examination: the lag of the right half of the chest during breathing,
crepitation along the V-VI ribs on the right side, by percussion - tympanic sound, by auscultation -
breathing is absent. What is the primary diagnosis?
A. * Closed chest trauma. Fractures of V-VI ribs on the right side. Posttraumatic pneumothorax.
B. Closed chest trauma. Fractures of V-VI ribs on the right side.
C. Closed chest trauma. Fractures of V-VI ribs on the right side. Posttraumatic hemothorax.
D. Closed chest trauma. Fractures of V-VI ribs on the right side. Posttraumatic subcutaneous
emphysema.
E. Closed chest trauma. Fractures of V-VI ribs on the right side. Posttraumatic complicated
hemothorax.
22. The patient received chest trauma 2 hours ago. Complains of the severe pain in the right half of
the chest, dyspnea at rest. On examination: the lag of the right half of the chest during breathing,
crepitation along the V-VI ribs on the right side, by percussion - tympanic sound, by auscultation -
breathing is absent. The primary diagnosis: Closed chest trauma. Fractures of V-VI ribs on the right
side. Posttraumatic pneumothorax. What is the typical treatment of rib fracture?
A. * Novocaine block
B. External fixation of ribs
C. Intrmedullary costal osteosynthesis;
D. Mechanical ventilation with positive end-expiratory pressure
E. Thoracotomy
23. The patient received chest trauma 2 hours ago. Complains of the severe pain in the right half of
the chest, dyspnea at rest. On examination: the lag of the right half of the chest during breathing,
crepitation along the V-VI ribs on the right side, by percussion - tympanic sound, by auscultation -
breathing is absent. The primary diagnosis: Closed chest trauma. Fractures of V-VI ribs on the right
side. Posttraumatic pneumothorax. What is the treatment of pneumothorax?
A. * Pleural drainage
B. Pleural puncture
C. Thoracotomy
D. Pneumonectomy, bilobectomy, lobectomy
E. Conservative treatment
24. The patient received chest trauma 2 hours ago. Complains of the severe pain in the right half of
the chest, dyspnea at rest. On examination: the lag of the right half of the chest during breathing,
crepitation along the V-VI ribs on the right side, by percussion - tympanic sound, by auscultation -
breathing is absent. The primary diagnosis: Closed chest trauma. Fractures of V-VI ribs on the right
side. Posttraumatic pneumothorax. Where the drainage of pleural space in pneumothorax is
performed?
A. * II intercostal space, midclavicular line
B. II intercostal space, scapular line
C. IV intercostal space, anterior axillary line
D. VII intercostal space, midclavicular line
E. VII intercostal space, scapular line
25. The patient received chest trauma 14 days ago. Complains of a moderate pain in the right half of
the chest, dyspnea, fever up to 38,5°C. Lag of the right half of the chest during breathing, narrowing
of the intercostal spaces. By percussion - a shortening of the percussion sound over the right lung,
by auscultation - weakening of breathing. What is the primary diagnosis?
A. * Right-side suppurative hemothorax.
B. Right-side posttraumatic pneumonia.
C. Right-side hemothorax.
D. Right-side pyopneumothorax.

E. Consolidated rib fractures.


26. Was pressed by the truck to the wall. Complains of the expressed dyspnea, difficult breathing, chest
pain. On examination the expressed cyanosis. The frequency of respiratory movements - 26-28 per
1 min. Unstable hemodynamics. The chest is deformed, abnormal mobility of the front wall. The
swelling of soft tissues of the neck with crepitation. What is the primary diagnosis?
A. * Mediastinal emphysema.
B. Posttraumatic pneumothorax.
C. Posttraumatic hemothorax.
D. Posttraumatic pneumonia
E. Subcutaneous emphysema.
27. Was pressed by the truck to the wall. Complains of the expressed dyspnea, difficult breathing, chest
pain. On examination the expressed cyanosis. The frequency of respiratory movements - 26-28 per
1 min. Unstable hemodynamics. The chest is deformed, abnormal mobility of the front wall. The
swelling of soft tissues of the neck with crepitation. The primary diagnosis: Mediastinal
emphysema. What is the treatment of mediastinal emphysema?
A. * Drainage of anterior mediastinum
B. Conservative treatment
C. Drainage of pleural cavity
D. Novocaine block
E. Pericardial puncture
28. Was pressed by the truck to the wall. Complains of the expressed dyspnea, difficult breathing, chest
pain. On examination the expressed cyanosis. The frequency of respiratory movements - 26-28 per
1 min. Unstable hemodynamics. The chest is deformed, abnormal mobility of the front wall. The
swelling of soft tissues of the neck with crepitation. The primary diagnosis: Mediastinal
emphysema. What is the main cause of mediastinal emphysema?
A. * Disruptions of trachea, bronchi
B. Rib fracture
C. Pneumothorax
D. Hemothorax
E. Mediastinal tumours
29. Was pressed by the truck to the wall. Complains of the expressed dyspnea, difficult breathing, chest
pain. On examination the expressed cyanosis. The frequency of respiratory movements - 26-28 per
1 min. Unstable hemodynamics. The chest is deformed, abnormal mobility of the front wall. The
swelling of soft tissues of the neck with crepitation. The primary diagnosis: Mediastinal
emphysema. What does the mediastinal emphysema result in?
A. * Cardiac tamponade
B. Hemoptysis
C. Pleural empyema
D. Pneumothorax
E. Lung atelectasis
30. Female patient, 62 years old, was got in accident. On examination was detected the region of the
right half of the chest, which disengages during inspiration. What are the most appropriate
therapeutic measures?
A. * External fixation of a floating area
B. Introduction of narcotic analgetics
C. Vagosympathetic block by Vishnevsky
D. Tight chest bandage
E. Paravertebral blockade

31. In the patient after the accident with multiple rib fracture during the puncture of pleural cavity
received the gastric content. What additional examination is the most suitable?
A. * Chest X-ray with contrast of the stomach
B. Plain X-ray of abdominal cavity
C. Esophagogastroscopy
D. Computer tomography
E. Tomography of the chest
32. In the patient on the fourth day after the chest trauma on X-ray - heterogeneous shadow in the
lower lobe. By puncture received a small amount of light yellow fluid with blood clots. What
treatment are the best for the patient?
A. * Drainage of the pleural cavity
B. Operational - lung decortication
C. Daily puncture
D. Resorbed therapy
E. Antibacterial therapy
33. In the patient after blunt chest trauma with a sternum fracture appeared the weakness, hypotension,
cyanosis of the upper half of the body, distension of the neck veins. By pleural puncture the content
is absent. Pulse 120 beats.per min, rhythmic, weakened. What is the primary diagnosis?
A. * Cardiac tamponade
B. Pulmonary embolism
C. Contusion of the heart
D. Acute myocardial infarction
E. Coagulated hemopericardium
34. The patient entered in 3 hours after the injury with expressed subcutaneous emphysema of the
upper half of the body, dyspnea, tachycardia, pulse - 120 beats/min. On X-ray the pneumothorax
was found out with significantly enlargement of the mediastinum in both sides. What is the first
A. * aid?
Drainage of the anterior mediastinum
B. Puncture of the pleural cavity
C. Drainage of the pleural cavity
D. Thoracoscopy
E. Thoracotomy
35. The patient entered in 3 hours after the injury with expressed subcutaneous emphysema of the
upper half of the body, dyspnea, tachycardia, pulse - 120 beats/min. On X-ray the pneumothorax
was found out with significantly enlargement of the mediastinum in both sides. What is the primary
A. * diagnosis?
Mediastinal emphysema.
B. Posttraumatic pneumothorax.
C. Posttraumatic hemothorax.
D. Posttraumatic pneumonia
E. Subcutaneous emphysema.
36. Patient S., 25 years old, entered the hospital after the chest trauma. During clinical and X-ray
examination was diagnosed the left-side tension pneumothorax. What is the first aid?
A. * Drainage of the pleural cavity
B. Intravenous infusion
C. Oxygenotherapy
D. Intubation
E. Analgesics

37. Patient S., 25 years old, entered the hospital after the chest trauma. During clinical and X-ray
examination was diagnosed the left-side tension pneumothorax. Where the drainage of pleural
space in pneumothorax is performed?
A. * II intercostal space, midclavicular line
B. II intercostal space, scapular line
C. IV intercostal space, anterior axillary line
D. VII intercostal space, midclavicular line
E. VII intercostal space, scapular line
38. Patient S., 35 years old, entered the hospital after the chest trauma. During clinical and X-ray
examination was diagnosed the left-side hemothorax. Where the drainage of pleural space in
hemothorax is performed?
A. * VII intercostal space, scapular line
B. II intercostal space, midclavicular line
C. II intercostal space, scapular line
D. IV intercostal space, anterior axillary line
E. VII intercostal space, midclavicular line
39. Patient S., 35 years old, entered the hospital after the chest trauma. During clinical and X-ray
examination was diagnosed the left-side hemothorax. What method is the most informative in the
diagnostic of hemothorax?
A. * Pleural puncture
B. General blood analysis
C. Sputum analysis
D. Auscultation
E. X-ray examination
40. Patient S., 35 years old, entered the hospital after the chest trauma. During clinical and X-ray
examination was diagnosed the left-side hemothorax. What test is used to determine the
continuity of pleural bleeding?
A. * Revilour-Greguar's test
B. Troyanov-Trendelenburg's test
C. Talman's test
D. Mayo-Pratt's test
E. Delbe-Pertess test (marching test)
41. In the patient, 35 years old, during the physical exertion appeared severe pain in the left half of the
chest. Objectively: the patient is covered with cold sweat, dyspnea, pain during inspiration. By
auscultation: vesicular breathing on the right side, on the left - is absent. Tachycardia, pulse 100
beats/min. What is the primary diagnosis?
A. * Spontaneous pneumothorax
B. Angina pectoris
C. Acute myocardial infarction
D. Left-sided pleurisy
E. Pneumonia
42. 54 years old patient complains of dysphagia. Two years ago noticed on the left side of neck the
appearance of protrusion after eating, vomiting by food, night cough. Began to lose his weight. On
X-ray of esophagus with barium at the level of the clavicle was revealed the depot of barium like
chicken egg by the size and shape. What is the most probable diagnosis?
A. * Diverticulum of the esophagus
B. Esophagotraheal fistula
C. Esophageal cancer
D. Stenosis of the esophagus

E. Esophageal achalasia
43. 54 years old patient complains of dysphagia. Two years ago noticed on the left side of neck the
appearance of protrusion after eating, vomiting by food, night cough. Began to lose his weight. On
X-ray of esophagus with barium at the level of the clavicle was revealed the depot of barium like
chicken egg by the size and shape. What complication is probable for this disease?
A. * Diverticulitis
B. Obstructive jaundice
C. Intestinal obstruction
D. Myocardial infarction
E. Lung atelectasis
44. 54 years old patient complains of dysphagia. Two years ago noticed on the left side of neck the
appearance of protrusion after eating, vomiting by food, night cough. Began to lose his weight. On
X-ray of esophagus with barium at the level of the clavicle was revealed the depot of barium like
chicken egg by the size and shape. What is the typical treatment of this disease?
A. * Surgical treatment
B. Spasmolytics
C. Analgetics
D. Nonsteroid antiinflammatory drugs
E. Antibiotics
45. 54 years old patient complains of dysphagia. Two years ago noticed on the left side of neck the
appearance of protrusion after eating, vomiting by food, night cough. Began to lose his weight. On
X-ray of esophagus with barium at the level of the clavicle was revealed the depot of barium like
chicken egg by the size and shape. What operation is performed in this disease?
A. * Resection of diverticulum
B. Esophagomyotomy
C. Esophagogastric anastomosis
D. Extirpation of esophagus
E. Esophageal plastics by intestine
46. To the hospital entered a man in the critical condition: acrocyanosis, dyspnea, subcutaneous
emphysema on the neck and upper part of body. Complains of severe pain behind the breastbone
and epigastrium. The body temperature of 38,9°C, pulse 130 beats/min, blood pressure 80/50 mm
Hg. From anamnesis 6 years ago after drinking appeared the vomit, which resulted in the signatic.
What is the primary diagnosis?
A. * Spontaneous rupture of esophagus
B. Incarceration of paraesophageal hernia
C. Spontaneous pneumothorax
D. Pulmonary embolism
E. Perforated ulcer
47. 38-year-old woman complains of difficulty passing of food through esophagus, periodic vomiting.
Has been ill for 1,5 years. Last 6 months notes appearance of food on the pillow during sleep. Lost
15 kg of body weight. There were constipations, stool once in 3-4 days. On plain X-ray film the
absence of the gas bubble of the stomach. What stage of dysphagia relates to such manifestations?
A. * II
B. I
C. III
D. IV
E. V

48. 38-year-old woman complains of difficulty passing of food through esophagus, periodic vomiting.
Has been ill for 1,5 years. Last 6 months notes appearance of food on the pillow during sleep. Lost
15 kg of body weight. There were constipations, stool once in 3-4 days. On plain X-ray film the
absence of the gas bubble of the stomach. What is the roentgenological sign of this disease in
contrast X-ray with barium?
A. * "Rat tail" sign
B. Filling defects
C. "Niche" sign
D. "Bell" sign
E. Blunt His angle
49. 38-year-old woman complains of difficulty passing of food through esophagus, periodic vomiting.
Has been ill for 1,5 years. Last 6 months notes appearance of food on the pillow during sleep. Lost
15 kg of body weight. There were constipations, stool once in 3-4 days. On plain X-ray film the
absence of the gas bubble of the stomach. What is the main method of diagnostic of this disease?
A. * X-ray examination with barium swallow
B. Pleural punctere
C. Ultrasound examination
D. Plain X-ray examination of the chest
E. Irrigoscopy
50. On X-ray of the esophagus in the right lateral projection in the middle third on the front wall was
found out the additional shadow, of round shape with smooth contours to 2 cm in diameter. What is
the most probable diagnosis?
A. * Diverticulum of the esophagus
B. Achalasia of the esophagus
C. Esophageal cancer
D. Chemical burn of the esophagus
E. Diaphragmatic hernia
51. The tool dilation of burn and peptic stricture of the esophagus has a risk of perforation with the
development of purulent mediastinitis and pleural empyema. What is the least dangerous method
for perforation should be applied in the first attempt of dilation of the stricture?
A. * Dilatation of the stricture by balloon dilatator with a stable diameter of the cylinder.
B. Bouginage under the control of esophagoscope.
C. Bouginage along the metal conductor.
D. The blind bouginage under local anesthesia.
E. Bouginage under the control of X-ray
52. Female complains of difficult passing of food through esophagus, vomiting by unchanged food,
regurgitation in night and weight loss. Anamnesis about 10 years. On X-ray study revealed
achalasia of esophagus of the IV stage with S-shaped deformation. What is the optimal treatment?
A. * Operation esophagocardiomyotomy with plastic by the stomach fundus.
B. Cardiodilatation by hard probe.
C. Cardiodilatation balloon probe.
D. Operation esophago-fundoanastomosis by Heyrovsky.
E. Resection of the cardia with esophageal anastomosis.
53. Female complains of difficult passing of food through esophagus, vomiting by unchanged food,
regurgitation in night and weight loss. Anamnesis about 10 years. On X-ray study revealed
achalasia of esophagus of the IV stage. What is the characteristic feature of the IV stage of this
A. * disease?
Considerable esophageal dilation with S-shaped elongation.
B. Cicatrical changes with expressed esophageal dilation, the peristalsis is absent
C. Asymptomatic

D. Functional spasm without esophageal dilation


E. Constant spasm with a moderate esophageal dilation and maintained peristalsis
54. Patient 52 entered the clinic with complaints of complete obstruction of the esophagus, salivation,
weakness, t-38,7°C. Dysphagia has been for 8 days, after the swallowing of piece of the meat with
bone. On X-ray the barium delays at the middle third of the esophagus. On fibroesophagoscopy
was found a wedged bone with hyperemia and edema of the mucous membrane, covered by fibrin.
What is the optimal treatment strategy in this case?
A. * Surgical treatment: thoracotomy, esophagotomy, removal of foreign body (bone), suturing of the
esophagus + gastrostomy.
B. Endoscopic removal of foreign body by rigid esophagoscope
C. Pushing of foreign body in the stomach by bougie.
D. Removal of foreign body by the Fogarty's probe
E. Removal of foreign body by fiberoptic endoscope.
55. Among the methods of esophageal plastic the most physiologic and safe modern method is:
A. * Isoperistaltic plastic by tube of the greater curvature of the stomach after the extirpation of the
esophagus through a cervical-laparotomy access.
B. Large intestine plastic in antiperistaltic position of the transplant.
C. Large intestine plastic in isoperistaltic position of the transplant.
D. Large intestine plastic with a skin flap.
E. Large intestine plastic by ileocecal segment.
56. The patient has the postburn stenosis of the esophagus. After the next bouginage felt the fever,
tachycardia, pain behind the breastbone. On X-ray: the horizontal level of fluid in the posterior
mediastinum. What is the most probable diagnosis?
A. * Acute posterior mediastinitis.
B. Acute anterior mediastinitis.
C. Diverticulum of the esophagus.
D. Acute pleural empyema.
E. Paraesophageal hernia.
57. Complaints of burning, pain behind the breastbone, loss of weight. Has been ill for 7 months. Last 2
weeks noticed difficult passing of solid food. On contrasting X-ray was diagnosed: filling defect of
lower thoracic part of the esophagus, a "niche" sign of the lesser curvature of stomach. What is the
most probable diagnosis?
A. * Gastric ulcer
B. Paraesophageal hernia
C. Decompensated pyloric stenosis
D. Sliding esophageal hernia
E. Peptic duodenal ulcer
58. Complaints of the pain behind the breastbone, difficult passage of solid food, weight loss,
dizziness. Has been ill for 3 months. Last 2 days disturbs the vomiting after fluid food, the
stagnation of fluid food. On EGDS severe narrowing of the esophagus, rigidity of the walls,
hyperemic mucosa without folds. What is the most probable diagnosis?
A. * Esophageal cancer
B. Sliding esophageal hernia
C. Paraesophageal hernia
D. Reflux esophagitis
E. Varicose veins of the esophagus

59. In the patient six months ago appeared the complaints of pain behind the sternum and a strong
burning sensation in the esophagus. Sometimes observed dysphagia. On X-ray examination found
the presence of diverticulum of the left wall of esophagus at the level of tracheal bifurcation 3?4
cm, just below the aortic arch. The patient was not treated. What tactics of treatment should be
A. * choused?
Right-side thoracotomy, diverticulectomy.
B. Left-side thoracotomy, diverticulectomy.
C. Right-side thoracotomy, resection of the esophagus.
D. Large intestine plastic of esophagus
E. Large intestine plastic of esophagus
60. The woman aged 52 complains of pain behind the breastbone, difficult passing of solid food
through esophagus, increased salivation. The doctor advised 0,1 % solution of atropine before
eating. After 3 days on X-rays no pathology was revealed. The doctor should do for this patient:
A. * Send to fibroesophagoscopy
B. Allow the job
C. Control visit after 2 months
D. Treatment by spasmolytics
E. Send to ECG
61. The woman, 38 years old, complains of difficulty passage of solid meal on esophagus, vomiting by
undigested food, night regurgitation (sign of ,,wet pillow"), loss of weight. Has been ill for 10
years. On X-ray examination with barium the sign of "rat tail", dilation of the esophagus to 6 cm
with maintained peristalsis. What stage of achalasia is there in this patient?
A. * II
B. I
C. 0
D. III
E. IV
62. 53 year old man complains of recurrent pain behind the breastbone, heartburn, especially in the
horizontal position. Sometimes the burning pain behind the sternum occurs after hot or spicy food.
Two weeks ago was vomiting by blood and lost of consciousness. Has entered the hospital after
repeated gastric bleeding. What is the most probable diagnosis?
A. * Sliding esophageal hernia, reflux esophagitis
B. Diverticulum of the esophagus
C. Varicose veins of the esophagus
D. Mallory-Weiss syndrome
E. Crohn's disease.
63. 53 year old man complains of recurrent pain behind the breastbone, heartburn, especially in the
horizontal position. Sometimes the burning pain behind the sternum occurs after hot or spicy food.
Two weeks ago was vomiting by blood and lost of consciousness. Has entered the hospital after
repeated gastric bleeding. What roentgenological sign confirms the pathology?
A. * "Bell" sign
B. "Rat tail" sign
C. Filling defects
D. "Bird-beak" sign
E. Esophageal dilatation
64. 53 year old man complains of recurrent pain behind the breastbone, heartburn, especially in the
horizontal position. Sometimes the burning pain behind the sternum occurs after hot or spicy food.
Two weeks ago was vomiting by blood and lost of consciousness. Has entered the hospital after
repeated gastric bleeding. What drugs are used for the treatment of this pathology?
A. * Blockers of proton pomp

B. Spasmolytics
C. Adrenoblockers
D. Blockers of calcium channel
E. Anticoagulants
65. 53 year old man complains of recurrent pain behind the breastbone, heartburn, especially in the
horizontal position. Sometimes the burning pain behind the sternum occurs after hot or spicy food.
Two weeks ago was vomiting by blood and lost of consciousness. Has entered the hospital after
repeated gastric bleeding. What disease should be this pathology differentiated from?
A. * Peptic ulcer
B. Pancreatitis
C. Intestinal obstruction
D. Cholecystitis
E. Bronchial asthma
66. 53 year old man complains of recurrent pain behind the breastbone, heartburn, especially in the
horizontal position. Sometimes the burning pain behind the sternum occurs after hot or spicy food.
Two weeks ago was vomiting by blood and lost of consciousness. Has entered the hospital after
repeated gastric bleeding. What is the main treatment of this pathology?
A. * Conservative treatment
B. Esophagostomy
C. Esophageal plastic
D. Resection of the esophagus
E. Resection of the stomach
67. 53 year old man complains of recurrent pain behind the breastbone, heartburn, especially in the
horizontal position. Sometimes the burning pain behind the sternum occurs after hot or spicy food.
Two weeks ago was vomiting by blood and lost of consciousness. Has entered the hospital after
repeated gastric bleeding. What type of operation is used for treatment of this pathology?
A. * Cruroplasty with Nissen's fundoplication
B. Esophagostomy
C. Cruroplasty
D. Resection of the esophagus
E. Resection of the stomach
68. After the birth of the child appeared the signs of respiratory failure. By auscultation on the left side
the breathing is absent. On X-ray expressed mediastinal shift to the right, the presence of protrusion
on the left side. What is the most probable diagnosis?
A. * Left-sided diaphragmatic hernia
B. Hypoplasia of right lung
C. Polycistosis of the left lung
D. Relaxation of the diaphragm
E. Esophageal stricture
69. The patient, 45 years old, complains of retrosternal pain, which increase at night, heartburn,
belching. Has been ill for 4 months. What research is the most appropriate?
A. * X-ray contrast study in Trendelenburg's position.
B. Plain chest X-ray
C. Fibroesophagogastroduodenoscopy
D. Plain X-ray of the abdominal cavity.
E. Computer tomography
70. The patient, 45 years old, complains of retrosternal pain, which increase at night, heartburn,
belching. Has been ill for 4 months. What is the most probable diagnosis?

A. * Sliding esophageal hernia, reflux esophagitis


B. Diverticulum of the esophagus
C. Varicose veins of the esophagus
D. Mallory-Weiss syndrome
E. Crohn's disease.
71. What is the most wide-spread cause of the peptic stricture of esophagus associated with
reflux-esophagitis?
A. * Sliding esophageal hernia.
B. Prolonged nasogastric intubation in the esophagus.
C. Short stay nasogastric intubation.
D. Frequent vomiting of pregnancy.
E. Achalasia of the esophagus
72. X-ray signs: the "bell" sign, blunt Hiss angle, absence of gas bubble of the stomach are
characteristic for:
A. * Sliding esophageal hernia.
B. Paraesophageal hernia.
C. Relaxation of the diaphragm.
D. Malignant tumor of the esophagus.
E. Achalasia of the esophagus
73. The patient has the pain behind the breastbone, heartburn, which increases after the meal. Has been
ill for 6 months. For 5 days has a black chair. On X-ray examination revealed the "bell" sign, blunt
Hiss angle, absence of gas bubble of the stomach. What is the most probable diagnosis?
A. * Sliding esophageal hernia.
B. Paraezofagalnaya esophageal hernia.
C. Relaxation of the diaphragm.
D. Malignant tumor of the esophagus.
E. Achalasia of the esophagus
74. Complaints of heaviness in the epigastric region, nagging pain after eating. When EGDS pathology
identified. On X-ray examination pronounced gas bubble of the stomach is is visible the to the level
III intercostal space on the left. What is the most probable diagnosis?
A. * Relaxation of the diaphragm.
B. Paraezofagalnaya esophageal hernia.
C. Sliding esophageal hernia.
D. Malignant tumor of the esophagus.
E. Achalasia of the esophagus.
75. The patients with sliding esophageal hernia mostly complain of:
A. * Heartburn, pain behind the breastbone.
B. Difficult passage of food, loss of weight.
C. The pain behind the breastbone, difficulty in food passage.
D. Pain in the epigastric region with irradiation into the right hypochondrium.
E. Pain in the epigastric region with irradiation into the back.
76. Complications paraesophageal hernias of esophagus:
A. * Incarceration
B. Malignancy.
C. Esophageal stricture.
D. Dysphagia.
E. Reflux esophagitis.

77. In the patient at night appeared a severe pain in the epigastric region, nausea, vomiting. 6 months
ago was diagnosed paraesophageal hernia. What complication arose?
A. * Incarceration
B. Malignancy.
C. Esophageal stricture.
D. Dysphagia.
E. Reflex esophagitis.
78. In the patient at night appeared a severe pain in the epigastric region, nausea, vomiting. 6 months
ago was diagnosed paraesophageal hernia. What type of operation is is indicated for this patient?
A. * Cruroplasty
B. Esophagostomy
C. Resection of the esophagus
D. Resection of the stomach
E. Cruroplasty with Nissen's fundoplication
79. Complaints of heartburn, pain behind the breastbone, outpour of food. Has been ill for 2 months.
On X-ray examination revealed the "bell" sign, blunt Hiss angle, absence of gas bubble of the
stomach. Your tactics?
A. * Conservative therapy.
B. Bouginage of esophagus.
C. Surgical intervention.
D. Large-intestinal plastic of esophageal cardia.
E. Small-intestinal plastic of esophageal cardia.
80. The patient has the postburn stenosis of the esophagus. After the next bouginage felt the fever,
tachycardia, pain behind the breastbone. On X-ray: the horizontal level of fluid in the posterior
mediastinum. What is the most probable diagnosis?
A. * Acute posterior mediastinitis.
B. Acute anterior mediastinitis.
C. Diverticulum of the esophagus.
D. Acute pleural empyema.
E. Paraesophageal hernia.
81. Complaints of burning, pain behind the breastbone. Has been ill for 4 months. On contrasting X-ray
was diagnosed: the "bell" sign, the absence of gas bubble of the stomach, blunt Hiss angle. What is
the most probable diagnosis?
A. * Sliding esophageal hernia
B. Paraesophageal hernia
C. Gastric ulcer
D. Peptic duodenal ulcer
E. Decompensated pyloric stenosis
82. 5 hours ago, after the bouginage of esophageal cicatricle stricture appeared the pain behind the
breastbone, a feeling of compression, subcutaneous crepitation on the neck, fever to 38,5°C. On
examination the weakening of the heart tones by auscultation. On plain X-ray of chest –
mediastinal emphysema, mediastinal extension of the shadows. What is the most probable
A. * diagnosis?
Acute mediastinitis
B. Reflux esophagitis
C. Sliding esophageal hernia
D. Paraesophageal hernia
E. Achalasia of the esophagus

83. A 34-year-old female with hypertension is considering to become a pregnant. Which of the
following medications would be absolutely contraindicated to control her BP during pregnancy?
A. Methyldopa
B. Metoprolol
C. Labetalol
D. * Captopril
E. Nifedipine
84. A 34-year-old male with isolated essential hypertension came to clinic and it was found high BP
of 180/100 mm Hg after failure of behavioral modifications. What is the most appropriate next step?
A. Start hydrochlorothiazide
B. * Start hydrochlorothiazide and lisinopril
C. Repeat BP in 4 weeks
D. Start amlodipine
E. Start doxazozin
85. A 34-year-old male with isolated essential hypertension came to clinic and it was found high BP
of 180/100 mm Hg item after failure of behavioral modifications. What is the most appropriate next
step?
A. Start hydrochlorothiazide
B. * Start hydrochlorothiazide and lisinopril
C. Repeat BP in 4 weeks
D. Start amlodipine
E. Start doxazozin
86. A 40-year-old diabetic patient with a blood pressure (BP) of 145/90 mm Hg item and proteinuria.
Which BP profile represents the best therapeutic goal for this patient?
A. 160/90
B. 140/90
C. 130/85
D. * 125/75
E. 140/85
87. A 40-year-old diabetic patient presents with a blood pressure (BP) of 145/90 mm Hg item and
proteinuria. What is the best medication for the initial management of this patient’s hypertension?
A. Calcium channel blockers
B. Beta blockers
C. * ACE-inhibitors / angiotensin receptor blockers
D. Alpha blockers
E. Diuretics
88. A 42-year-old female with chronic obsctructive pulmonary disease is found on multiple office visits
to have elevated BP measurements. Which of the following medications is contraindicated?
A. Hydrochlorothiazide
B. Metoprolol
C. Lisinopril
D. * None of above
E. All of the above
89. A 42-year-old male comes in for a routine physical examination. He is noted to have impaired
glucose methabolism, and a BP of 135/85 mmHg. What is the best treatment plan for this
A. * individual?
Aggressive lifestyle modification
B. Institute thiazide diuretic regimen
C. No treatment at this time

D. Initiate an ACE-inhibitor
E. Initiate a beta-blocker
90. A 45-year-old male comes in for a routine physical examination. He is noted to have impaired
glucose methabolism, and a BP of 140/85 mmHg. What is the best treatment plan for this
A. * individual? lifestyle modification
Aggressive
B. Institute thiazide diuretic regimen
C. No treatment at this time
D. Initiate an ACE-inhibitor
E. Initiate a beta-blocker
91. A 48-year-old male with diabetes mellitus, hypertension, and hyperlipidemia has a hypertensive
emergency. His arterial pressure is 150/100 mmHg item. Which medications would be most
appropriate therapy for this patient?
A. * Nitroprusside
B. Enteral metoprolol
C. Fenoldopam
D. Intravenous nitroglycerine
E. Any of the above
92. ?A 48-year-old obese male with hypertension, dyslipidemia, and diabetes mellitus presents to the
outpatient clinic for his yearly physical. He has refused medications in the past, but now is willing
to consider treatment. His BP is 145/95 mmHg with a HR of 80 bpm. His laboratory data are
significant for the presence of microalbuminuria. Which of the following medications would be the
most appropriate?
A. Carvedilol
B. Methyldopa
C. * Lisinopril
D. Chlorthalidone
E. Terazozin
93. A 48-year-old obese male with hypertension, dyslipidemia, and diabetes mellitus presents to the
outpatient clinic for his yearly physical. He has refused medications in the past, but now is willing
to consider treatment. His BP is 145/95 mmHg with a HR of 80 bpm. His laboratory data are
significant for the presence of microalbuminuria. Which of the following medications would be the
most appropriate?
A. Carvedilol
B. Methyldopa
C. * Lisinopril
D. Chlorthalidone
E. Terazozin
94. A 56-year-old male on hydralazine, hydrochlorothiazide, lisinopril, and metoprolol begins to
develop a malar rash and arthralgias. Which of the above antihypertensive agents is known to cause
drug-induced lupus?
A. Hydrochlorothiazide
B. Lisinopril
C. * Hydralazine
D. Metoprolol
E. None of the above
95. A 53-year-old male on hydralazine, hydrochlorothiazide, lisinopril, and metoprolol begins to
develop a malar rash and arthralgias. Which of the above antihypertensive agents is known to cause
drug-induced lupus?
A. Hydrochlorothiazide

B. Lisinopril
C. * Hydralazine
D. Metoprolol
E. None of the above
96. A 60-year old white man consults you with a headache. Examination data are unremarkable, except
that the blood pressure is raised and subsequent measurements confirm readings of 170/106 mmHg.
He is obese, smokes 15 cigarettes per day and drinks 2 pints of beer per day. His investigations
reveal a cholesterol of 6 mmol/l with a normal blood sugar and electrolyte profile and normal renal
function. There is no evidence of target organ damage.
A. This man needs immediate treatment with antihypertensive drugs, aspirin and a statin
B. * Initiate antihypertensive therapy with two-drug combination
C. At this age it is far more important to control the hyperlipidaemia than the BP
D. This man’s blood pressure could be wholly ascribed to his alcohol intake
E. This patient can be observed and counseled on maintaining a reasonable BMI
97. A 62-year-old man with isolated essential hypertension, currently taking hydrochlorothiazide 25 mg
PO daily, comes to you for his first clinic visit. He notes that his BP at home is always less than
140/80 mmHg, but in clinic it is always at least 155/95 mmHg. What is the next step?
A. Increase dose of thiazide
B. Addition of second antihypertensive medication
C. * Do nothing as he has white coat hypertension
D. Evaluate for secondary causes of hypertension
E. Start metoprolol
98. A 62-year-old man with isolated essential hypertension, currently taking hydrochlorothiazide 25 mg
PO daily, comes to you for his first clinic visit. He notes that his BP at home is always less than
140/80 mmHg item, but in clinic it is always at least 155/95 mmHg. What is the next step?
A. Increase dose of thiazide
B. Addition of second antihypertensive medication
C. * Do nothing as he has white coat hypertension
D. Evaluate for secondary causes of hypertension
E. Start metoprolol
99. At a routine examination, an asymptomatic 46-year-old man is found to have a BP of 150/110
mmHg item, but no other abnormalities are present. What do you do next?
A. Reassure the patient and repeat the physical examination in 12 months
B. Initiate antihypertensive therapy
C. * Obtain repeated BP recordings in your office and/or the patient’s home or work site
D. Hospitalize patient for renal arteriography
E. Order a 24-h ambulatory BP monitoring
100. For the case below, select the most significant adverse effect of the antihypertensive and/or cardiac
agent in question: a 45-year old female has been on diuretic, but BP remains elevated at 145/95,
leading to the proposed addition of lisinopril. Which key potential adverse effect should be
discussed?
A. Increased triglyceride levels
B. Peripheral edema
C. Lupus-like syndrome
D. * Cough
E. Gynecomastia

101. For the case below, select the most significant adverse effect of the antihypertensive and/or cardiac
agent in question: a 58-year old male truck driver has significant hypertension, still not controlled
on a diuretic plus calcium channel blocker. Clonidine is being considered as the next medication,
but in this patients is concerned by sedation, sexual dysfunction.
A. Increased triglyceride levels
B. Cough
C. Gynecomastia
D. * Rebound hypertension
E. Urinary retention
102. In patients with a history of stroke or TIA the preferred drug combination is:
A. * ACE-inhibitor and diuretic
B. Calcium-channel blocker and beta-blocker
C. Beta-blocker and diuretic
D. Angiotensin receptor blocker and beta blocker
E. Beta- and alpha blocker
103. The initial antihypertensive medication recommended for patients who have no compelling
indications or contraindications is
A. ACE-inhibitor
B. Calcium-channel blocker
C. Diuretics
D. Beta blocker
E. * Any of the above
104. To reduce the patient’s cardiovascular morbidity and mortality, which therapy would you prescribe?
A. Hydralazine
B. Atenolol
C. * Losartan
D. Doxazosin
E. Clonidine
105. What is the appropriate course of action regarding the patient’s antihypertensive therapy?
A. Advise a low-sodium diet
B. * Finish doxazosin therapy and consider an alternative agent
C. Advise high dietary intake of calcium and potassium
D. Increase the doxazosin to 4 mg a day
E. Advise magnesium supplements
106. You see a diabetic patient presents with BP readings that are 155/95 or higher. All of the following
statements about the treatment of this patient's hypertension are correct EXCEPT:
A. Pharmacologic blocade of the renin-angiotensin system reduces the risk of both microvascular and
macrovascular events
B. Aggressive BP control reduces cardiovascular events more in diabetics than in nondiabetics
C. * Calcium channel blockers show no benefit in reducing cardiovascular events
D. The goal BP for this patient is <130/80 mmHg
E. All the above statements are correct
107. During the operation concerning strangular impassability of bowels, which conducted under
general anaesthesia of i/v with ALV, at a patient the stop of cardiac diyal-nosti happened 50 years in
the moment of mesenretium streching by the surgeon. What would prevent the stop of heart in this
A. * case?
i/v injection atropin
B. i/v injection of cardiac glycozidis
C. Deepening of general anaesthesi

D. Additional injection of relaxants


E. Additional injection droperidol
108. Patient 30 years after a road-transport failure complaints of the acute tahypno Ob-ly: a skin is
pale, cyanosi Hypodermic emphysema in the region of thorax, stomach, right side of the neck.
Auscultative: breathing on the right side is not conducted; pulse – 130/min., AP – 80/60
mm.mercury., CVP – 140 mm wt., FB – 30 /min., Ht – 0,27, Hb – 90 g/l. Subsequent therapy must
include above all things:
A. * punction of pleura cavity.
B. Urgent ALV
C. Massive infusion therapy of the crystalloid solutions
D. Infusion of dofamin, 2-5 mcg/cg/min
E. 100% oxygen
109. Patient has BP - 80/40 mm mercury, pulse - 120 per min, shock index for him:
A. * 120/80=1,5
B. 80+40)120=1
C. 80/120=0,67
D. 120/40=3
E. Not determined, as blood loss is unknown
110. Patient N., 47 years, treated in the hospital with the diagnosis: carbuncle of right kidney. Suddenly
general condition deteriorated, body temperature 39.50 C, skin cold, humid, consciousness kept,
expressed choking. Breath deep, noisy, 26/min. Pulse - 110/min., SC - 90/60 mm.mercury., oliguri
Which complications we can think about?
A. * Infectious-toxic shock
B. Vascular dystonia
C. Epilepsy
D. Anaphylactic shock
E. Orthostatic collapse
111. Patient, 40 years, with the trauma of both thighs is delivered from the scene of accident by a
“passing transport”. Objectively: cyanosys, rubor of the lower half of the neck, tahypnoe, AP 60/40
mm mer st., HBA=120 /min, in lungs moist wheezes, diuresis – 20 ml/h., Nb 100 g/l. Which from
the offered diagnoses most reliable?
A. * Fatty embolism
B. Traumatic shock
C. Hemorhagic shock
D. Pain shock
E. Tromboembolism
112. The patient 60 years is delivered in the department of intensive therapy with a diagnosis: bite of
bee, anafilactic shock.. Which medicine will be primary and most effective?
A. Adrenalin
B. Calcium the chloride
C. * Prednizolon
D. Dimedrol
E. Suprastin
113. The patient 25 years is hospitalized in the surgical department with a diagnosis: penetrable wound
of abdominal region. Objectively: it is excited, skin covers and visible mucous shells are pale;
peripheral pulse of the weak filling, frequent, AP – 110/60 mm mercury Positive symptom
of ”desolation” of peripheral hypodermic vein Diuresis is lowere How to characterize this state?
A. * The compensated shock
B. Preagonia

C. Circulating shock
D. Agony
E. Irreversible shock
114. The patient 36 years with ulcerous of gaster in anamnesis is hospitalized in the surgical department
with complaints on vomiting by "coffee-grounds", diarhea, moderately expressed thirst.
Objectively: a skin is pale, covered by a death-damp, a tongue is dry, AP – 80/60 mm rt.st., HBA –
120/min., BF – 28/min., diuresis – 25 ml/h. Blood test: Era - 2,8 1012/l, Hb – 98 g/l. What will be
most expedient in medical treatment:
A. * solutions with colloid
B. 5% solution of glucose
C. Whole blood
D. Red corpuscles mass
E. Colloid solutions with red corpuscles mass
115. 118. Patient 48 years the second day in dpartment of intensive therapy concerning the acute
front-partition heart attack of myocardium. During a review “wheezing” suddenly, non-permanent
tonic constricting of muscles is marked, pupils are extended, pulse on carotis not palpitat What
doctor have to do in the first place?
A. * Triple Safar method
B. Record of ECG
C. Cardial hit in the area of heart
D. Artificial respiration by the method of Silvester
E. Intracardial injection of adrenalin with an atropine
116. The patient entered the department with a diagnosis: acute intestinal impassability. Complaints:
insignificant thirst, dizziness at an attempt to get up from a be At a review: patient apathetical,
turgor is lowered, eyeballs are soft, tongue is dry with crack Pulse – 110 min., AP – 80 /60 mm of
mer item, diuresis – 25 ml /h. Electrolyte composition: Na+ - 142 mmol/l, C+ - 4 mmol/l,
glucose – 6 mmol/l, urea – 7 mmol/l. What variant of infusion is most expedient during operation?
A. * Transfusion of crystalloid
B. Transfusion of solution of glucos
C. Transfusion of albumen.
D. Transfusion of native plasm
E. Transfusion of poliglucin.
117. The patient, 28 years, 2 hours ago fell down from the ground floor of hous Sopor, pale, there are
the plural scratches of face, the lacerated hemorhagic wounds on the left forearm. The closed break
of the left shoulder and thigh. Pulse –110, Lc-10T/l, Hb – 100, AP – 90/40 mm of rt. item In the
blood test: red corpuscles – 3,5 g/l. What infouziyniy serednic does not need to be used for medical
treatment of shock?
A. * 5% solution of glucose
B. Solution of crystalloids
C. Solutions gelatin
D. Solutions of calcium
E. Solution of albumen
118. Anafilactic shock appeared at a patient. The state heavy and progressively gets wors HBA – 110
in a 1 minute, AP – 60/30 mm mercury. Prescribe medicine for the rescue of life of patient above all
things?
A. * Adrenalin.
B. Chloride of calcium.
C. Prednizolon.
D. Dofamin.

E. Suprastin.
119. At a patient 20 years on a background the injection of vitamin B1 suddenly there was excitation,
fear of death, falling of AP to 50 mm rt.st., hard breathing. Which of medicine it is necessary to
inject firstly?
A. * Adrenalin
B. Prednizolon
C. Calcium
D. Dimedrol
E. Eufilin
120. At a patient 60 years with the third day after an exterpation uterus acute insufficiency of breathing
developed suddenly, a skin became at first cyanotic, and then ash-colored color. Tachypnoe, cough
with bloody sputum, retrosternal pain. BP – 100/70 mm. mercury, HR – 120, BR – 32 in 1 min.,
CVP – 300 mm wt.col. What most reliable reason of worsening of the state of patient ?
A. * Tromboembolism of pulmonary artery
B. Bleeding
C. Pain shock
D. Hypostatic pneumonia
E. Heart attack of myocardium
121. At a patient in the ward of intensive therapy you marked appearance on the monitor of fibrillation
of ventricule Your first actions?
A. * To conduct defibrillation three times
B. To inject adrenalin
C. To inject a chloride
D. To begin the closed massage of heart
E. To inject lidocain
122. At a patient with the acute heart attack of myocardium best of all to warn relapsing fibrillation of
ventricles with :
A. cordaronum
B. lidocainum
C. ornidinum
D. * electrocardiostimulation
E. there is no right answer
123. At a patient with the acute heart attack of myocardium in the region of partition on a 5th day after
the brief episode of loss of consciousness there is reduction of frequency of pulse to 32 in a
minut BP - 80/40 mm Consciousness at the level of sopor. He immediately needs :
A. to put right a craniotserebral hypothermia, to enter lasics, prednisoloni, tserebrolizin
B. to enter an atropine, eoufilin, to begin infouziyo of aloupenta
C. * to conduct urgent cardiostimoulation
D. all answers are faithful
E. there is no right answer
124. At a patient, carried to a 2 year ago the heart attack of myocardium, the acute decline of
cholecystyties planned cholecystectomy, signs of electric instability of myocardium . Actions of
anaesthesiologist must include:
A. injection of prednisoloni, lidocaini, hyperventilation, take the ECG
B. * injection of dopamini, after stabilisation of BP - nitroglycerine + infusion therapy under the control
CVP, conducting of neurovegetative defence, take the ECG
C. injection of streptodecasol, stream infusion of reopoliglyocinum, injection of lidocainum, increase
of dose of analgetics

D. correctly A) and C)
E. all answers are faithful
125. At a patient, that is found on medical treatment in the therapeutic department, the sudden stopping
of circulation of blood happene Medical personnel begun the reanimation measure Define the
most rational way of injection of adrenalin for renewal of heart abbreviations in default of vein
A. * access:
To enter to a 3 ml solution of adrenalin in a trachea
B. To enter to a 1 ml solution of adrenalin in muscle
C. To enter adrenalin in muscle, multiplying a dose in 3 time
D. The intracardial injection.
E. Adrenalin can be not enterea
126. At a patient, that is found under the permanent electrocardioscopic supervision, microwave
fibrillation of myocardium and diagnosed clinical death develope It is necessary to do:
A. to inject the solution of calcium in cor
B. * to conduct high-voltage electric defibrillation
C. to inject solution of atropine in cor
D. to inject solution of adrenalin in cor
E. All answers are wrong
127. At the patient operated concerning the festering peritonitis coused by perforation of gastric ulcer, in
a postoperation period appeared: high temperature, frequency of breathing 35/min., AP – 70/40 mm
of mer item, diuresis -20 ml/h temperature of body to 39 , leucocitosi Transfusion during 12
hours 1,8 of a 0,9% solution of chlorous sodium and 0,8 of solution of reopoliglucin did not
improve general condition. Central vein pressure – 130 mm wt.st. For stabilization of
hemodinamics will be optimum infusion:
A. * Dopamin
B. Mezaton
C. Adrenalin
D. Noradrenalin
E. Ephedrine
128. At the ventilated patient with the edema of lungs at low pressure and septic shock intravenous
infusion 7,5 mcg/kg/min dopamini will increase
A. RaO2, saturation of oxygen of the mixed vein blood, consumption of oxygen
B. diuresis
C. * cardiac systolic volume
D. right A) and B)
E. faithful all answers
129. In 2 hours after renewal of cardial activity at a patient, that carried the sudden stop of heart on a
background hemorhagic shock (blood lost near 2,5 l) and is found on ALV, unstable hemorhagia
(AP – 80/40 – 90/60 mm of mer item, tahycardia)is marked, central vein pressure – 5 mm wt.st. It
is related to:
A. * By Hypovolume syndrome
B. By cardia insufficiency
C. By the inadequate interchange of gases
D. Vasoplegia
E. By the inadequate anaesthetizing
130. In a clinic a patient with the traumatic tearing of both lower extremities off at the level of
knee-joints is delivere A patient is extremly inert, languid, pale, pulse 140 bmin, threadlike, AP
500. On both lower extremities there are the imposed plait Bleeding at the receipt is not present.
From the words of doctor of first-aid, lost about 3 litres of blood in place of event. What principal
reason of heavy of the state of Patient?

A. * acute hemorrhag
B. Pain shock.
C. Ishemia of extremities as a result of application of tourniquet
D. Fatty embolism
E. acute kidney insufficiency
131. On a 4 day after incompatible (on a group) blood transfusion at a patient acutely reduced diouresis,
anuria developed, the common state became worse acutely, arterial pressure ros At laboratory
research: creatinin plasma – 680 mlmol/l, urea of plasma - 24 mmol/l. What illness and what stage
of illness it follows to think about in the first place?
A. * Acute kidney insufficiency, anuria
B. Anaphylactic shock, acute kidney insufficiency, anuria
C. Hemotransfusion shock, postrenal acute kidney insufficiency, anuria
D. Posthemoragic acute kidney insufficiency, anuria
E. acute interstitsial nephritis, postrenal anuria
132. Patient 20 years for verification of the functional state of kidneys the X- ray examination with v/v
injection of cardiotrast is conducte At the end of injection the state of patient acutely became
worse, the shortness of breath, hyperemia of skin, itch appeare AP – 60/20 mm of mer item,
HBA – 132/min. A similar research was conducted 3 months ago, such effects were not observe
What most reliable diagnosis?
A. * Medicinal anafilactic shock
B. Acute kidney insufficiency
C. Tromboembolism of pulmonary artery
D. Stress on the conducted manipulation
E. Heart attack miocardium
133. Patient 38 years, native plasma was poure At the end of infusion the state became worse: Patient
confused, excited, cyanosys, hypersalivation. Breathing frequency 36 on 1 min., AP – 70/40 mm of
mer item, whistling dry wheeze Which from the following mediceni must be injected firstly?
A. * Adrenalin.
B. Eufilin.
C. Suprastin.
D. Noradrenalin.
E. Prednizolon.
134. Patient 40 years with the acute gastro - intestinal bleeding a canned blood was poured in a volume
400 ml after conducting of all tests on compatibility. After hemotransfusion the state of patient
became worse, appeared head pains and pains in muscles athe temperature of body rose to 38,8
What can explaine the state of patient?
A. Pyroxene reaction of middle heavy
B. By development of hemotransfusion shock
C. * Allergic reaction
D. By development of bacterial-toxic shock
E. By air embolism
135. Patient 40 years with the acute gastro - intestinal bleeding a canned blood was poured in a volume
400 ml after conducting of all tests on compatibility. After hemotransfusion the state of patient
became worse, appeared head pains and pains in muscles athe temperature of body rose to 38,8
What can explaine the state of patient?
A. Pirogenic reaction of middle heavy
B. * By development of hemotransfusion shock
C. Alergic reaction
D. By development of bacterial-toxic shock

E. By air embolism
136. Patient 62 years the third day of presense in department of intensive therapy concerning the acute
transmural heart attack of myocardium of front-partition localization. At night woked up from a
suffocating cough, feeling of fear and troubl At a review: cyanosys, FB – 30 after 1 min., HBA –
132/ min., a rhythm is correct, tones of heart are deaf, accent ІІ tone above a pulmonary artery,
AP – 180/110 mm mercury. There is the loosened breathing above lungs with the far of
moist wheezes in lower fate What probably became the reason of worsening of the state?
A. * Edema of lungs
B. Embolism of pulmonary artery
C. The repeated heart attack miocardium
D. Hypertensive crisis
E. Attack of bronchial asthma
137. Patient L is hospitalized in gynecological department with the temperature of 39 degrees C, with
complaints of pain in the bottom of stomach, vomit, diarrhea . Criminal abortion have been done 4
days befor AP 80/60, breathing is difficult, psychosomatic excitation. Symptom of
Schotkin-blumberg is positiv Uterus is enlarged as on 9 weeks of pregnancy, limitedly mobile,
painles Pus with blood appeare Your Diagnosis?
A. * septic shock
B. Perforation of uterus
C. Pelvic peritonitis
D. Acute appendicitis
E. Acute adnexia inflammation
138. Patient N., 28 years ol 6 day after the complicated birth The clinical hematological signs of
subacute disseminate intravascular coagulation syndrome developed after skin hemorrhage and
uterine bleeding. The state of patient is very ba blood: Er-2,7 of T/l, Hb-78 of gm/l, CI - 0,93,
L-4,7 of Gm/l, thrombocytes-88 of gm/l, time of blood cloating - 16 min, prothrombin time - 25 sec,
ethanol test +, fibrinogen-1,4 gramme/l, What preparations should be prescribed ?
A. * freezed plasma
B. Heparinum
C. Reopoliglycin
D. Cryoprecipitate
E. U-aminokapric acid
139. Patient N., 40 years, groom. In anamnesis there is an allergy to nonsteriidal antiinflammatory. After
injection of antitetanus on a method Besredco concerning the hammered wound of right shin,
through 20 mines, there was a acute weakness, labouring breath, through 10 mines, loss of
consciousnes What mechanism of development of anafilactic form of illness?
A. * Sensitization to the albumen of horse whey
B. Low quality of horse whey
C. Breach of the technique of PPS injection
D. Presence in anamnesis of medical allergy
E. Infection of whey
140. Patient N., 40 years, groom. In anamnesis there is an allergy to nonsteroidal antyinflammatory.
After injection of antytetanus on a method Besredco concerning the hammered wound of right shin,
through 20 mines, there was a acute weakness, labouring breath, through 10 mines, loss of
consciousnes What mechanism of development of anafilactic form of illness?
A. * Sensitization to the albumen of horse whey
B. Low quality of horse whey
C. Breach of the technique of PPS injection
D. Presence in anamnesis of medical allergy
E. Infection of whey
141. Persons 48 years, patient by the heart attack of miocardium, suddenly lost consciousness, breathing
and palpitation. On ECG of highwave fibrilation of ventricule Conducted defibrilation. Did not
pick up normal cardial activity. What medicine needs to be entered for the rise of sensuality to
defibrillation?
A. * Amiodaron
B. Propranolon
C. Lidocain
D. Strofantin
E. Atropini sulfati
142. Pharmacological medicine, that diminish the (afterload) left ventricle at a patient with the acute
heart attack of myocardium, are not included
A. nitroglycerine
B. * strophantine
C. nitroproussid sodium
D. esmolol (brevibloc)
E. nifedipinum
143. Sick 46 years treated oneself in a therapeutic department with pneumonia of lower dole of right
lung. Planned antibacterial therapy - amoxiklav. After 40 min after intramuscular injection of duty
dose, the patients feeled dizziness, pain behind a breastbon AT 60/40 mm mer , pulse, - 120 a min.,
rhythmical. During examination of lungs: wheezes under both lung Temperature is 38,5 What
is worsening of the condition related to?
A. * Anaphylactic shock
B. Infectious toxic shock
C. Collapse
D. Tromboembolia of pulmonary artery
E. Infectious shock
144. The patient 20 years old, delivered to ambulance department on the 2nd day of illness in a grave
condition: temperature of body 39°c, symptoms of intoxication are expresse On extremities,
trunk, buttocks, present hemorrhagic rash as eczema with necrosis in the center. One day before cut
his leg. Now has the wound in that plac In 2 hours the decline of AP is registered from 100/70 to
60/30 mm of Hg, diffused cyanosi Application of prednisolon of 120 mg and reopolyglucin did not
give any effect. What complication does it follow to think about?
A. acute sub renal failure
B. * Septic shock
C. hypovolemic shock
D. Hemorragic shock
E. Respirator distress syndrom of adults
145. The patient 32 years have infusion of native plasm At the end of infusion the state became worse:
disorientation, cyanosys, excitation, appeared hypersalivation, tahypnoe, AP =70/40 mm mer st., in
lungs – the dissipated dry wheeze What medicine must be injected firstly?
A. * Adrenalin.
B. Suprastin.
C. Gidrocortizon.
D. Dopamin.
E. Eufilin.
146. To patient P., 50 years, with an unspecific ulcerous colitis with the purpose of correction of anaemia
transfusion of selfgroup blood 500 ml A(ІІ) the Rh(-) was conducte A doctor went out from
a chamber after conducting of necessary tests before hemotransfusion. In 20 minutes he
was quickly asked to the patient. Patient without consciousnes The cyanosys of upper
body part. Irregular breathing with the selection of a plenty of foamy, with the admixtures of
blood, phlegm. Pulse on peripheries and arterial pressure are not determine Tones of
heart are deaf, unrhythmical. An ampoule and transfusion system is empty. What
complication arose up as a result of hemotransfusion?
A. * Air embolism of pulmonary artery
B. Tromboembolism of pulmonary artery
C. Edema of lungs
D. Heart attack of myocardium
E. Syndrome of massive hemotransfusion
147. To the patient 45 years with suspicion on holecystitis the rentgencontrast i/v is quickly injecte
Tahycardiya, arterial hypotension, cyanosys, shortness of breath appeared, acute swelling of veins
of neck, extension of liver, CVT to 200 mm wt.st. acute insufficiency of what part of the
cardial-vascular system is observed at a patient?
A. * Right ventricle of heart
B. The left ventricle of heart
C. Both ventricles of heart
D. Vessels
E. Uneffective heart
Назва наукового напрямку (модуля): Семестр: 11
General questions in surgery 6 course
Опис:

Перелік питань:
1. In preparations for parenteral nutrition include:
A. plasma; Mr. casein hydrolysates;
B. albumin;
C. * Mr. casein hydrolysates
D. protein;
E. polivinilpirrolidon
2. In preparations for parenteral nutrition do not include:
A. * dextran
B. protein hydrolysates;
C. 10% glucose;
D. 20% glucose;
E. lipomays
3. In preparations for parenteral nutrition do not include:
A. * salt
B. hidrolizyn;
C. casein hydrolysates;
D. 10% glucose;
E. lipofundin
4. Frequently the cause of early complications after surgery using ditylinu:
A. laryngism;
B. inhibition of the respiratory center;
C. collapse;
D. * zapadinnya root of the tongue
E. Arrhythmia
5. Frequently the cause of early complications after surgery using Arduan:
A. * zapadinnya root of the tongue
B. inhibition of the respiratory center;
C. dehydration;
D. laryngism;
E. bronhiolospazm
6. Frequently the cause of early complications after surgery using mononarkozu ketaminom:
A. laryngism;
B. violations heart rate;
C. collapse;
D. * halyutsynoz, inadequate behavior
E. breathing "anarchy"
7. Frequently the cause of early complications after surgery using the central analgesia:
A. * depressed respiration
B. bronhiolospazm;
C. hypotension;
D. cardiac arrest;
E. acute liver failure
8. Immediately after the operation under anesthesia prozeryn vykorystovuyut to:
A. restore tone respiratory center;
B. removing the residual after applying kuraryzatsiyi depolyaryzuyuchyh muscle relaxants;
C. * removing the residual after applying kuraryzatsiyi antydepolyaryzuyuchyh muscle relaxants
D. stimulation of intestinal peristalsis;
E. Prevention bronhiolospazmu
9. Please list the required corrective surgery therapy
A. narcotic analgesics, antibiotics, cardiovascular drugs, electrolytes, vitamins;
B. correction of hemostasis, anesthesia, parenteral nutrition, exercise;
C. Correction pain, external respiration, volume hidremiyi, stimulation of peristalsis;
D. anesthesia, the use of central respiratory stimulants, antibiotics, drugs for parenteral breathing;
E. * correction of homeostasis, pain, antibacterial therapy, exercise therapy
10. Peritonitis is divided into:
A. * reactive, toxic, terminal
B. compensated, subkompensovanyy, terminal;
C. stage (I; II; III);
D. upper and lower half of the abdomen;
E. Early and late
11. In a nakrkozu in patients with peritonitis significantly increases the risk of such complications:
A. respiratory depression due to pulmonary edema;
B. bronhiolospazmu;
C. acute heart failure due to mediastinal shift raised diaphragm;
D. * regurgitation and aspiration
E. hiperkaliyemichnoyi cardiac arrest
12. The most frequent disorders of homeostasis in peritonitis are:
A. * metabolic acidosis
B. respiratory acidosis;
C. deep vein thrombosis of lower extremities, thromboembolism
D. respiratory alkalosis;
E. metabolic alkalosis
13. When jet peritonitis hemodynamics often seen:
A. collapse;
B. hipodynamichnym regime;
C. * hiperdynamichnym regime
D. hypovolemic shock;
E. depends on initial blood pressure
14. When toxic peritonitis Cardiac Output:
A. depends on the value of blood pressure;
B. increases;
C. does not change significantly;
D. * reduced
E. depends on the total peripheral vascular resistance
15. The optimal anesthetic during the operation y of patients with peritonitis are:
A. * with intravenous mioplehiyeyu and ventilation
B. Maskovyy inhalant;
C. epidural anesthesia;

D. spinal anesthesia;
E. conductor and infiltrative anesthesia
16. Acute intestinal obstruction causes are primarily disorders of homeostasis:
A. intoksykatsiynyy syndrome;
B. respiratory violations;
C. * hipohidratatsiya, dyzelektrolitemiya
D. acute hepatic, kidney failure;
E. septic state due to the phenomenon of translocation
17. Decompensated pilorostenoz especially dangerous:
A. intoksykatsiynym syndrome;
B. hiponatriyemiyeyu and hipohidratatsiyeyu;
C. Respiratory alkalosis and metabolic acidosis;
D. * Hypokalemia, hipohloremiyeyu, metabolic alkalosis
E. acute adrenal insufficiency
18. Intestinal obstruction significantly increased risk:
A. * regurgitation and aspiration
B. septic state;
C. multiple organ failure;
D. liver failure;
E. painful shock
19. What is characterized by acute destructive pancreatitis?
A. drop intoksykatsiynoho cardiac output due to myocardial lesions;
B. metabolic and respiratory acidosis;
C. * hipohidratatsiyeyu, hypotension
D. compensatory hipertenziynym syndrome;
E. Injuring hepatocytes
20. The most desirable preparation for infusion therapy of pancreatitis are:
A. reosorbilakt;
B. polihlyukin;
C. starch derivatives;
D. izotonichnymy Mr. sodium chloride;
E. * glucose solution
21. The most frequent complication of pancreatic necrosis are:
A. * pankreatohennyy shock
B. acute heart failure;
C. sepsis;
D. acute liver failure;
E. pulmonary edema
22. What is under ICE - Syndrome?
A. primary, consumption coagulopathy, anemia;
B. hypercoagulation, consumption coagulopathy, abnormal fibrinolysis, resolution;
C. hypercoagulation, hipokoahulyatsiyi;
D. * hypercoagulation, consumption coagulopathy, abnormal fibrinolysis, pixels
E. offset, subkompensated, decompensated
23. The most reasonable treatment for drug-ICE syndrome are:
A. Kriopretsipitat;

B. erytrotsytna weight;
C. whole blood;
D. Refortan;
E. * quick-frozen plasma
24. When expressed anemia caused by acute massive hemorrhage in obstetric practice, primarily to:
A. * eliminate hypovolemia
B. pour erytrotsytnu mass;
C. enter hemostatyky;
D. Kriopretsipitat apply;
E. ensure the infusion of fibrinogen
25. To be carried out by blood tests?
A. now the whole blood transfusion is therefore not necessary to conduct tests;
B. determining blood type, biological samples;
C. to save time and Rh blood group-membership can be estimated by documented data (in passport),
to conduct a biological sample;
D. blood group O (I) Rh (-) is universal for transfusions for any recipient;
E. * blood grouping, Rh accessories, group and individual compatibility of biological samples
26. Which of the following belong to the mullion components of blood?
A. antystafilokokova plasma antyhemofilna plasma erytrotsytna mass, fibrinogen, Kriopretsipitat;
B. native plasma mass erytrotsytna, washed erythrocytes, Kriopretsipitat, polibiolin;
C. All types of plasma fibrinogen, washed erythrocytes, leykotsytna weight, albumin;
D. * native plasma mass erytrotsytna, washed red cells, platelet mass
E. native plasma mass erytrotsytna, washed erythrocytes, aminokrovin, polibiolin, thrombus and
leykomasa
27. Which of the symptoms characteristic for diagnosis hemotransfuziynoho complications due to
incompatible system AB (0) during anesthesia?
A. znobinnya patient;
B. sudden pulmonary edema;
C. expressed hyperthermia;
D. acute anuria;
E. * motiveless sudden hypotension
28. What happens in the bloodstream of the patient with an incompatible blood transfusion him?
A. thrombus;
B. * hemolysis
C. arteriospazm acute;
D. hypertension due hiperkateholaminemiyi;
E. Go beyond the liquid part of blood vessel wall
29. Severe anemia occurs when a massive loss of blood volume:
A. 30% of bcc;
B. * 40% of the bcc
C. 50% of bcc;
D. 60% of bcc;
E. 70% of the bcc
30. Bcc in adults is:
A. 1 / 10 of body weight;
B. 5-6% of body weight;
C. 5000 ml;

D. * 7% of body weight
E. 1 / 20 on body weight
31. The reason for ICE - Syndrome can be:
A. incompatible blood transfusion;
B. massive hemorrhage;
C. septic state;
D. amniotic fluid embolism;
E. * All listed conditions
32. Laboratory features of 1-under ICE - syndrome are:
A. decrease in fibrinogen;
B. reduce the clotting time by Lee-White;
C. reduction of bleeding by Dyuk'om;
D. reduction in prothrombin time;
E. * all listed features
33. How is the diagnosis of the general peritonitis set to the operation?
A. roentgenologic
B. anamnestetic
C. * by laboratory determination the signs of inflammatory reaction
D. on clinical signs
E. on the level secretion the gastric juice
34. For the late stage of peritonitis all is characteristic, except for:
A. swelling of stomach
B. hypovolemia
C. disappearance of intestinal noises
D. * hypoproteinemia
E. increased peristalsis
35. Diffusive festering peritonitis can be investigation of all transferred diseases, except for:
A. perforations Meckel's diverticulum
B. destructive appendicitis
C. * stenosis of large duodenal nipple
D. Richter strangulation of hernia
E. acute intestinal impassability
36. Fibrinogenous impositions on a peritoneum are not at peritonitis:
A. * fibrinogenous
B. festering
C. putrid
D. excrement
E. serosal
37. The exsudate painted blood in an abdominal region is observed always, except for:
A. * tubercular peritonitis
B. violations of extra-uterine pregnancy
C. mesenteric ischemia
D. acute pancreatitis
E. twisted oothecoma
38. Middle laparotomy must be conducted at:
A. * diffusive peritonitis

B. local unlimited peritonitis


C. abscess of Duglas space
D. periappendiceal infiltration
E. acute appendicitis
39. . The best method of treatment the subhepatic abscess is:
A. thoracolaparotomy
B. lumbotomy
C. double-stage transpleural approach
D. laparotomy by Fedorov
E. * extrapleural extra-peritoneal method
40. Inexpressive leucocytosis in acute appendicitis is characteristic for:
A. * elderly patients
B. females
C. children
D. pregnant
E. males
41. For the acute appendicitis, complicated by appendicular infiltrate, in contrast to the tumour of
caecum, is characteristic:
A. * Tendency to diminishing of the tumour in the process of supervision
B. Long-term anamnesis
C. Excretion of blood from rectum
D. Curvuasier's sign
E. Frequent partial intestinal obstruction in anamnesis
42. The conditions, which contribute to the formation of appendicular infiltrate include:
A. * Phlegmonous changes of appendix
B. Chronic appendicitis
C. Meckel's diverticulum
D. Pylephlebitis
E. Perforation of appendix
43. Only during the operation is possible the differential diagnostics of acute appendicitis with:
A. * terminal ileitis
B. renal colic
C. acute pyelonephritis
D. acute paraproctitis
E. acute pancreatitis
44. After appendectomy for pregnant is recommended
A. * Application of abortion prophylaxis.
B. More frequent use of peritoneal dialysis
C. Active postoperative period
D. More rare use of peritoneal dialysis
E. More prolonged draining of the abdominal cavity
45. The distinctive peculiarities of acute appendicitis in the second half of pregnancy are:
A. * Weak express of pain syndrome, similar to the ligamentary tension of uterus
B. Absence of Volkovcha-Kocher's sign
C. Expressed signs of peritoneal irritation
D. The express local muscular tension in a right iliac area

E. Expressed of Obraztsov's sign


46. For the differential diagnostics of acute appendicitis with the urology diseases is not used
A. * Irrigoscopy
B. Urography
C. Cystochromoscopy
D. X-ray of kidneys
E. Urine analysis
47. In the diagnostics of pelvic appendicitis the most valuable is:
A. * rectal and vaginal examination
B. laboratory analyses
C. laparocentesis
D. laparoscopy
E. colonoscopy
48. For the retrocaecal appendicitis is not typical:
A. * Volkovcha-Kocher's sign
B. delayed diagnostics
C. late entrance of patients in the hospital
D. frequent development of destructive forms
E. weak expressed signs of peritoneal irritation
49. For the perforation of appendix is not characteristic:
A. * Decrease of body temperature
B. Acute pain in a right iliac area, especially expressed after false improvement
C. Tension of the abdominal wall at first in a right iliac area, and then spreading on other departments
D. Increasing swelling of abdomen
E. Leucocytosis
50. Initially-gangrenous appendicitis differs from inflammatory-gangrenous form mostly developing in
persons:
A. * of elderly age
B. children of early age
C. pregnant in the first half of pregnancy
D. pregnant in the second half of pregnancy
E. with concomitant diseases
51. What does the Bartomier-Mikhelson's sign mean?
A. * The increase of pain intensity during the palpation of right iliac area when the patient lies on the
left side.
B. Increased pain with coughing
C. Pain in right lower quadrant during palpation of left lower quadrant
D. Increase of pain in a right iliac area when the patient lies on the left side
E. Migration of pain to the right iliac area from epigastric
52. What does the Blumberg's sign mean?
A. * The sharp increase of pain quick taking off the hand during palpation of anterior abdominal wall.
B. Increased pain with coughing
C. Pain in right lower quadrant during palpation of left lower quadrant
D. Increase of pain in a right iliac area when the patient lies on the left side
E. Migration of pain to the right iliac area from epigastric
53. What does the Voskresenky’s sign mean?

A. * The increase of pain during quick sliding movements by the tips of fingers from epigastric to right
iliac area.
B. Increased pain with coughing
C. Pain in right lower quadrant during palpation of left lower quadrant
D. Increase of pain in a right iliac area when the patient lies on the left side
E. Migration of pain to the right iliac area from epigastric
54. What does the Rozdolsky’s sign mean?
A. * Painfulness in a right iliac area during percussion.
B. Increased pain with coughing
C. Pain in right lower quadrant during palpation of left lower quadrant
D. Increase of pain in a right iliac area when the patient lies on the left side
E. Migration of pain to the right iliac area from epigastric
55. What does the Yaure-Rozanov sign mean?
A. * Painfulness during palpation of Petit triangle
B. Increased pain with coughing
C. Pain in right lower quadrant during palpation of left lower quadrant
D. Increase of pain in a right iliac area when the patient lies on the left side
E. Migration of pain to the right iliac area from epigastric
56. What does the Gabay’s sign mean?
A. * Blumberg’s sign in Petit triangle
B. Increased pain with coughing
C. Pain in right lower quadrant during palpation of left lower quadrant
D. Increase of pain in a right iliac area when the patient lies on the left side
E. Migration of pain to the right iliac area from epigastric
57. What does the psoas sign mean?
A. * Pain on extension of right thigh
B. Increased pain with coughing
C. Pain in right lower quadrant during palpation of left lower quadrant
D. Increase of pain in a right iliac area when the patient lies on the left side
E. Migration of pain to the right iliac area from epigastric
58. The sign of gas migration is called:
A. * Rovsing's sign
B. Kocher’s sign
C. Sitkovsky’s sign
D. Bartomier’s sign
E. Dunphy's sign
59. The Rovsing's sign is typical for:
A. * Simple appendicitis
B. Retrocecal appendicitis
C. Retroperitoneal appendicitis
D. Pelvic appendicitis
E. Left-side appendicitis
60. The Sitkovsky’s sign is typical for:
A. * Simple appendicitis
B. Retrocecal appendicitis
C. Retroperitoneal appendicitis

D. Pelvic appendicitis
E. Left-side appendicitis
61. The Bartomier’s sign is typical for:
A. * Simple appendicitis
B. Retrocecal appendicitis
C. Retroperitoneal appendicitis
D. Pelvic appendicitis
E. Left-side appendicitis
62. The Dunphy's sign is typical for:
A. * Simple appendicitis
B. Retrocecal appendicitis
C. Retroperitoneal appendicitis
D. Pelvic appendicitis
E. Left-side appendicitis
63. The Blumberg’s sign is typical for:
A. * Phlegmonous appendicitis
B. Simple appendicitis
C. Retrocecal appendicitis
D. Retroperitoneal appendicitis
E. Pelvic appendicitis
64. The Voskresenky’s sign is typical for:
A. * Phlegmonous appendicitis
B. Simple appendicitis
C. Retrocecal appendicitis
D. Retroperitoneal appendicitis
E. Pelvic appendicitis
65. The Rozdolsky’s sign is typical for:
A. * Phlegmonous appendicitis
B. Simple appendicitis
C. Retrocecal appendicitis
D. Retroperitoneal appendicitis
E. Pelvic appendicitis
66. The Yaure-Rozanov sign is typical for:
A. * Retrocecal appendicitis
B. Phlegmonous appendicitis
C. Simple appendicitis
D. Left-side appendicitis appendicitis
E. Pelvic appendicitis
67. The Gabay’s sign is typical for:
A. * Retrocecal appendicitis
B. Phlegmonous appendicitis
C. Simple appendicitis
D. Left-side appendicitis appendicitis
E. Pelvic appendicitis
68. The Pasternatsky’s sign is typical for:
A. * Retrocecal appendicitis

B. Phlegmonous appendicitis
C. Simple appendicitis
D. Left-side appendicitis appendicitis
E. Pelvic appendicitis
69. The psoas sign is typical for:
A. * Retrocecal appendicitis
B. Phlegmonous appendicitis
C. Simple appendicitis
D. Left-side appendicitis appendicitis
E. Pelvic appendicitis
70. The expressed pain in a right lumbar area is typical for:
A. * Retrocecal appendicitis
B. Phlegmonous appendicitis
C. Simple appendicitis
D. Left-side appendicitis appendicitis
E. Pelvic appendicitis
71. The dysuria is typical for:
A. * Pelvic appendicitis
B. Retrocecal appendicitis
C. Phlegmonous appendicitis
D. Simple appendicitis
E. Left-side appendicitis appendicitis
72. The pulling rectal pain is typical for:
A. * Pelvic appendicitis
B. Retrocecal appendicitis
C. Phlegmonous appendicitis
D. Simple appendicitis
E. Left-side appendicitis appendicitis
73. The tenesmi are typical for:
A. Left-side appendicitis appendicitis
B. * Pelvic appendicitis
C. Retrocecal appendicitis
D. Phlegmonous appendicitis
E. Simple appendicitis
74. The absence of muscular tenderness is typical for:
A. * Pelvic appendicitis
B. Retrocecal appendicitis
C. Phlegmonous appendicitis
D. Simple appendicitis
E. Left-side appendicitis appendicitis
75. The painfulness of anterior rectal wall is typical for:
A. * Pelvic appendicitis
B. Retrocecal appendicitis
C. Phlegmonous appendicitis
D. Simple appendicitis
E. Left-side appendicitis appendicitis

76. Painfulness of posterior vaginal vault is typical for:


A. * Pelvic appendicitis
B. Retrocecal appendicitis
C. Phlegmonous appendicitis
D. Simple appendicitis
E. Left-side appendicitis appendicitis
77. The painfulness of the left iliac region is typical for:
A. * Left-side appendicitis appendicitis
B. Pelvic appendicitis
C. Retrocecal appendicitis
D. Phlegmonous appendicitis
E. Simple appendicitis
78. For the simple appendicitis is typical:
A. * Rovsing's sign
B. Blumberg's sign
C. Yaure-Rozanov sign
D. Voskresensky's sign
E. Kulenkampf's sign
79. For the simple appendicitis is typical:
A. * Sitkovsky’s sign
B. Blumberg's sign
C. Yaure-Rozanov sign
D. Voskresensky's sign
E. Kulenkampf's sign
80. For the simple appendicitis is typical:
A. * Bartomier’s sign
B. Blumberg's sign
C. Yaure-Rozanov sign
D. Voskresensky's sign
E. Kulenkampf's sign
81. For the simple appendicitis is typical:
A. * Dunphy's sign
B. Blumberg's sign
C. Yaure-Rozanov sign
D. Voskresensky's sign
E. Kulenkampf's sign
82. For the retrocecal appendicitis is typical:
A. * Yaure-Rozanov sign
B. Rovsing's sign
C. Blumberg's sign
D. Voskresensky's sign
E. Kulenkampf's sign
83. For the retrocecal appendicitis is typical:
A. * Gabay’s sign
B. Rovsing's sign
C. Blumberg's sign

D. Voskresensky's sign
E. Kulenkampf's sign
84. For the retrocecal appendicitis is typical:
A. * Pasternatsky’s sign
B. Rovsing's sign
C. Blumberg's sign
D. Voskresensky's sign
E. Kulenkampf's sign
85. For the retrocecal appendicitis is typical:
A. * Psoas sign
B. Rovsing's sign
C. Blumberg's sign
D. Voskresensky's sign
E. Kulenkampf's sign
86. For the retrocecal appendicitis is typical:
A. * Expressed pain in a right lumbar area
B. Flank tenderness in right lower quadrant
C. The painfulness of the left iliac region
D. Clinic of irritation of pelvic organs
E. Painfulness of anterior rectal wall and posterior vaginal vault
87. For the retroperitoneal appendicitis is typical:
A. * Flank tenderness in right lower quadrant
B. Peritoneal signs
C. The painfulness of the left iliac region
D. Clinic of irritation of pelvic organs
E. Painfulness of anterior rectal wall and posterior vaginal vault
88. For the left-side appendicitis is typical:
A. * The painfulness of the left iliac region
B. Expressed pain in a right lumbar area
C. Flank tenderness in right lower quadrant
D. Clinic of irritation of pelvic organs
E. Painfulness of anterior rectal wall and posterior vaginal vault
89. For the pelvic appendicitis is typical:
A. * Clinic of irritation of pelvic organs
B. The painfulness of the left iliac region
C. Expressed pain in a right lumbar area
D. Flank tenderness in right lower quadrant
E. Peritoneal signs
90. For the pelvic appendicitis is typical:
A. * Painfulness of anterior rectal wall
B. The painfulness of the left iliac region
C. Expressed pain in a right lumbar area
D. Flank tenderness in right lower quadrant
E. Peritoneal signs
91. For the pelvic appendicitis is typical:
A. * Painfulness of posterior vaginal vault

B. The painfulness of the left iliac region


C. Expressed pain in a right lumbar area
D. Flank tenderness in right lower quadrant
E. Peritoneal signs
92. Characteristic changes in the general blood analysis in appendicitis:
A. * neutrophil leucocytosis with deviation of the differential count to the left
B. neutrophil lymphocytosis with deviation of the differential count to the left
C. neutrophil eosonophilia with deviation of the differential count to the left
D. neutrophil leucocytosis with deviation of the differential count to the right
E. white cells neutrophilia with deviation of the differential count to the right
93. The most informing method of instrumental diagnostics of acute appendicitis is:
A. * tomography
B. esophagogastroscopy
C. colonoscopy
D. gastroscopy
E. contrasting roentgenoscopy
94. The most informing method of instrumental diagnostics of acute appendicitis is:
A. * ultrasound examination
B. contrasting roentgenoscopy
C. gastroscopy
D. esophagogastroscopy
E. colonoscopy
95. Acute appendicitis in the 1st phase is necessary to differentiate from:
A. * gastric ulcer
B. pancreatitis
C. cholecystitis
D. intestinal obstruction
E. strangulated hernia
96. Appendicular infiltrate is treated:
A. * conservative therapy, then surgery
B. only conservative therapy
C. puncture
D. drainage
E. only surgical treatment
97. Appendicular infiltrate is treated:
A. * antibiotics, paranephral blockade, detoxication therapy
B. antiseptics, analgesia, antibiotics, anti-inflammatory therapy
C. antibiotics, diuretics, antispasmodic, anti-inflammatory therapy
D. analgesia, antibiotics, diuretics, anti-inflammatory therapy
E. anti-inflammatory drugs, paranephral blockade, detoxication therapy
98. Appendicular infiltrate appears after:
A. * 3-5 days
B. 1-2 days
C. 5-6 days
D. 7-8 days
E. 8-10 days

99. Conservative treatment of appendicular infiltrate is going on:


A. * 1-2 weeks
B. 1 week
C. 3-4 weeks
D. 1-2 months
E. 2-4 months
100. Appendectomy after the treatment of appendicular infiltrate performed after:
A. * 2-4 months
B. 1-2 weeks
C. 3-4 weeks
D. 1-2 months
E. 3-5 days
101. For appendectomy the most suitable surgical access is:
A. * Volkovich-Dyakonov
B. McBurney
C. Lenander
D. Sprengel
E. Kocher
102. The removal of appendix from apex - is :
A. * antegrade appendectomy
B. retrograde appendectomy
C. retrocecal appendectomy
D. antececal appendectomy
E. laparoscopic appendectomy
103. The removal of appendix from the base is:
A. * Retrograde appendectomy
B. antegrade appendectomy
C. retrocecal appendectomy
D. antececal appendectomy
E. laparoscopic appendectomy
104. On the line between the anterior-superior process of the iliac bone and umbilicus located the point:
A. * McBurney's
B. Kalk's
C. Kehr's
D. Lenander's
E. Volkovich-Dyakonov
105. The point through which the Volkovich-Dyakonov access is performed located on the line between
anterior-superior process of the iliac bone and umbilicus:
A. * between external and middle third
B. between external and internal third
C. in the internal third
D. in the middle
E. in external third
106. Which method of appendectomy is used in children before age 3?
A. * ligation
B. amputation

C. retrograde
D. antegrade
E. laparoscopic
107. Modern method of appendectomy is:
A. * laparoscopic
B. microlaparotomy
C. laparocentesis
D. laparotomy
E. ligation
108. After appendectomy the patient stands out of bed on:
A. * first day
B. second day
C. third day
D. fourth day
E. fifth day
109. During appendectomy the most frequent complication is:
A. * bleeding
B. infiltrate
C. leak of the sutures
D. infecting
E. peritonitis
110. After appendectomy to early postoperative complications belongs:
A. * peritonitis
B. intestinal fistula
C. ventral hernia
D. ligature fistula
E. colitis
111. Appendectomy, as a rule, is performed under such anaesthesia:
A. * intravenous anaesthesia
B. local anaesthesia
C. ether anaesthesia
D. conducting anaesthesia
E. endotracheal anaesthesia
112. Pain during palpation in a lumbar region - is the sign:
A. * Yaure-Rozanov sign
B. Sitkovsky's sign
C. Obrastsow's sign
D. Voskresensky's sign
E. Kulenkampf's sign
113. Pain during palpation in the Petit triangle - is the sign:
A. * Yaure-Rozanov sign
B. Sitkovsky's sign
C. Obrastsow's sign
D. Voskresensky's sign
E. Kulenkampf's sign
114. Pain during palpation in a lumbar region after taking away of the hand is the sign:

A. * Gabay's sign
B. Sitkovsky's sign
C. Obrastsow's sign
D. Voskresensky's sign
E. Kulenkampf's sign
115. For retrocecal appendicitis is characteristic the sign:
A. * Yaure-Rozanov sign
B. Sitkovsky's sign
C. Obrastsow's sign
D. Voskresensky's sign
E. Kulenkampf's sign
116. For retrocecal appendicitis is characteristic the sign:
A. * Gabay's sign
B. Sitkovsky's sign
C. Obrastsow's sign
D. Voskresensky's sign
E. Kulenkampf's sign
117. For retroperitoneal appendicitis is characteristic the sign:
A. * Pasternatsky's
B. Sitkovsky's
C. Yaure-Rozanov
D. Rovzing's
E. Koer's
118. Microhematuria is typical for such kind of appendicitis:
A. * retroperitoneal
B. retrocecal
C. pelvic
D. subhepatic
E. left-side
119. The pelvic appendicitis manifests by:
A. * dysurination
B. dyspepsia
C. hyperthermia
D. hematuria
E. dystrophy
120. The pelvic appendicitis manifests by:
A. * tenesmi
B. spasms
C. myalgia
D. paresis
E. enuresis
121. For pelvic appendicitis is characteristic the sign:
A. * Kulenkampf's sign
B. Yaure-Rozanov sign
C. Sitkovsky's sign
D. Obrastsow's sign

E. Voskresensky's sign
122. The pelvic appendicitis manifests by:
A. * diarrhea
B. vomiting
C. constipation
D. nausea
E. colicks
123. The undiagnosed destructive appendicitis complicated by:
A. * infiltrate
B. fistula
C. adhesions
D. bleeding
E. colic
124. Causes of the appendicular infiltrate development:
A. * late hospitalisation, misdiagnosed appendicitis
B. aggressive infection, impaired immunity
C. adhesions, increased immunity
D. peritonitis, abscessing
E. surgical trauma, infection
125. The clinical manifestation of appendicular infiltrate is:
A. * swelling
B. the signs of peritoneal irritation
C. muscular tension
D. high temperature
E. leucocytosis
126. Tumour with fluctuation are the main clinical manifestation of:
A. * appendicular abscess
B. appendicular peritonitis
C. appendicular infiltrate
D. appendicular mesadenitis
E. appendicular typhlitis
127. The most frequent complications of appendicitis are:
A. * infiltrate, abscess, pilephlebitis, peritonitis
B. infiltrate, abscess, thrombophlebitis, hepatitis
C. conglomerate, adhesions, cystitis, peritonitis
D. infiltrate, conglomerate, hepatitis
E. abscess, peritonitis, adhesions, phlebitis
128. The peculiarities of the clinical course of appendicitis in children are caused:
A. * by the bailer form of appendix
B. by the tubular form of appendix
C. by hypertrophy of appendix
D. by atrophy of appendix
E. by the spherical form of appendix
129. Lymphoid hypoplasia determines the peculiarities of the clinical course of appendicitis in:
A. * children
B. elderly patients

C. pregnant
D. males
E. females
130. The pain all over the whole abdomen in acute appendicitis is characteristic for:
A. * children
B. females
C. pregnant
D. males
E. elderly patients
131. The omental hypoplasia influences on the peculiarities of the course of acute appendicitis in:
A. * children
B. females
C. males
D. pregnant
E. elderly patients
132. Dyspeptic syndrome is characteristic for acute appendicitis in:
A. * children
B. females
C. males
D. pregnant
E. elderly patients
133. The rapid spread of inflammatory process in acute appendicitis is characteristic for:
A. * children
B. females
C. males
D. pregnant
E. elderly patients
134. Reduced reactivity of the organism influences on the peculiarities of the course of acute
appendicitis in:
A. * elderly patients
B. females
C. males
D. pregnant
E. children
135. Rapid destruction of the appendix in the course of acute appendicitis is characteristic for:
A. * elderly patients
B. children
C. pregnant
D. males
E. females
136. Inexpressive abdominal pain in acute appendicitis is characteristic for:
A. Children
B. * elderly patients
C. females
D. males
E. pregnant

137. Inexpressive muscular tension of anterior abdominal wall in acute appendicitis is characteristic for:
A. * elderly patients
B. females
C. children
D. pregnant
E. males
138. Inexpressive leucocytosis in acute appendicitis is characteristic for:
A. * elderly patients
B. females
C. children
D. pregnant
E. males
139. The expressed deviation of the differential leukocyte count to the left in acute appendicitis is
characteristic for persons :
A. females
B. * elderly patients
C. males
D. pregnant
E. children
140. The clinical manifestation of acute appendicitis does not relate to destructive changes in the
appendix in:
A. children
B. * elderly patients
C. females
D. males
E. pregnant
141. The destructive changes in the appendix don't relate to the clinical manifestation of acute
appendicitis in:
A. children
B. females
C. males
D. * elderly patients
E. pregnant
142. The clinical manifestation of acute appendicitis in pregnancy depends on:
A. * the term of pregnancy
B. degree of inflammatory changes
C. the relation of appendix to peritoneum
D. the duration of appendicitis
E. the form of appendicitis
143. The clinical manifestations of acute appendicitis in the first trimester of the pregnancy are:
A. * typical
B. atypical
C. expressed
D. unexpressed
E. absent
144. The clinical manifestations of acute appendicitis in the second trimester of the pregnancy are:
A. * typical

B. atypical
C. expressed
D. unexpressed
E. absent
145. The clinical manifestations of acute appendicitis in the third trimester of the pregnancy are:
A. * atypical
B. typical
C. expressed
D. unexpressed
E. absent
146. The clinical manifestations of acute appendicitis in pregnancy are characterised by the changes of:
A. * localization of pain
B. severity of pain
C. irradiation of pain
D. duration of pain
E. character of pain
147. The changes of clinical manifestations of acute appendicitis in pregnancy are caused by:
A. * distension of anterior abdominal wall by uterus
B. inflammation of uterus
C. irritation of anterior abdominal wall by uterus
D. compression of appendix by uterus
E. inflammation of the right ovarium
148. The changes of clinical manifestations of acute appendicitis in pregnancy are caused by:
A. * absence of muscular tension of anterior abdominal wall
B. absence of tension of the uterus
C. presence of tension of the uterus
D. expressed muscular tension of anterior abdominal wall
E. presence of tension of peritoneum of anterior abdominal wall
149. The changes in clinical manifestation of acute appendicitis in pregnancy is characterized :
A. * by the absence of signs of peritoneal irritation
B. by the presence of signs of peritoneal irritation
C. by the presence of expressed signs of peritoneal irritation
D. by displacement of the signs of peritoneal irritation
E. by the change of the character of signs of peritoneal irritation
150. What is the medical tactic of the acute appendicitis in pregnant:
A. * to operate
B. to prescribe antibiotics
C. to prescribe conservative therapy
D. to observe
E. to interrupt pregnancy
151. The changes of clinical manifestations of acute appendicitis in pregnancy are caused by the
displacement of appendix in relation to cecum:
A. * upword
B. lateral
C. downword
D. medial

E. retroperitoneal
152. What is the lethality in acute appendicitis caused by?
A. * late hospitalization
B. tactical errors
C. concomitant diseases
D. technical errors during an operation
E. severity of disease
153. The bailer form of appendix is characteristic for:
A. * new-born
B. males
C. females
D. pregnant
E. elderly patients
154. The appendix ends its formation at the age of:
A. * 7 years
B. 6 months
C. 1 year
D. 3 years
E. 3 months
155. The purulent inflammation of portal vein as the complication of acute appendicitis - is:
A. * pilephlebitis
B. mesadenitis
C. tiphlitis
D. thrombophlebitis
E. adnexitis
156. The most informative for differentiation of appendicitis with a basal pleurisy is:
A. * X-ray film
B. percussion
C. tomography
D. auscultation
E. bronchoscopy
157. The most informative for differentiation of appendicitis with an epigastric form of myocardial
infarction are the changes in:
A. * ECG
B. hemodynamic disturbances
C. expressed shortness of breath
D. auscultation
E. tachycardia
158. The most informative for differentiation of appendicitis with intercostal neuralgia is:
A. * paravertebral blockade
B. laparoscopy
C. microlaparotomy
D. laparocentesis
E. peridural blockades
159. The most informative for differentiation of appendicitis with food poisoning is:
A. * frequent vomit

B. single vomit
C. nausea
D. increased peristalsis
E. slow peristalsis
160. The most informative for differentiation of appendicitis with gastric phlegmon is:
A. * esophagogastroscopy
B. roentgenoscopy
C. palpation
D. laparocentesis
E. ultrasound examination
161. The most informative for differentiation of appendicitis with perforative ulcer of duodenum is:
A. * absence of hepatic dullness
B. presence of hepatic dullness by percussion
C. absence of the splenic dullness
D. presence of a high tympanic sound by percussion
E. absence of the gastric dullness
162. The most informative for the differentiation of appendicitis with cholecystitis is:
A. * ultrasound examination
B. X-ray film
C. anamnesis
D. laparocentesis
E. laparoscopy
163. The most informative for the differentiation of appendicitis with pancreatitis is:
A. * ultrasound examination
B. blockades
C. laparostomy
D. laparoscopy
E. X-ray film
164. The most informative for the differentiation of appendicitis with intestinal obstruction is:
A. * X-ray film
B. ultrasound examination
C. blockade
D. laparotomy
E. laparoscopy
165. What form of the appendicitis results in the developing of fibrosis of the appendix?
A. * chronic
B. phlegmonous
C. catarrhal
D. gangrenous
E. perforative
166. What form of the appendicitis results in the obliteration of the appendix??
A. * chronic
B. phlegmonous
C. catarrhal
D. gangrenous
E. perforative

167. Chronic primary appendicitis - is the development of pathological changes in appendix after:
A. * without the signs of acute appendicitis in anamnesis
B. acute appendicitis
C. appendicular infiltrate
D. appendicular abscess
E. pilephlebitis
168. To the chronic secondary appendicitis belongs:
A. * residual
B. catarrhal
C. empyema
D. phlegmonous
E. gangrenous
169. Chronic residual appendicitis arises up after:
A. * acute appendicitis
B. chronic appendicitis
C. colicks
D. recurrent appendicitis
E. primary chronic appendicitis
170. Chronic residual appendicitis arises up after:
A. * appendicular infiltrate
B. chronic appendicitis
C. recurrent appendicitis
D. colicks
E. primary chronic appendicitis
171. Chronic residual appendicitis arises up after:
A. * appendicular abscess
B. colicks
C. chronic appendicitis
D. recurrent appendicitis
E. primary chronic appendicitis
172. The most prominent clinical sign of chronic appendicitis is:
A. * pain by deep palpation
B. pain by percussion
C. pain by superficial palpation
D. skin hyperesthesia
E. pain by bimanual palpation
173. What form of appendicitis the signs of peritoneal irritation are absent in?
A. * chronic
B. calculous
C. perforative
D. appendicular infiltrate
E. appendicular abscess
174. Hyperaemia, thickening, oedema of appendix are the signs of:
A. * catarrhal appendicitis
B. phlegmonous appendicitis
C. gangrenous appendicitis

D. gangreno-perforating appendicitis
E. dystrophic appendicitis
175. Hyperemia, fibrino-purulent fur, pus the lumen are the signs of:
A. * phlegmonous appendicitis
B. catarrhal appendicitis
C. gangrenous appendicitis
D. gangreno-perforative appendicitis
E. dystrophic appendicitis
176. Black colour, fibrino-purulent fur, perforation are the signs of:
A. * gangreno-perforative appendicitis
B. phlegmonous appendicitis
C. gangrenous appendicitis
D. catarrhal appendicitis
E. dystrophic appendicitis
177. The peculiarities of the clinical course of appendicitis in children are caused:
A. * by the bailer form of appendix
B. by the tubular form of appendix
C. by hypertrophy of appendix
D. by atrophy of appendix
E. by the spherical form of appendix
178. Lymphoid hypoplasia determines the peculiarities of the clinical course of appendicitis in:
A. * children
B. elderly patients
C. pregnant
D. males
E. females
179. The pain all over the whole abdomen in acute appendicitis is characteristic for:
A. * children
B. females
C. pregnant
D. males
E. elderly patients
180. The omental hypoplasia influences on the peculiarities of the course of acute appendicitis in:
A. * children
B. females
C. males
D. pregnant
E. elderly patients
181. Dyspeptic syndrome is characteristic for acute appendicitis in:
A. * children
B. females
C. males
D. pregnant
E. elderly patients
182. For acute appendicitis typical t° is:
A. * 38° С

B. 37° С
C. subfebrile
D. 37-39° С
E. 38-40° С
183. The rapid spread of inflammatory process in acute appendicitis is characteristic for:
A. * children
B. females
C. males
D. pregnant
E. elderly patients
184. Reduced reactivity of the organism influences on the peculiarities of the course of acute
appendicitis in:
A. * elderly patients
B. females
C. males
D. pregnant
E. children
185. Rapid destruction of the appendix in the course of acute appendicitis is characteristic for:
A. * elderly patients
B. children
C. pregnant
D. males
E. females
186. Inexpressive abdominal pain in acute appendicitis is characteristic for:
A. children
B. females
C. males
D. pregnant
E. * elderly patients
187. Inexpressive muscular tension of anterior abdominal wall in acute appendicitis is characteristic for:
A. females
B. children
C. pregnant
D. males
E. * elderly patients
188. What kind of acute intestinal obstruction the invagination belongs to?
A. * Mixed
B. Paralytic
C. Volvulus
D. Strangulation
E. Dynamic
189. Invagination much more frequent in:
A. * Children
B. Pregnant
C. Elderly people
D. Teenagers

E. Does not depend on age


190. What is the most frequent localization of invagination:
A. * The region of cecum
B. Splenic angle
C. Hepatic angle
D. Rectosygmoid angle
E. Patients with the Led's syndrome
191. The most frequent cause of the large intestinal obstruction is:
A. * Tumours
B. Invagination
C. Volvulus
D. Hemorrhoids of IV degree
E. Errors in the diet
192. The first phase of the clinical course of acute intestinal obstruction lasts:
A. * To 12 hours
B. To 2 hours
C. To 1 days
D. More than 1 day
E. To 1 hour
193. The leading signs in acute intestinal obstruction are:
A. * Wave-like pain, vomiting, delay of gases and stool
B. „Knife-like” pain, wooden abdomen, proper anamnesis
C. „Knife-like” pain, wooden abdomen, vomiting
D. Wave-like pain, anaemia
E. Nausea, loss of appetite, metallic taste in the mouth
194. Name the character of peristalsis in the onset of the acute intestinal obstruction:
A. * Hyperperistalsis
B. Normal peristalsis
C. Absent
D. Variable
E. Heard only in regions upper the obstruction
195. The Sklyarov's sign in acute intestinal obstruction is:
A. * Noise of splash
B. Good heard cardiac tones during auscultation of the abdomen
C. Dullness in the lower regions
D. Sound of falling drop
E. Gaping of anus
196. The Grekov's sign in acute intestinal obstruction is:
A. * Gaping of anus
B. Good heard cardiac tones during auscultation of the abdomen
C. Dullness in the lower regions
D. Sound of falling drop
E. Noise of splash
197. In acute intestinal obstruction the basic X-ray sign is:
A. * Air-fluid levels, Kloiber's cups
B. Expressed limitation of mobility of the right dome of diaphragm

C. Diffusely dilated loops of bowels


D. Free gas in the abdomen
E. Sklyarov's sign
198. The Spasokukotsky's sign in acute intestinal obstruction is:
A. * Sound of falling drop
B. Good heard cardiac tones during auscultation of the abdomen
C. Dullness in the lower regions
D. Noise of splash
E. Gaping of anus
199. What are the Kloiber's cups?
A. * Horizontal air-fluid levels
B. Gas bubble of the stomach
C. Folds of intestine
D. Gas sickles under the domes of diaphragm
E. None of mentioned
200. Name the method of examination which is not obligatory in acute intestinal obstruction:
A. * All are obligatory
B. General blood analysis
C. General urine analysis
D. Coagulogramm
E. Electrolytes
201. Is obligatory the X-ray examination at suspicion on acute intestinal obstruction?
A. * Yes
B. No, if you know that acute intestinal obstruction is of obturative origin
C. Yes, if you know that acute intestinal obstruction is of obturative origin
D. No
E. Yes, except for children and pregnant
202. Is obligatory the digital examination of rectum at suspicion on acute intestinal obstruction?
A. * Yes
B. No, if you know that acute intestinal obstruction is of obturative origin
C. Yes, if you know that acute intestinal obstruction is of obturative origin
D. No
E. Yes, except for children, pregnant
203. The purpose of conservative therapy in compensated acute intestinal obstruction:
A. * All mentioned
B. Preoperative preparation
C. Treating
D. Detoxication
E. Diagnostic
204. The purpose of conservative therapy in decompensated acute intestinal obstruction:
A. * Preoperative preparation
B. Treating
C. Detoxication
D. All mentioned
E. None of mentioned
205. What does not belong to conservative therapy of acute intestinal obstruction?

A. * Liquidation of hypervolemia
B. Decompression of gastrointestinal tract
C. The struggle against abdominal-pain shock
D. Detoxication
E. Correction of microcirculation
206. What does not belong to the fight against abdominal-pain shock?
A. * Performing of siphon enema
B. Paranephral novocaine blockade
C. Neuroleptanalgesia
D. Peridural anaesthesia
E. Spasmolytic therapy
207. The decompression of gastrointestinal tract includess everything, except:
A. * Lavage of abdominal cavity
B. Endoscopic intubation
C. Enterotomy with aspiration
D. Washing of the stomach
E. Performing of siphon enema
208. Name duration of conservative treatment of acute intestinal obstruction in the stage of
compensation?
A. * 5-7 days
B. 1-2 days
C. 12-24 hours
D. To 12 hours
E. Not less than 2 weeks
209. Treatment of patients with acute intestinal obstruction in the stage of decompensation must be:
A. * 2-4 hours of conservative, then operative
B. To 24 hours of conservative, then operative
C. Immediately operative
D. During the first days conservative treatment with the gradual increase of volume of infusion
E. Conservative in ambulatory conditions
210. The treatment of patients with strangulation acute intestinal obstruction which accompanied by the
manifestations of peritonitis must include:
A. * 2 hours of conservative treatment, then operative
B. To 12 hours conservative treatment, then operative
C. Immediately operative without conservative
D. Conservative in ambulatory conditions
E. During the first days conservative with the gradual increase of volume infusion
211. The criteria of the efficiency of gastrointestinal tract passage renewal during conservative therapy
of acute intestinal obstruction is:
A. * Pulling of gases and stool
B. Normalization of rectal temperature
C. Absence of Shchotkin-Blumberg's sign
D. Feeling of heartburn
E. None of mentioned
212. To the criteria of permanent renewal of the gastrointestinal tract passage as efficiency of
conservative treatment belongs:
A. * Absence of stagnant content in the stomach

B. Absence of Shchotkin-Blumberg's sign


C. Normalization of rectal temperature
D. Feeling of heartburn
E. None of mentioned
213. The absolute indication for operative treatment of acute intestinal obstructionє:
A. * III phase of the course of acute intestinal obstruction
B. II phase of the course of acute intestinal obstruction
C. I phase of the course of acute intestinal obstruction
D. The prolonged anamnesis of acute intestinal obstruction
E. Dynamic acute intestinal obstruction
214. The indication for operative treatment of acute intestinal obstruction is:
A. * Mechanical acute intestinal obstruction in inefficient conservative treatment
B. I phase of the course of acute intestinal obstruction
C. II phase of the course of acute intestinal obstruction
D. The prolonged anamnesis of acute intestinal obstruction
E. Mechanical acute intestinal obstruction
215. In what case the drainage of the abdominal cavity is inadvisable in operative treatment of acute
intestinal obstruction?
A. * None of mentioned cases
B. In formation of anastomosis
C. In formation of haematoma
D. In formation of stoma
E. In all these cases
216. When is the operative intervention for acute intestinal obstruction accompanied by the drainage of
abdominal cavity?
A. * In all mentioned cases
B. In formation of stoma
C. In increased bleeding during dissecting of adhesions
D. In formation of anastomosis
E. None of mentioned cases
217. Is the programmable laparostomy suitable in the treatment of І-ІІ stage of acute intestinal
obstruction?
A. * No
B. Yes
C. Only in the case of formation of anastomosis
D. Only in strangulation acute intestinal obstruction
E. Only in obturation acute intestinal obstruction
218. Optimal access in the operative treatment of acute intestinal obstruction is:
A. * Middle laparotomy
B. Phanenstil's
C. Vinkelman's
D. Fedorov's
E. Right pararectal
219. Choose the correct algorithm of the operative intervation for the II stage of acute intestinal
obstruction :
A. * Laparotomy, liquidation of obstruction, intestinal intubation, sanation of abdominal cavity, suturing
of the abdomen

B. Laparotomy, liquidation of the source of peritonitis, sanation of abdominal cavity, suturing of the
abdomen
C. Laparotomy, liquidation of obstruction, sanation of abdominal cavity, suturing of the abdomen
D. Laparotomy, liquidation of obstruction, intestinal intubation, sanation of abdominal cavity,
laparostomy
E. Laparotomy, liquidation of obstruction, liquidation of the source of peritonitis, intestinal intubation,
sanation of abdominal cavity, suturing of the abdomen
220. Choose the correct algorithm of operative intervation for the III stage of acute intestinal
A. * obstruction:
Laparotomy, liquidation of the source of peritonitis, intestinal intubation, sanation of abdominal
cavity, suturing of the abdomen or laparostomy
B. Laparotomy, liquidation of obstruction, intestinal intubation, sanation of abdominal cavity, suturing
of the abdomen
C. Laparotomy, liquidation of obstruction, intestinal intubation, sanation of abdominal cavity,
laparostomy
D. Laparotomy, liquidation of obstruction, intestinal intubation, sanation of abdominal cavity, suturing
of the abdomen
E. Laparotomy, liquidation of obstruction, liquidation of peritonitis, sanation of abdominal cavity,
suturing of the abdomen
221. Arterial mesenteric acute intestinal obstruction belongs to:
A. * Obturation
B. Strangulation
C. Paralytic
D. Spastic
E. Mixed
222. What is the essence of arterial mesenteric intestinal obstruction?
A. * Superior mesenteric artery compresses the duodenum
B. Duodenum compresses the superior mesenteric artery
C. Acute intestinal obstruction on the background of mesenteric thrombosis
D. Mesenteric thrombosis caused by obstruction
E. Duodenum compresses inferior mesenteric artery
223. What treatment is indicated in gall-stones intestinal obturation?
A. * Only operative
B. Only conservative
C. Operative in the case of the development of peritonitis
D. Treatment is not required
E. Tactic depends on the size of stone
224. The tumour obturation of cecum requires:
A. * Right-side hemicolectomy
B. Resection of cecum
C. Cecostomy
D. Only ileostomy
E. Only intubation of small intestine
225. What treatment tactic of acute intestinal obstruction, caused by a tumour obturation is required?
A. * Operative intervation
B. Liquidation of tumour by a chemotherapy
C. Liquidations of tumour by radiotherapy
D. Operative intervation only after chemotherapy
E. Only symptomatic treatment

226. What is the volvulus?


A. * Torsion of the bowel with its mesentery along longitudinal axis
B. Torsion of the bowel with the mesentery of another loop
C. Invagination of one part of the bowel in another
D. Obturation of the bowel lumen
E. Torsion of the bowel with its mesentery along transverse axis
227. The most frequently the sygmoid volvulus arises in:
A. * Elderly patients with frequent constipations
B. Females with menstrual arrest
C. Children
D. Elderly patients people with permanent diarrhea
E. New-borns
228. What is the aim of the operative treatment of volvulus if the bowel „alive”?
A. * Detorsion, decompression, fixing to the abdominal wall
B. Detorsion, resection, fixing to the abdominal wall
C. Detorsion, dilation, decompression, fixing to the abdominal wall
D. Detorsion, dilation, decompression
E. Decompression, fixing to the abdominal wall
229. The nodulus requires:
A. * Untie the knot, if impossible – resection of the bowel
B. Resection of the bowel
C. Untie the knot
D. To perform the stoma. The second stage the resection of the bowel
E. None of mentioned
230. A typical sign for invagination in irrigoscopy is:
A. * „Cockades”
B. „Candles”
C. „Rat tail”
D. Spizharny's sign
E. Bartomier-Mikhelson's sign
231. The indication for cecopexia in the operative treatment of invagination is:
A. * For the prophylaxis of relapses
B. For self desinvagination
C. For better desinvagination
D. Is not indicated
E. Not performed
232. The peritonitis, caused by perforation of gastric ulcer is characterised by such type of obstruction:
A. * Paralytic
B. Spastic
C. Strangulation
D. There is no characteristic type
E. The obstruction can not develop in this case
233. The peritonitis, caused by perforation of duodenal ulcer is characterised by such type of
A. * obstruction:
Paralytic
B. Spastic
C. Strangulation

D. There is no characteristic type


E. The obstruction can not develop in this case
234. Describe the Kloiber's cups in small intestinal obstruction:
A. * Wide, not high, maltiple
B. Not wide, high, single
C. Not characteristic
D. Wide, not high, with folds
E. Of different size, localization
235. Describe the Kloiber's cups in large intestinal obstruction:
A. * Not wide, high, single
B. Wide, not high, maltiple
C. Not characteristic
D. Wide, not high, with folds
E. Of different size, localization
236. The air-fluid levels (Kloiber's cups) are not characteristic for such type of acute intestinal
obstruction, as:
A. * Spastic
B. Paralytic
C. Obturation
D. Invagination
E. All kinds
237. The "trident", "crescent" signs are characteristic for such type of acute intestinal obstruction, as:
A. * Invagination
B. Spastic
C. Obturation
D. Strangulation
E. All kinds
238. Which type of acute intestinal obstruction is connected with previous operations:
A. * Strangulation
B. Spastic
C. Obturation
D. Invagination
E. All kinds
239. For strangulation is not typical:
A. * Normal body temperature
B. Tension of abdominal wall
C. Leucocytosis
D. Frequent vomit
E. Wahl's symptom
240. For strangulation is not typical:
A. * Leucopenia
B. Tension of abdominal wall
C. Frequent vomit
D. Body temperature 37,5°C and higher
E. Wahl's symptom
241. The contributory factor of the development of strangulation is:

A. * Long intestinal mesentery


B. Stool stones
C. Gall-stones
D. Tumour
E. None of mentioned
242. The contributory factor of the development of obturation is:
A. * Stool stones
B. Long intestinal mesentery
C. Adhesions in abdominal cavity
D. All of mentioned
E. None of mentioned
243. Where is the pain localized in acute appendicitis?
A. * Right iliac region
B. Epigastric region
C. Left iliac region
D. Left subcostal region
E. Right lumbar region
244. Where does the pain arise in the onset of acute appendicitis?
A. * Epigastric region
B. Left iliac region
C. Right iliac region
D. Left subcostal region
E. Right lumbar region
245. Where does the pain irradiate in acute appendicitis?
A. * Not irradiate
B. Lumbar region
C. Left iliac region
D. Right scapular
E. Perineum
246. What dyspeptic manifestations are typical for acute appendicitis?
A. * Single nausea and vomiting
B. Constant vomiting and nausea without any relief
C. Vomiting by bile without any relief
D. Absence of peristalsis
E. Constant diarrhea
247. What objective manifestations are typical for acute appendicitis?
A. * Muscular tension in a right iliac area
B. Abdominal distension
C. Absence of hepatic dullness
D. Absence of peristalsis
E. Rigidity of anterior abdominal wall
248. What signs are typical for phlegmonous appendicitis in contrast to simple appendicitis?
A. * Peritoneal signs
B. Signs of gas migration
C. Signs of pain migration
D. Muscular tension in a right iliac area

E. Nausea and vomiting


249. What sign is typical for phlegmonous appendicitis in contrast to simple appendicitis?
A. * Blumberg's sign
B. Kocher’s sign
C. Bartomier’s sign
D. Sitkovsky’s sign
E. Dunphy's sign
250. What sign is typical for phlegmonous appendicitis in contrast to simple appendicitis?
A. * Voskresenky's sign
B. Sitkovsky’s sign
C. Bartomier’s sign
D. Kocher’s sign
E. Dunphy's sign
251. What does the Voskresenky's sign mean?
A. * Increase of pain during quick sliding movements by the tips of fingers from epigastric to right iliac
area
B. Pain in right lower quadrant during palpation of left lower quadrant
C. Increase of pain in a right iliac area when the patient lies on the left side
D. Increased pain with coughing
E. Migration of pain to the right iliac area from epigastric
252. What does the Rozdolsky’s sign mean?
A. * Painfulness in a right iliac area during percussion
B. Pain in right lower quadrant during palpation of left lower quadrant
C. Increase of pain in a right iliac area when the patient lies on the left side
D. Increased pain with coughing
E. Migration of pain to the right iliac area from epigastric
253. What signs are typical for gangrenous appendicitis in contrast to simple appendicitis?
A. * Signs of intoxication
B. Signs of gas migration
C. Retention of stool or single diarrhea
D. Muscular tension in a right iliac area
E. Single nausea and vomiting
254. Who usually suffer from gangrenous appendicitis?
A. People of old age
B. * Newborns
C. Children
D. Pregnant women
E. Young men
255. Where is the pain localized in retrocaecal appendicitis?
A. * Right lumbar region
B. Right iliac region
C. Epigastric region
D. Left iliac region
E. Left subcostal region
256. What objective manifestations are typical for retrocaecal appendicitis?
A. * Pain and muscular rigidity in a right iliac area during palpation

B. Abdominal distension
C. Absence of hepatic dullness
D. Clinic of retroperitoneal phlegmon
E. Rigidity of anterior abdominal wall
257. What sign is typical for retrocaecal appendicitis in contrast to simple appendicitis?
A. * Pasternatsky’s sign
B. Kocher’s sign
C. Bartomier’s sign
D. Sitkovsky’s sign
E. Dunphy's sign
258. What sign is typical for retrocaecal appendicitis in contrast to simple appendicitis?
A. * Psoas sign
B. Sitkovsky’s sign
C. Bartomier’s sign
D. Kocher’s sign
E. Dunphy's sign
259. What does the Pasternatsky’s sign mean?
A. * Tapping of lumbar region cause the pain
B. Pain in right lower quadrant during palpation of left lower quadrant
C. Increase of pain in a right iliac area when the patient lies on the left side
D. Increased pain with coughing
E. Migration of pain to the right iliac area from epigastric
260. What does the Yaure-Rozanov sign mean?
A. * Painfulness during palpation of Petit triangle
B. Pain in right lower quadrant during palpation of left lower quadrant
C. Migration of pain to the right iliac area from epigastric
D. Tapping of lumbar region cause the pain
E. Increase of pain in a right iliac area when the patient lies on the left side
261. What does the Gabay’s sign mean?
A. * Blumberg’s sign in Petit triangle
B. Pain in right lower quadrant during palpation of left lower quadrant
C. Migration of pain to the right iliac area from epigastric
D. Tapping of lumbar region cause the pain
E. Increase of pain in a right iliac area when the patient lies on the left side
262. What does the psoas-sign mean?
A. * Pain on extension of right thigh
B. Painfulness during palpation of Petit triangle
C. Migration of pain to the right iliac area from epigastric
D. Tapping of lumbar region cause the pain
E. Increase of pain in a right iliac area when the patient lies on the left side
263. What manifestation is predominant for retroperitoneal appendicitis?
A. * Clinic of retroperitoneal phlegmon
B. Clinic of acute abdomen
C. Dyspeptic syndrome
D. Clinic of acute intestinal obstruction
E. Clinic of acute pancreatitis

264. What manifestation is predominant for pelvic appendicitis?


A. * Clinic of irritation of pelvic organs (dysuria, pulling rectal pain, tenesmi)
B. Clinic of acute abdomen
C. Clinic of retroperitoneal phlegmon
D. Clinic of acute intestinal obstruction
E. Clinic of acute pancreatitis
265. What manifestation is typical for pelvic appendicitis?
A. * Absence of muscular tenderness
B. Clinic of retroperitoneal phlegmon
C. Clinic of acute intestinal obstruction
D. Clinic of acute abdomen
E. Clinic of acute pancreatitis
266. What objective manifestations are typical for retrocaecal appendicitis?
A. * Painfulness of anterior rectal wall and posterior vaginal vault
B. Abdominal distension
C. Absence of hepatic dullness
D. Clinic of retroperitoneal phlegmon
E. Rigidity of anterior abdominal wall
267. Where is the pain localized in left-side appendicitis?
A. * Left iliac region
B. Epigastric region
C. Right iliac region
D. Left subcostal region
E. Right lumbar region
268. What clinical picture is typical for appendicitis in children?
A. * Clinic of destructive forms of appendicitis and intoxication
B. Abdominal distension
C. Absence of dyspeptic manifestation
D. Absence of muscular tenderness
E. Clinic of acute intestinal obstruction
269. What does the examination of infant children in acute appendicitis require to use?
A. * Chloralhydrate enema
B. Contrast enema
C. Siphon enema
D. Cleaning enema
E. X-ray with barium swallow
270. What complication is typical for acute appendicitis?
A. * Appendicular infiltrate
B. Appendicular bleeding
C. Acute intestinal obstruction
D. Appendicular-intestinal fistula
E. Malignization
271. What complication is typical for acute appendicitis?
A. * Appendicular abscess
B. Appendicular bleeding
C. Acute intestinal obstruction

D. Appendicular-intestinal fistula
E. Malignization
272. What is the treatment of appendicular infiltrate?
A. * Conservative treatment
B. Draining operation
C. Appendectomy
D. Hemicolectomy
E. Caecostomy
273. Typical complications of the appendicitis are:
A. * infiltrate, abscess, peritonitis, pilephlebitis
B. abscess, phlegmon, paraproctitis, pilephlebitis
C. infiltrate, gangrene, paraproctitis, pilephlebitis
D. abscess, phlegmon, peritonitis, pilephlebitis
E. infiltrate, abscess, osteomyelitis, pilephlebitis
274. For acute appendicitis is typical:
A. * Kocher-Volkovitch's sign
B. Ortner's sign
C. Homans sign
D. Sklyarov's sign
E. Meyo-Robson sign
275. For acute appendicitis is typical:
A. * Rovsing's sign
B. Ortner's sign
C. Homans sign
D. Sklyarov's sign
E. Meyo-Robson sign
276. For acute appendicitis is typical:
A. * Sitkovsky’s sign
B. Ortner's sign
C. Homans sign
D. Sklyarov's sign
E. Meyo-Robson sign
277. For acute appendicitis is typical:
A. * Bartomier’s sign
B. Ortner's sign
C. Homans sign
D. Sklyarov's sign
E. Meyo-Robson sign
278. For acute appendicitis is typical:
A. * Dunphy's sign
B. Ortner's sign
C. Homans sign
D. Sklyarov's sign
E. Meyo-Robson sign
279. For acute appendicitis is typical:
A. * Blumberg’s sign

B. Ortner's sign
C. Homans sign
D. Sklyarov's sign
E. Meyo-Robson sign
280. For acute appendicitis is typical:
A. * Voskresenky’s sign
B. Ortner's sign
C. Homans sign
D. Sklyarov's sign
E. Meyo-Robson sign
281. For acute appendicitis is typical:
A. * Rozdolsky’s sign
B. Ortner's sign
C. Homans sign
D. Sklyarov's sign
E. Meyo-Robson sign
282. For acute appendicitis is typical:
A. * Yaure-Rozanov's sign
B. Ortner's sign
C. Homans sign
D. Sklyarov's sign
E. Meyo-Robson sign
283. For acute appendicitis is typical:
A. * Gabay’s sign
B. Ortner's sign
C. Homans sign
D. Sklyarov's sign
E. Meyo-Robson sign
284. For acute appendicitis is typical:
A. * Psoas sign
B. Ortner's sign
C. Homans sign
D. Sklyarov's sign
E. Meyo-Robson sign
285. For acute appendicitis is typical:
A. * Obrastsow's sign
B. Ortner's sign
C. Homans sign
D. Sklyarov's sign
E. Meyo-Robson sign
286. Kocher-Volkovitch's sign is typical for:
A. * acute appendicitis
B. acute cholecystitis
C. acute intestinal obstruction
D. food poisoning
E. acute pancreatitis

287. Rovsing's sign is typical for:


A. * acute appendicitis
B. acute cholecystitis
C. acute intestinal obstruction
D. food poisoning
E. acute pancreatitis
288. Sitkovsky’s sign is typical for:
A. * acute appendicitis
B. acute cholecystitis
C. acute intestinal obstruction
D. food poisoning
E. acute pancreatitis
289. Bartomier’s sign is typical for:
A. * acute appendicitis
B. acute cholecystitis
C. acute intestinal obstruction
D. food poisoning
E. acute pancreatitis
290. Dunphy's sign is typical for:
A. * acute appendicitis
B. acute cholecystitis
C. acute intestinal obstruction
D. food poisoning
E. acute pancreatitis
291. Blumberg’s sign is typical for:
A. * acute appendicitis
B. acute thrombophlebitis
C. pneumothorax
D. food poisoning
E. bleeding ulcer
292. Rozdolsky’s sign is typical for:
A. * acute appendicitis
B. acute thrombophlebitis
C. pneumothorax
D. food poisoning
E. bleeding ulcer
293. Voskresenky’s sign is typical for:
A. * acute appendicitis
B. acute thrombophlebitis
C. pneumothorax
D. food poisoning
E. bleeding ulcer
294. Yaure-Rozanov's sign is typical for:
A. * acute appendicitis
B. acute thrombophlebitis
C. pneumothorax

D. food poisoning
E. bleeding ulcer
295. Gabay’s sign is typical for:
A. * acute appendicitis
B. acute thrombophlebitis
C. pneumothorax
D. food poisoning
E. bleeding ulcer
296. Psoas sign is typical for:
A. * acute appendicitis
B. acute thrombophlebitis
C. pneumothorax
D. food poisoning
E. bleeding ulcer
297. Obrastsow's sign is typical for:
A. * acute appendicitis
B. acute thrombophlebitis
C. pneumothorax
D. food poisoning
E. bleeding ulcer
298. Pain in the iliac region during elevation of the leg - is:
A. * Obrastsow's sign
B. Sitkovsky's sign
C. Kocher-Volkovitch's sign
D. Bartomier-Mikhelson's sign
E. Voskresensky's sign
299. Pain during sliding of hand on abdominal wall - is:
A. * Voskresensky's sign
B. Sitkovsky's sign
C. Obrastsow's sign
D. Kocher-Volkovitch's sign
E. Bartomier-Mikhelson's sign
300. Strengthening of pain in right iliac region on the left side - is:
A. * Sitkovsky's sign
B. Obrastsow's sign
C. Voskresensky's sign
D. Kocher-Volkovitch's sign
E. Bartomier-Mikhelson's sign
301. Pain in the right iliac region during palpation of the iliac region on the left side - is:
A. * Bartomier-Mikhelson's sign
B. Sitkovsky's sign
C. Obrastsow's sign
D. Voskresensky's sign
E. Kocher-Volkovitch's sign
302. Pain during digital examination of rectum - is:
A. * Kulenkampf's sign

B. Obrastsow's sign
C. Voskresensky's sign
D. Kocher-Volkovitch's sign
E. Sitkovsky's sign
303. Pain during percussion by fingers of anterior abdominal wall - is:
A. * Rozdolsky's sign
B. Obrastsow's sign
C. Sitkovsky's sign
D. Voskresensky's sign
E. Kulenkampf's sign
304. Pain in the right iliac region by pushing of the left - is:
A. * Rovsing's sign
B. Sitkovsky's sign
C. Obrastsow's sign
D. Voskresensky's sign
E. Kulenkampf's sign
305. Pain during removing of the hand from abdominal wall after its pressing - is:
A. * Shchotkin-Blumberg's sign
B. Rovsing's sign
C. Sitkovsky's sign
D. Obrastsow's sign
E. Kulenkampf's sign
306. Migration of pain to the right iliac area from epigastric is:
A. * Kocher-Volkovitch's sign
B. Rovsing's sign
C. Sitkovsky's sign
D. Obrastsow's sign
E. Kulenkampf's sign
307. Increased pain with coughing is:
A. * Dunphy's sign
B. Rovsing's sign
C. Sitkovsky's sign
D. Obrastsow's sign
E. Kulenkampf's sign
308. Painfulness during palpation of Petit triangle is:
A. * Yaure-Rozanov's sign
B. Rovsing's sign
C. Sitkovsky's sign
D. Obrastsow's sign
E. Kulenkampf's sign
309. Blumberg’s sign in Petit triangle is:
A. * Gabay’s sign
B. Rovsing's sign
C. Sitkovsky's sign
D. Obrastsow's sign
E. Kulenkampf's sign
310. Pain on extension of right thigh is:
A. * Psoas sign
B. Rovsing's sign
C. Sitkovsky's sign
D. Obrastsow's sign
E. Kulenkampf's sign
311. What does the Kocher’s sign mean?
A. * Migration of pain to the right iliac area from epigastric
B. Pain in right lower quadrant during palpation of left lower quadrant
C. Increase of pain in a right iliac area when the patient lies on the left side
D. The increase of pain intensity during the palpation of right iliac area when the patient lies on the
left side.
E. Increased pain with coughing
312. What does the Rovsing's sign mean?
A. * Pain in right lower quadrant during palpation of left lower quadrant
B. Increase of pain in a right iliac area when the patient lies on the left side
C. The increase of pain intensity during the palpation of right iliac area when the patient lies on the
left side.
D. Increased pain with coughing
E. Migration of pain to the right iliac area from epigastric
313. What does the Dunphy's sign mean?
A. * Increased pain with coughing
B. Pain in right lower quadrant during palpation of left lower quadrant
C. Increase of pain in a right iliac area when the patient lies on the left side
D. The increase of pain intensity during the palpation of right iliac area when the patient lies on the
left side.
E. Migration of pain to the right iliac area from epigastric
314. In what vein is a venous outflow carried out in from a stomach?
A. * V. Portae;
B. V. odd;
C. V. pair;
D. V. overhead hollow;
E. V. lower hollow;
315. What time urgent operations are executed at acute bleeding
A. * 6 – 12 hours;
B. 6 – 10 hours;
C. 6 – 8 hours
D. 6 – 14 hours;
E. 6 – 20 hours
316. Esophagogastroduodenoscopy can find out next changes in a stomach, except for
A. tumours
B. ulcers
C. bleeding polypuses
D. erosions
E. * changes of evacuation function
317. Hemobilia is
A. * all answers are correct;
B. bleeding the bilious ways and liver;
C. bleeding the general bilious channel;
D. bloody clot in the big duodenal papilla;
E. all answers are not correct.
318. Giant ulcer is an ulcer measuring
A. over 4,5 cm
B. * over 3 cm
C. over 4 cm
D. over 5 cm
E. over 3,5 cm
319. Duration the period of primary shock at a perforeted ulcer
A. * 3-6 hours
B. 6-12 hours
C. 1-3 hours
D. 12-24 hours
E. 24-36 hours
320. For bleeding ulcer characteristically
A. * melena
B. tension the muscles of front abdominal wall
C. Spazarskiy’s sing
D. sickliness the back vault of vagina
E. irradiation pain in a shoulder or shoulder-blade
321. For bleeding ulcer characteristic sign is
A. pain in an epigastrium;
B. knife-like pain;
C. signs irritation of peritoneum;
D. presence fresh blood in incandescence
E. * melena;
322. For perforeted ulcer characteristically
A. * tension the muscles of front abdominal wall
B. melena
C. vomiting by coffee-grounds
D. high intestinal impassability
E. vomiting stagnant gastric maintenance
323. For pneumoperitoneum is characteristic symptom
A. * Zhober’s;
B. Khelatid’s;
C. Podlag’s;
D. Vigats’s;
E. Udin’s.
324. For the heavy flow of ulcerous illness characteristically
A. 2 and anymore relapses on a year
B. * 3 and anymore relapses on a year
C. 4 and anymore relapses on a year
D. 5 and more relapses are on a year
E. 6 and more relapses are on a year

325. For the heavy flow of ulcerous illness characteristically


A. * development of complications
B. seasonal exacerbation more not frequent 1-2 times per a year
C. 1-2 relapse on a year
D. liquid, but protracted exacerbation
E. exacerbation duration more than 10 days
326. For motion of disease ulcerous illness of middle weight characteristically
A. development of complications
B. * relapses 1-2 times per a year
C. 4 and anymore relapses on a year
D. 5 and more relapses are on a year
E. 3 and anymore relapses on a year
327. To absolute indication to operative interference at ulcerous illness does not belong
A. * scarry-ulcerous stenosis
B. perforation of ulcer
C. profuse bleeding
D. diameter ulcer a more than 3 cm
E. bleeding what does not stopped with conservative
328. Diet at bleeding gastric and duodenum ulcers
A. * Meulengracht's
B. № 1 by Pevznerom
C. № 5 by Pevznerom
D. № 15 by Pevznerom
E. № 7 by Pevznerom
329. From what department degestyive tract developmentp more frequent than all the bleeding at the
Mallory-Weiss syndrome
A. gastric fundus
B. * cardial pert;
C. pyloric department;
D. from duodenal;
E. from a thick intestine
330. At ulcerous illness can a bleeding source be
A. artery;
B. veins;
C. shallow vessels and ulcers;
D. all answers are not correct.
E. * all answers are correct
331. Disappearance or diminishing the pain with beginning of bleeding from an ulcer is
A. * Bergman’s sing
B. Spazarskiy’s sing
C. Mendel’s suing
D. De Keven’s sing
E. Eleker’s sing
332. To the gastric – intestinal bleeding of unulcerous etiology belong
A. * Mallory-Weiss syndrome;
B. hemorragic erosive gastritis;

C. diseases by Randyu – Oslera – Vebera;


D. Menetrie's sing;
E. all answers are correct.
333. What from the transferred operations on the stomach organ protective is
A. * selective proximal vagotomy
B. resection by Bilrot I
C. resection by Bilrot II
D. gastrectomy
E. all are transferred
334. What from the transferred operations does not belong to organ protective
A. trunk vagotomy
B. * resection by Bilrot II
C. selective vagotomy
D. selective proximal vagotomy
E. the all transferred does not belong
335. What localization of ulcer is most characteristic for the patients of elderly and senile age
A. * cardial department of stomach
B. overhead third of gullet
C. lower third of gullet
D. bulb of duodewnum
E. small curvature
336. What colouring of chair is most characteristic for bleeding from the ulcer of stomach and
duodwnum?
A. * Tarry excrement
B. Presence on the formed excrement of strokes of red blood
C. Littlechanged blood in an excrement
D. Excrement of the raspberry colouring with the admixtures of mucus
E. Acholic excrement
337. What external signs are characteristic for the profuse bleeding from a gastric ulcer?
A. Vomiting by the littlechanged blood, excrement of the raspberry colouring
B. * Vomiting by the littlechanged blood, tarry darkly-cherry chair
C. Vomiting by a complete mouth by dark blood with clots, black formed excrement
D. Vomiting on the type of "coffee-grounds", presence on the formed incandescence of strokes of red
blood
E. Tarry darkly-cherry chair
338. What preparations, except for other properties, own yet and a bacteriostatic effect on Hеlісоbасtеr
руlоrіs
A. * all are transferred preparations
B. Almagel
C. Vinylin
D. De-nol
E. Claritromycin
339. What products are recommended in the diet of № 1?
A. * hen in a steam kind
B. pancakes
C. raw egg-white
D. bread rye fresh

E. acute cheeses
340. What violation of mineral exchange is characteristic for patients with ulcerous illness of stomach
and intestine
A. hypocalcemia
B. * hypokaliemia
C. Hyponatremia
D. Hypercalcinemia
E. hyperkaliemia
341. What complication the ulcerous illness of stomach is most characteristically for the patients of
elderly and senile age
A. perforation
B. * perforation bleeding
C. pylorus stenosis
D. * malignization penetration
E. * bleeding
342. What from preparations has the expressed bacteriostatic action on Hеlісоbасtеr руlоrіs
A. * trichopol
B. Licviroton
C. oxiferiskorbon
D. atropine
E. pirinzsipin
343. What method diagnostics hte ulcerous illness most informing
A. * esophagogastroduodenoscopy
B. analysis of excrement on the hidden blood
C. X-rye
D. global analysis of blood
E. research of gastric secretion
344. What preparation does behave to blocker H2-retseptors?
A. tavegil
B. obzidan
C. hystdol
D. cerucal
E. * oraza
345. What syndrome is characteristic for hemorragic erosive gastritis?
A. * ulcerous
B. hemorragic;
C. pain;
D. all answers are correct;
E. all answers are not correct.
346. When apply Teylor’s method at ulcerous illness
A. * at conservative treatment perforeted ulcers
B. at conservative treatment sanguifluous ulcers
C. at conservative treatment penetration ulcers
D. at conservative treatment malignization ulcers
E. at conservative treatment of cicatrical pyloristenosis
347. Blood loos I stage characterized such indexes
A. Ht 48-44, Hb 120

B. Ht 23 and below, Hb 50 and below


C. Ht 31-23, Hb 80
D. * Ht 38-32, Hb 100
E. Ht 44-40, Hb 110 /e
348. Blood loos II stage characterized
A. * Ht 23 and below,
B. Hb 50 and below
C. Ht 31-23, Hb 80
D. Ht 44-40, Hb 110
E. Ht 48-44, Hb 120
349. Blood loos III stage characterized a degree such indexes
A. Ht 23 and below, Hb 50 and below
B. * Ht 31-23, Hb 80
C. Ht 38-32, Hb 100
D. Ht 44-40, Hb 110
E. Ht 48-44, Hb 120
350. At perforated ulcer the pulled wooden belly is determined in the stage of peritonitis
A. terminal
B. toxic
C. * reactive
D. terminal-toxic
E. there is not a right answer
351. At percusion in the first clock after perforation the ulcer more possibly
A. expansion of percusion border of liver
B. * dulling in the gently sloping places of stomach
C. tympanitis in left subcosctal area
D. expansion of percusion border the spleen
E. there is not a right answer
352. For differentiation of acute appendicitis with the covered perforete ulcer useful:
A. gastroduodenoscopy, X-rye of abdominal region, USD of abdominal region
B. survey X-rye of abdominal region, USD of abdominal region, irrigoscopy
C. gastroduodenoscopy, X-rye of abdominal region, laparoscopy
D. X-rye of abdominal region, irrigoscopy
E. * X-rye of abdominal region, laparoscopy
353. Ulcerous diaeases behaves to the diseases
A. because of alcoholism
B. because of the broken circulation of blood
C. * hronic recurrent
D. innate
E. traumatic
354. Hepatic dullness is not determined at
A. * perforations of gastric ulcer
B. break of bud
C. break of spleen
D. volvulus of stomach
E. mesenterial ishemia

355. Penetration of gastric content in an abdominal region possibly at


A. stenosis
B. penetration
C. bleeding
D. malignixation
E. * perforations
356. Hectic fever is possible at
A. bleeding from an ulcer
B. perforations of ulcer in the first clock
C. * penetration
D. the uncomplicated ulcer
E. cicatricle-ulcerous stenosis
357. For clarification diagnosis of perforete ulcer used
A. X-rye of abdominal region
B. * X-rye of abdominal region, after gastroscopy, then X-rye of abdominal region
C. Gastroscopy
D. CT
E. There is not a right answer
358. At operations on a stomach sometimes by mistake bandage an additional hepatic artery, that, in
same queue, can result to necrosis the segment, sectors or even stakes of liver. An additional
hepatic artery more frequent walks away from an artery
A. splenic
B. general hepatic
C. overhead mesenteric
D. all answers are faithful
E. * left stomach
359. Most guarantee against the recedive of ulcer during an operation concerning ulcerous diseases of
duodenum gives
A. trunks vagotomy with a pyloroplasty
B. * resection no less than a 2/3 stomach
C. selective-proximal vagotomy
D. antrumectomy with selective vagotomy
E. veritable antrumectomy
360. Direct sign of ulcer at x-ray research
A. violation of evacuation from a stomach
B. change of tone of stomach
C. form the stomach as "sand-glasses"
D. * symptom of "niche"
E. defect of filling
361. Hectic fever is possible at
A. to the uncomplicated ulcer
B. cicatrical-ulcerous stenosis
C. * penetration
D. perforations of ulcer in the first clock
E. bleeding from an ulcer
362. Tactic of family doctor during the covered perforation of ulcer
A. * urgent hospitalization in surgical permanent establishment

B. planned hospitalization in surgical permanent establishment


C. supervision on to the house
D. hospitalization in therapeutic permanent establishment
E. there is not a right answer
363. Change in the analysis of blood at a perforete ulcer
A. leucopenia
B. anaemia
C. eosinophilia
D. * leucocytosis with a neutrophilic change
E. there is not a right answer
364. In treatment of ulcerous illness the stomach and duodenum executed only on urgent indications
A. stomach resection by Bilrot-II
B. * sewing up of the perforete opening
C. selective-proximal vagotomy
D. trunk vagotomy with a pyloroplasty
E. stomach resection by Bilrot-I
365. At the perforete ulcer of duodenum used more frequent
A. * sewing up gastroenteroanastomosis
B. resection of stomach
C. sewing up of the perforate opening
D. resection of stomach for a shutdown
E. different types of vagotomy in combination with the economy resection of stomach and other
draining operations
366. The ways distribution of gastroenteric content during the perforation of ulcer depend on
A. anatomic structure of the lateral ductings
B. locations of stomach
C. localizations of the perforate opening
D. * only transferred
E. forms and locations of transversal rim bowel
367. Sudden and painful pain with localization in the middle departments of stomach with an irradiation
in the back more characteristic for
A. heart attack the myocardium
B. * break aneurysm the aorta
C. bilious colic
D. perforate ulcers
E. nephrocolic
368. In the moment of perforation the gastric or duodenum ulcer meets most often
A. * suddenly arising up megalgia
B. cramp-like pain
C. noncommunicative, moderate pain
D. liquid chair
E. tachycardia
369. Numbers of complications the ulcerous illness
A. * 5
B. 4
C. 1
D. 3

E. 2
370. A stomach-ache develops suddenly at (complication of ulcerous illness)
A. bleeding
B. malignization
C. stenosis
D. penetration
E. * perforations
371. For a perforete gastric ulcer in the first six clock typical
A. great sudden pains in a stomach, frequent vomiting, swelling of stomach, disappearance the
hepatic dullness, "sickle" under the right dome of diaphragm
B. frequent vomiting, swelling of stomach, disappearance the hepatic dullness, "sickle" under the right
dome of diaphragm
C. great sudden pains in a stomach, wooden belly, swelling of stomach
D. * great sudden pains in a stomach, wooden belly, disappearance the hepatic dullness, "sickle" under
the right dome of diaphragm
E. combinations of signs are equivalent
372. Most informing method diagnostics the perforate ulcers
A. X-rye examination
B. USD
C. EGDS
D. * laparoscopy
E. laparocentezis
373. Tension the muscles of stomach in an initial period of perforation ulcer
A. absents
B. * visible
C. sickliness under the left shoulder-blade
D. appears at palpation
E. there is not a right answer
374. Rational operation at the subcompensated ulcerous stenosis of pylorus
A. antrumectomy
B. * resection 2/3 stomach
C. front gastroenteroanastomosis
D. gastrectomy
E. selective proximal vagotomy
375. At suspicion on a duodenal ulcer conduct above all things
A. research of gastric secretion
B. * EGDS
C. X-rye examination organs of abdominal region
D. determination the level of gastrin the whey blood
E. cholecystography
376. At a perforation gastric ulcer, vomiting blood is
A. often
B. very often
C. it is never
D. * rarely
E. there is not a right answer
377. What basic method the treatment of acute pancreatitis is:

A. Surgical
B. * Conservative
C. Homoeopathic
D. Physical therapy
E. A right answer is not present
378. Acute pancreatitis with a heavy flow treat in terms:
A. Ambulatory
B. Permanent establishment
C. * Department of intensive therapy
D. All answers are faithful
E. A faithful answer is not present
379. Conservative treatment the acute pancreatitis includes:
A. * Hunger
B. Povzner’s diet №5
C. Povzner’s diet №15
D. Moderation in a meal
E. Morning gymnastics
380. Early complications the acute pancreatitis is not:
A. * Fistula of pancreas
B. Pancreatic shock
C. Collapse
D. Ferment peritonitis
E. Pancreatic delirious syndrome
381. Name principal reason the acute pancreatitis:
A. Trauma the pancreas
B. * Bile-stone diseases
C. Alimentary factor
D. Chronic alcoholic pancreatitis
E. Cardiospasm
382. What operation on a pancreas is indicated at the oedematous form the acute pancreatitis:
A. * Interference on a pancreas is not needed
B. Pancreatectomy
C. Omentopankreatopeksy
D. Abdominisation pancreas
E. Marsupialization
383. At a frequent „fat” chair with disseminations of undigested meat and permanent thirst, it is
foremost necessary to think about:
A. * Chronic pancreatitis
B. Chronic duodenitis
C. Chronic hepatocholecystitis
D. Ulcerous diseases of duodenum
E. Hepatocirrhosis
384. Name most frequent complication after ERDPH:
A. * Pancreatitis
B. Cholangitis
C. Pancreatic sepsis

D. Reactive cholecystitis
E. Obstructing papillitis
385. How often does the pancreatitis department the general bilious channel pass through the head of
pancreas?:
A. * 80-90 %
B. 90-100 %
C. 75-85 %
D. 50-60 %
E. 40-50 %
386. What primary purpose treatment the patients with the heavy form of hemorragic pancreatonecrosis
to the operation is:
A. Liquidations the pain
B. * Disintoxication the organism
C. Liquidations crampy the big duodenal papilla
D. Declines secretory activity the pancreas
E. Improvements microcirculation
387. What primary purpose treatment the patient with fatty pancreatonecrosisto the operation is:
A. Liquidations the pain
B. * Disintoxication the organism
C. Liquidations crampy the big duodenal papilla
D. Declines secretory activity the pancreas
E. Improvements microcirculation
388. What most optimum resort is which used for pathology the pancreas:
A. Pyatigorsk
B. * Morshin
C. Kuyal'nik
D. Truskavets
E. Nemirov
389. Name the optimum volume of operation at acute biliary pancreatitis:
A. Deleting exsudate from an abdominal region
B. Decapsulation pancreas
C. Pancreatectomy
D. * Sanitization and draining the bilious ways
E. Draining the chanel of pancreas
390. Name the optimum lines the operative interference concerning a acute pancreatitis after the
beginning of disease:
A. 7-8 days
B. 1-3 days
C. 1-5 days
D. 3-5 days
E. * Surgical treatment is not indicate
391. Among acute surgical diseases acute pancreatitis occupies:
A. First place
B. * Third place
C. Fifth place
D. Second - third place
E. Most widespread

392. What is condition hematomesis at acute pancreatitis:


A. Presence concomitant gastric ulcer
B. Presence concomitant gastritis
C. * Formation erosions in a stomach
D. Violation of microcirculation
E. Enzymes in blood
393. ERCP apply at:
A. Postcholecystectomy syndrome
B. Stenosing papillitis
C. * Stenosis the supraduodenal department of choledoch
D. Stricture the terminal department of choledoch
E. Mechanical icterus
394. What preparation is attributed to the proteases inhibitors:
A. Garamycin
B. Gaviskon
C. * Gordox
D. Halidor
E. Gramicidin
395. What preparation is attributed to the proteases inhibitors:
A. Tocopherol
B. Triampur
C. Tagamet
D. * Trasylol
E. Trypsin
396. What preparation is not attributed to the inhibitors proteases:
A. Pantripin
B. Kontrikal
C. Gordox
D. Trasylol
E. * Trypsin
397. What preparation is applied at violation the extrasecretory function of pancreas at a chronic
pancreatitis:
A. Pyracetam
B. Papaverin
C. Pantocrin
D. * Panzinorm
E. Panthenol
398. What operation is used for suppuration the pseudocysts of pancreas:
A. Cystojejunostomy on the eliminated loop
B. * External draining the cyst
C. Cystogastrostomy
D. Cystoduodenostomy
E. Cystoenteroanastomosis
399. What operation is used for the pseudocyst of pancreas in the 3th stage of its forming:
A. * Cystojejunostomy on the eliminated loop
B. External draining the cyst

C. Cystogastrostomy
D. Cystoduodenostomy
E. Cystoenteroanastomosis
400. What operation is most often used for localization the formed pseudocyst in the area of tail the
pancreas:
A. Cystojejunostomy on the eliminated loop
B. External draining the cyst
C. Cystogastrostomy
D. * Cystoduodenostomy
E. Cystoenteroanastomosis
401. What preparations from the cytostatic group use for intensifying the chronic pancreatitis:
A. Cyanocobalamin
B. Methyluracil
C. * 5-fluorouracil
D. Furadolizon
E. Mezimforte
402. What most effective blocker secretion of pancreas at acute pancreatitis:
A. Cyanocobalamin
B. Ubretid
C. Arginine
D. * Sandostatin
E. Benzogeksoniy
403. Indication to early operative interference at acute pancreatitis is:
A. Acute oedematous pancreatitis
B. Acute pancreatolysis
C. * Acute biliary pancreatitis
D. Acute fatty pancreatitis
E. Forming of pseudocyst
404. Indication to early operative interference at acute pancreatitis is:
A. Acute pancreatolysis
B. Acute oedematous pancreatitis
C. * Progressive multiple organ failure what not added conservative therapy during 48-72 hours
D. Acute fatty pancreatitis
E. Forming of pseudocyst
405. What from operations does not execute at surgical treatment complicated acute pancreatitis:
A. Through draining the stuffing-box bag
B. Abdominisation the pancreas
C. Omentopankreatopeksiy
D. Left-side resection of gland
E. * Pancreatojejunostomy
406. At pancreatitis abscesses and infected necrosises execute such operations, except for:
A. Opening of abscess with draining
B. Pancreaticnecrsekvestrektomy
C. Pancreaticsekvestrektomy
D. Pancreaticsekvestrektomy with laparostomy
E. * Total pancreatotomy

407. What most effective treatment the unformed uncomplicated cyst is:
A. * Conservative treatment
B. External draining cyst
C. Resection cyst within the limits of the unchanged gland
D. Cysticenterostomy
E. Cystogastrostomy
408. What most effective treatment the unformed complicated cyst is:
A. Conservative treatment
B. * External draining cyst
C. Resection cyst within the limits of the unchanged gland
D. Cysticenterostomy
E. Cystogastrostomy
409. What most effective treatment the formed uncomplicated cyst is:
A. External draining the cyst
B. Marsupialization
C. Resection the cyst within the limits of the unchanged gland
D. * Cysticenterostomy
E. Cystogastrostomy
410. What nosotropic conditionality Voscresencky’s sing at acute pancreatitis:
A. Reflex paresis of colon
B. Thrombosis of abdominal aorta
C. * Inflammatory edema of pancreas
D. Embolism of abdominal aorta
E. Development of peritonitis
411. All surgical interferences at the destructive forms of acute pancreatitis divide on:
A. * Early, late, deferred operations
B. Primary, second, repeated operations
C. Invasion, not invasion operations
D. Complicated, operations are not complicated
E. Not divided
412. What is sequestrotomy:
A. * Delete the necrotic area within the limits of nonviable fabrics
B. Delete the necrotic area within the limits of healthy fabrics
C. Delete part of organ with his transversal cutting within the limits of the changed fabrics
D. Total delete of organ
E. There is not a faithful answer
413. What is necrectomy:
A. Delete the necrotic area within the limits of nonviable fabrics
B. * Delete the necrotic area within the limits of healthy fabrics
C. Delete part of organ with his transversal cutting within the limits of the changed fabrics
D. Total delete of organ
E. There is not a faithful answer
414. What is resection the pancreas:
A. Delete the necrotic area within the limits of nonviable fabrics
B. Delete the necrotic area within the limits of healthy fabrics
C. * Delete part of organ with his transversal cutting within the limits of the changed fabrics

D. Total delete of organ


E. There is not a faithful answer
415. What is pancreatectomy:
A. Delete the necrotic area within the limits of nonviable fabrics
B. Delete the necrotic area within the limits of healthy fabrics
C. Delete part of organ with his transversal cutting within the limits of the changed fabrics
D. * Total delete of organ
E. There is not a faithful answer
416. What is pancreas located in relation to a peritoneum:
A. * Retroperitoneal
B. Mesoperitoneal
C. Intraperitoneal
D. All answers are incorrect
E. Variously
417. What is blood supply the body and tail pancreas:
A. * Splenic artery
B. A.gastroduodenalis
C. A.gastrica sinistra
D. A.cystica
E. Variously
418. Mondor’s sing at acute pancreatitis:
A. * Violet spots on face and trunk
B. Cyanosis sides of stomach and trunk
C. Cyanosis skin of stomach
D. Icteritiousness round a belly-button
E. Cyanosys of hands
419. Turner’s sing at acute pancreatitis:
A. * Violet spots on face and trunk
B. Cyanosis sides of stomach and trunk
C. Cyanosis skin of stomach
D. Icteritiousness round a belly-button
E. Cyanosys of hands
420. Holsted’s sing at acute pancreatitis:
A. Violet spots on face and trunk
B. Cyanosis sides of stomach and trunk
C. * Cyanosis skin of stomach
D. Icteritiousness round a belly-button
E. Cyanosys of hands
421. Kulen’s sing at acute pancreatitis:
A. Violet spots on face and trunk
B. Cyanosis sides of stomach and trunk
C. Cyanosis skin of stomach
D. * Icteritiousness round a belly-button
E. Cyanosys of hands
422. Voskresenskiy’s sing at acute pancreatitis:
A. * Absence pulsation the abdominal aorta

B. Sickliness in left costal-vertebral coal


C. Sickliness and proof tension the muscles in an epigastrium with passing to left subcostal area
D. Icteritiousness round a belly-button
E. Skin hyperesthesia in projection the gland
423. Kerte’s sing at acute pancreatitis:
A. Absence pulsation the abdominal aorta
B. Sickliness in left costal-vertebral coal
C. * Sickliness and proof tension the muscles in an epigastrium with passing to left subcostal area
D. Icteritiousness round a belly-button
E. Skin hyperesthesia in projection the gland
424. Meyo-Robson’s sing at acute pancreatitis:
A. Absence pulsation the abdominal aorta
B. * Sickliness in left costal-vertebral coal
C. Sickliness and proof tension the muscles in an epigastrium with passing to left subcostal area
D. Icteritiousness round a belly-button
E. Skin hyperesthesia in projection the gland
425. How many is the period of haemodynamic violations and pancreatic shock lasts:
A. * 1-3 days
B. 3-7 days
C. More 7 days
D. 2 weeks
E. 2 hour
426. How many is the period of functional insufficiency of parenchymatous organs lasts:
A. 1-3 days
B. * 3-7 days
C. More 7 days
D. 2 weeks
E. 2 hour
427. How many is the period of degenerative and festerings complications lasts:
A. 1-3 days
B. 3-7 days
C. * More 7 days
D. 2 weeks
E. 2 hour
428. What hormone of pancreas has influences on metabolism glucose:
A. * Insulin
B. Vasopressin
C. Adrenalin
D. Somatotropin
E. Tiroksin
429. What cages of pancreas are make insulin:
A. ? клетки
B. * ? клетки
C. ? -клетки
D. None of cages
E. D-клетки

430. What hormone of pancreas has influences on the exchange of fats:


A. Insulin
B. Glyukagon
C. * Lipocainu
D. Adrenalin
E. Somatotropin
431. At what disease pain of girdle character is characteristic:
A. Gastric ulcers
B. Acute cholecystitis
C. Intestinal impassability
D. * Acute pancreatitis
E. Acute cystitis
432. The leading clinical symptoms the acute pancreatitis is:
A. * Stomach-ache
B. Vomiting by „coffee-grounds”
C. Disuria
D. Febrile temperature of body
E. Lock
433. pathognomonic symptom at acute pancreatitis is:
A. Pasternatskiy’s sing
B. * Kulen’s sing
C. Ker’s sing
D. Lenander’s sing
E. Rovsing’s sing
434. Characteristic complication the acute pancreatitis is:
A. Paranephritis
B. Duglas’s abscess
C. Pylephlebitis
D. * Pancreonecrosis
E. Cyst of pancreas
435. The laboratory signs of total pancreonecrosis is:
A. Growth glucose concentration
B. Diminishing maintenance fibrinogenum
C. * Growth activity diastase
D. Diminishing activity diastase
E. Growth index AST
436. For diagnostics of acute pancreatitis most informing is:
A. * USD
B. CT
C. Cholangiography
D. Esophagogastroduodenoscopy
E. Colonoscopy
437. What norm diastase is:
A. * To 160
B. To 50
C. To 200

D. To 300
E. To 1200
438. What level diastase answers a chronic pancreatitis:
A. * 200-500
B. 100-160
C. 600-1000
D. 1000-1500
E. 10-20
439. What level diastase answers a acute pancreatitis:
A. 100-160
B. 200-500
C. * 600-1000
D. 1000-1500
E. 10-20
440. What level diastase answers pancreonecrosis:
A. 600-1000
B. 200-500
C. 100-160
D. * 1000-1500
E. 10-20
441. For what pathology appearance of violet spots on the skin of person and trunk is characteristic:
A. Perforated ulcer
B. Acute cholecystitis
C. * Acute pancreatitis
D. Intestinal obstruction
E. Acute appendicitis
442. At what pathology is absence pulsation of abdominal aorta
A. * Acute cholecystitis
B. Acute pancreatitis
C. Perforated ulcer
D. Intestinal obstruction
E. Acute appendicitis
443. At what pathology is appearance cyanosys the sides of stomach:
A. * Acute pancreatitis
B. Perforated ulcer
C. Acute cholecystitis
D. Intestinal obstruction
E. Acute appendicitis
444. At what pathology is appearance of sickliness in left costal-vertebral coal:
A. * Acute pancreatitis
B. Acute cholecystitis
C. Perforated ulcer
D. Intestinal obstruction
E. A right answer is not present
445. As the first period flowing of acute pancreatitis is named:
A. * Haemodynamic violations and pancretic shock

B. To functional insufficiency of parenchymatous organs


C. Degenerative and festerings complications
D. All answers are faithful
E. A right answer is not present
446. As the second period flowing of acute pancreatitis is named:
A. * Haemodynamic violations and pancretic shock
B. To functional insufficiency of parenchymatous organs
C. Degenerative and festerings complications
D. All answers are faithful
E. A right answer is not present
447. As the third period flowing of acute pancreatitis is named:
A. Haemodynamic violations and pancretic shock
B. * To functional insufficiency of parenchymatous organs
C. Degenerative and festerings complications
D. All answers are faithful
E. A right answer is not present
448. What complications at a acute pancreatitis is behave to early:
A. * Peritonitis
B. Phlegmon retroperitoneum space
C. Formation of pseudocysts
D. Development of saccharine diabetes
E. Intestinal impassability
449. What complications at a acute pancreatitis is behave to late:
A. Peritonitis
B. * Phlegmon retroperitoneum space
C. Formation of pseudocysts
D. Development of saccharine diabetes
E. Intestinal impassability
450. What a clinical flow can be at acute pancreatitis:
A. Easy, middle, heavy
B. Acute, chronic
C. * Abortive, making progress
D. Edema, necrosis
E. Any variant
451. For the abortive flowing characteristically:
A. * A process limited to the acute edema with convalescence in 7-8 days
B. A process limited to tearing away of the pathologically changed gland
C. A process limited to tearing away of the pathologically unchanged gland
D. A disease completed so not attaining clinical displays
E. Changes from the side of organ are not present
452. Specify the optimum volume operation at acute biliar pancreatitis:
A. Deleting exsudate from abdominal region
B. Decapsulation pancreas
C. Pancreatectomy
D. * Sanitization and draining bilious ways
E. Draining pancreas channel

453. What preparation is applied at violation the extrasecretory function pancreas at a chronic
pancreatitis:
A. Pyracetam
B. Papaverin
C. Pantocrin
D. * Mezim-forte
E. Panthenol
454. In obedience to classification complications of acute pancreatitis, after etiology and pathogeny,
distinguish such complications, except for:
A. Infectiously inflammatory
B. Enzymic
C. * Allergic
D. Mixed
E. Trombogemoragic
455. Characteristic complication acute pancreatitis is:
A. * Pancreonecrosis
B. Pylephlebitis
C. Hepatocirrhosis
D. Veritable pancreas cyst
E. Hepatitis
456. After time of origin complications acute pancreatitis select:
A. Premature and remote
B. Urgent and deferred
C. Primary and second
D. * Early and late
E. Any of variants faithful
457. To early complications acute pancreatitis attribute:
A. Shock
B. Acute hepatic-kidney insufficiency
C. Poured out peritonitis
D. Icterus
E. * All answers are faithful
458. In the postoperative period the patient must be treated with volume infusion likely for:
A. 50 ml / kg;
B. 10 ml / kg;
C. 20 ml / kg;
D. * 40 ml / kg
E. 30 ml / kg
459. Vital body functions:
A. hemodynamic state of consciousness;
B. respiration, activity of kidney, liver, gastrointestinal tract;
C. hemodynamics, somatic and autonomic nervous system;
D. breathing, heart activity, liver;
E. * respiratory, hemodynamic, central nervous system activity
460. Please provide optimal kalorazh during postoperative parenteral nutrition patients:
A. * 40 kcal / kg
B. 10 kcal / kg;

C. 20 kcal / kg;
D. 30 kcal / kg;
E. 50 kcal / kg
461. Parenteral nutrition patients displayed:
A. * if you can not enteral
B. All patients who are treated in VAITi;
C. all terminally ill;
D. with disorders of vital functions;
E. in the postoperative period
462. In preparations for parenteral nutrition include:
A. polihlyukin;
B. albumin;
C. plasma;
D. starch derivatives;
E. * lipofundin
463. In preparations for parenteral nutrition include:
A. * 10% glucose solution
B. albumin;
C. plasma;
D. starch derivatives;
E. karbikarb
Назва наукового напрямку (модуля): Семестр: 12
KROK 2 SURGERY 6 year
Опис:

Перелік питань:
1. ?During medical examination a cadet in the naval college was detected to have a painless dense
ulcer 1.5x0.5 in size in his perianal area at the 2 o’clock position. The ulcer ?oor resembles ”old
fat”. What is the provisional diagnosis?
A. * Hard syphilitic chancre of the rectum
B. Rectal ?ssure
C. Rectal ?stula
D. Anal cancer
E. Anal crypt suppuration
2. A 10-year-old boy, who was outdoors in windy and cold weather, developed moderate pain and
tingling in his ?ngers and toes. When he had returned home, his parents noticed that the tips of
his ?ngers and toes were white and their sensitivity was lost. The affected areas are warming up,
the ?ngers are tingling and in pain. Skin pallor changed into redness, tingling stopped, slight itching
and swelling of the ?ngers appeared. Determine the frostbite degree in this child:
A. * Frostbite of the I degree
B. Perniosis
C. Frostbite of the II degree
D. Frostbite of the III degree
E. Frostbite of the IV degree
3. 4 days after a patient received a gunshot wound to the soft tissues of middle third of the thigh, his
condition suddenly began deteriorating. There are complaints of bursting pain in the wound; pain
has been increasing during the last 12 hours. Oedema of skin and hypodermic tissue quickly grows.
Body temperature is 38,2oC, heart rate is 102/min. The wound edges gape, are dull in colour; the
muscles, viable as of day before, now protrude into the wound, look boiled, are dull in colour, have
dirty-grey coating, and fall apart when held with forceps. What infection has developed in the
A. * wound?
Anaerobic
B. Aerobic gram-negative
C. Putrid
D. Aerobic gram-positive
E. Diphtheria of the wound
4. A patient received ?ame burns of both hands. On the dorsal and palmar surface of the hands there
are blisters ?lled with serous ?uid. The wrist joint region is hyperaemic. The forearms were not
injured. What is the provisional diagnosis?
A. * II degree ?ame burn of the hands, 4% of body surface area
B. II degree ?ame burn of the hands, 2% of body surface area
C. IIIa degree ?ame burn of the hands, 4% of body surface area
D. III degree ?ame burn of the hands, 4% of body surface area
E. IIb degree ?ame burn of the hands, 2% of body surface area
5. On the 4th day after recovery from a cold a patient was hospitalized with complaints of solitary
spittings of mucoid sputum. On the 2nd day there was a single discharge of about 250 ml of
purulent blood-streaked sputum. Objectively: the patient’s condition is moderately severe.
Respiratory rate - 2830/min., Ps- 96/min., BP- 110/70 mm Hg. Respiration over the left lung is
vesicular, over the right lung - weakened. There are various moist crackles over the lower lobe and
amphoric breath sounds near the angle of scapula. What is the most likely diagnosis?
A. * Acute pulmonary abscess
B. Exudative pleuritis
C. Acute focal pneumonia
D. Pleuralempyema
E. Pyopneumothorax
6. A 65-year-old woman on abdominal palpation presents with a tumour in the umbilical region and
above it; the tumour is 13x8 cm in size, moderately painful, nonmobile, pulsing. On auscultation
systolic murmur can be observed. What is the most likely diagnosis?
A. * Abdominal aneurysm
B. Gastric tumour
C. Arteriovenous aneurysm
D. Tricuspid insuf?ciency
E. Bicuspid insuf?ciency
7. After a case of purulent otitis a 1-year-old boy has developed pain in the upper third of the left
thigh, body temperature up to 39oC. Objectively: swelling of the thigh in its upper third and
smoothed-out inguinal fold. The limb is in semi-?exed position. Active and passive movements are
impossible due to severe pain. What diagnosis is the most likely?
A. * Acute haematogenous osteomyelitis
B. Acute coxitis
C. Intermuscular phlegmon
D. Osteosarcoma
E. Brodie’s abscess
8. A 74-year-old patient was delivered into admission room with clinical presentations of acute deep
vein thrombosis of the shin. What symptom is the most typical of this pathology?
A. * Homans’ sign
B. Rovsing’s sign
C. Courvoisier’s sign
D. Mayo-Robson’s sign
E. Grey Turner’s sign
9. A 50-year-old patient was delivered to a hospital with complaints of blood traces in urine. Urination
is painless and undisturbed. Macrohematuria had been observed for 3 days. Objectively: kidneys
cannot be palpated, suprapubic area is without alterations, external genitalia are non-pathologic. On
rectal investigation: prostate is not enlarged, painless, has normal structure. Cystoscopy revealed no
alterations. What is the most likely diagnosis?
A. * Renal carcinoma
B. Bladder tuberculosis
C. Varicocele
D. Dystopic kidney
E. Necrotic papillitis
10. A man complains of constant dull pain in the perineum and suprapubic area, weak ?ow of urine,
frequent dif?cult painful urination, nocturia. The patient has been suffering from this condition for
several months, during which urination was becoming increasingly dif?cult, and pain in the
perineum has developed. On rectal examination: the prostate is enlarged (mainly its right lobe),
dense, asymmetrical, central ?ssure is smoothed out, the right lobe is of stony density, painless,
tuberous. What disease is it?
A. * Prostate cancer
B. Prostate sclerosis
C. Urolithiasis, prostatolith of the right lobe
D. Prostate tuberculosis
E. Chronic congestion prostatitis

11. A 17-year-old young man complains of general weakness, trismus, twitching of the muscles in his
left shin. 7 days ago he pierced his foot with a nail. Objectively: at the sole of the foot there is a
wound, 0,3х0,2 mm in size, with small amount of serous-purulent discharge, the skin around
the wound is hyperaemic. What is the most likely diagnosis?
A. * Tetanus
B. Phlegmon
C. Osteomyelitis
D. Infected wound
E. Erysipelas
12. A patient with trauma of the lower third of the forearm volar surface caused by a glass shard came
to a ?rst-aid centre. Objectively: ?exion of the IV and V ?ngers is impaired, sensitivity of the inner
dorsal and palmar surfaces of the hand and IV ?nger is decreased. What nerve is damaged?
A. * Ulnar
B. Radial
C. Median
D. Musculocutaneous
E. Axillary
13. A man diagnosed with closed-angle glaucoma, grade IIa, of the right eye is registered for regular
medical check-ups. In the evening an acute glaucoma attack occurred in his right eye; an
ambulance was called. What emergency aid would be optimal in this case?
A. * Pilocarpine, Diacarb (Acetazolamide), lytic mixture
B. Atropine eye drops
C. Antibiotic eye drops, broad-spectrum
D. Sulfacetamide sodium eye drops
E. Dexamethasone eye drops
14. A 58-year-old patient complains of pain in the lower left extremity, which aggravates during
walking, and sensation of cold and numbness in the both feet. The patient has been suffering from
this condition for 6 years. Objectively: the skin is pale and dry, with hyperkeratosis. On the left shin
hair is scarce. Pulse cannot be detected over the pedal and popliteal arteries and is weakened over
the femoral artery. On the right limb pulsation of the popliteal artery is retained. What is the most
likely diagnosis?
A. * Atherosclerosis obliterans of the lower extremities
B. Obliterating endarteritis
C. Femoral artery thrombosis
D. Raynaud’s disease
E. Buerger’s disease (thromboangiitis obliterans)
15. A 47-year-old woman came to the admission room with complaints of general weakness, dizziness,
vomiting with blood clots. Condition onset was 3 hours ago. The patient has no preceding illnesses.
Blood pressure is 90/60 mm Hg, pulse is 106/min., of poor volume. The abdomen is soft, with mild
tenderness in the epigastrium. Blood test: erythrocytes - 2.1 • 1012/L, Нb- 70 g/L, hematocrit -
28%. What tactics should the doctor on duty choose?
A. * Consult the surgeon
B. Refer the patient to the family doctor
C. Give spasmolytics
D. Perform gastric lavage
E. Make an appointment for colonoscopy
16. A 52-year-old patient complains of pain in the right part of her chest, dyspnoea, cough with large
amounts of foamy sputum emitting foul smell and resembling ”meat slops”. Objectively: the
patient’s condition is grave, cyanosis is observed, breathing rate is 31/min., percussion sound above
the right lung is shortened, auscultation revealed various moist rales (crackles). What is the most
likely diagnosis?
A. * Lung gangrene
B. Lung abscess
C. Empyema of pleura
D. Multiple bronchiectasis
E. Chronic pneumonia
17. A man complains of sore throat on the left, pain in his left ear, to up to 39oC, and nasal sound of his
voice. Disease onset was 5 days ago. Marked trismus and increased salivation are observed. The
head tilts to the left shoulder. Left side of the soft palate presents with swelling, hyperaemia, and
in?ltration. Retromandibular lymph nodes on the left are acutely painful on palpation. Otoscopy
results are normal. Make the diagnosis:
A. * Left-sided peritonsillar abscess
B. Retropharyngeal abscess
C. Parapharyngeal phlegmon
D. Peritonsillitis on the left
E. Cervical phlegmon on the left
18. A patient has the second and third degree burns of the 15% of the body surface. On the 20th day
after the trauma the patient presents with sharp increase of body temperature, general weakness,
rapid vesicular respiration; facial features are sharpened, BP is 90/50 mm Hg, heart rate is 112/min.
What complication is it?
A. * Sepsis
B. Pneumonia
C. Acute intoxication
D. Purulent bronchitis
E. Anaerobic infection
19. A patient in the state of clinical death is being resuscitated through mouth-to-mouth arti?cial
pulmonary ventilation and external cardiac massage. A doctor noticed that air does not ?ow into the
patient’s airways and his head and torso are positioned at the same level. Why is arti?cial
respiration ineffective in the given case?
A. * Tongue retraction
B. Low breathing volume
C. External cardiac massage
D. Probe is absent from the stomach
E. The patient’s mouth is too small
20. A patient complains of suppuration from the ear and impaired hearing of the left ear, which have
been observed for the past 6 years. The patient had periodical headaches, general indisposition,
fever. Objectively: otoscopy of the external auditory meatus revealed mucopurulent odorless
substance. The eardrum is of normal colour, with central perforation. What is the most likely
A. * diagnosis?
Chronic mesotympanitis
B. Otosclerosis
C. Acute otitis media
D. Chronic epitympanitis
E. Chronic sensorineural hearing loss
21. In 2 hours after a traf?c accident a 28-yearold man in grave condition was delivered to a hospital.
The patient complains of abdominal pain. He received a blow to the abdomen with the steering
wheel. Objective examination revealed the following: the abdomen does not participate in
respiration, is tense and acutely painful on palpation; abdominal guarding is present, peritoneal
irritation signs are positive, hepatic dullness is absent. BP is 90/60 mm Hg, heart rate is 120/min.
What further treatment tactics should be chosen?
A. * Laparotomy
B. Laparoscopy
C. Cold to the abdomen
D. Abdominal X-ray
E. Laparocentesis
22. A 48-year-old woman was arrived to the surgical unit with wounds in her thigh. On examination the
wound surface has dirty-grey coating with unpleasant sweet smell. Wound content resembles
raspberry jelly. Skin tissues around the wound are glossy and turgid. Palpation reveals moderate
crepitation in the tissues. What micro?ora is the most likely to cause such in?ammation?
A. * Anaerobic clostridial
B. Anaerobic non-clostridial
C. Streptococci
D. Staphylococci
E. Blue pus bacillus
23. A 30-year-old patient was hospitalized in an intensive care unit with a diagnosis of multiple bee
stings. Skin is pale and covered with cold sweat. Pulse can be palpated only at the carotid arteries
and is 110/min.; breathing rate is 24/min., rhythmical, weakened. What drug must be administered
immediately?
A. * Epinephrine hydrochloride
B. Prednisolone
C. Norepinephrine hydrochloride
D. Dopamine
E. Tavegyl (Clemastine)
24. A 46-year-old woman has been hospitalized with open fracture of the left thigh in its middle third.
She underwent the surgery-?xation with extraosseous osteosynthesis plates. On the 4th day after the
surgery she developed pain in the wound, body temperature rose over 39oC. What measures should
be taken in this case?
A. * Undo the sutures, drain the wound, and prescribe antibiotics
B. Prescribe broad spectrum antibiotics and hormonal agents
C. Administer antibiotics intraosseously and hypothermia locally
D. Inject antibiotics into the area surrounding the wound, prescribe spasmolytics and analgesics
E. Remove the ?xation, prescribe sulfanilamides
25. A 42-year-old man was delivered to a surgical in-patient department with complaints of icteric skin,
pain in the right subcostal area. Biochemical blood analysis: total bilirubin 140 mcmol/l, direct
bilirubin - 112 mcmol/l. On US: choledoch duct - 1,4 cm, a concrement is detected in the distal area.
Gallbladder is 40 cm, no concrements. What treatment tactics should be chosen?
A. * Endoscopic papillo sphincterotomy
B. Laparoscopic cholecystectomy
C. Laparotomy with choledoch duct drain
D. Laparotomy with cholecystectomy
E. Treatment in an infectious diseases hospital
26. 4 weeks after myocardial infarction a 56year-old patient developed acute heart pain, marked
dyspnoea. Objectively: the patient’s condition is extremely grave, there is marked cyanosis of face,
swelling and throbbing of neck veins, peripheral pulse is absent, the
carotidarterypulseisrhythmic,130/min., BPis 60/20mmHg.Auscultationoftheheartreveals extremely
muf?ed sounds, percussion reveals heart border extension in both directions. What is the optimal
treatment tactics for this patient?
A. * Pericardiocentesis and immediate thoracotomy
B. Oxygen inhalation
C. Puncture of the pleural cavity on the left
D. Conservative treatment, infusion of adrenomimetics
E. Pleural cavity drainage
27. A 43-year-old woman complains of persistent stomach ache with recurrent pain attacks, nausea,
repeated vomiting with stagnant bowel content, abdominal distension, and ?atulence. She has been
presenting with these signs for 7 hours. Pulse is 116/min. The tongue is dry and brown. The
abdomen is symmetrically distended, soft, painful. Percussion reveals tympanitis. On auscultation
there are bowel sounds with metallic overtone, splashing, and dripping. Make the diagnosis:
A. * Acute intestinal obstruction
B. Acute necrotizing pancreatitis
C. Acute destructive cholecystitis
D. Acute erosive gastritis
E. Acute nonspeci?c colitis
28. An infant cries during urination, the foreskin swells and urine is excreted in drops. What approach
to treatment should be chosen?
A. * Create an opening into the foreskin cavity
B. Prescription of ?-adrenergic blocking agents
C. Prescription of antispasmodic agents
D. Urinary bladder catheterization
E. Epicystostomy
29. A 20-year-old patient complains of pain in the left lumbar region, arterial pressure rises up to
160/110 mm Hg. US revealed that the structure and size of the right kidney were within age norms,
there were signs of the 3rd degree hydronephrotic transformation of the left kidney. Doppler
examination revealed an additional artery running to the lower pole of the kidney. Excretory
urogram shows a narrowing in the region of ureteropelvic junction. Specify the treatment tactics:
A. * Surgical intervention
B. Administration of spasmolytics
C. Administration of ACE inhibitors
D. Kidney catheterization
E. Administration of ?-blockers
30. A 49-year-old patient consulted a doctor about dif?cult swallowing, voice hoarseness, weight loss.
These symptoms have been gradually progressing for the last 3 months. Objectively: the patient is
exhausted, there are enlarged supraclavicular lymph nodes. Oesophagoscopy revealed no
oesophageal pathology. Which of the following investigations is the most appropriate in this case?
A. * Computed tomography of chest and mediastinum
B. X-ray of lungs
C. Multiplanar imaging of oesophagus
D. Radioisotope investigation of chest
E. Ultrasound investigation of mediastinum
31. A 38-year-old patient has been delivered by an ambulance to a surgical department with complaints
of general weakness, indisposition, black stool. On examination the patient is pale, there are dotted
haemorrhages on the skin of his torso and extremities. On digital investigation there are black
faeces on the glove. Blood test: Hb- 108 g/L, thrombocytopenia. Anamnesis states that similar
condition was observed 1 year ago. Make the diagnosis:
A. * Thrombocytopenic purpura
B. Haemophilia
C. Bleeding from an ulcer
D. Rectal tumour
E. Nonspeci?c ulcerative colitis
32. A 3-year-old child presents with sharp deterioration of his general condition. He has a history of
purulent otitis. His temperature is now 38.5oC. The left leg is pressed to the torso, active
movements are absent, the lower third of the thigh and knee joint are thickened, hyperaemic, with
localized fever. Axial load leads to acute discomfort of the patient. What is the most likely
A. * diagnosis?
Epiphyseal osteomyelitis on the left
B. Left hip fracture
C. Rheumatoid arthritis
D. Osteogenic sarcoma
E. Hygroma of the knee
33. A patient after a blow to the head developed general symptoms of cerebral disturbance, nausea,
vomiting, focal signs hemi-hyperre?exia S>D, hemihyperesthesia on the left, marked meningeal
syndrome. Neither cranial X-ray nor computer tomography revealed any pathologies. What
examination method would allow making and clari?cation of the diagnosis?
A. * Lumbar puncture
B. Echoencephalography
C. Electroencephalography
D. Angiography
E. Pneumoencephalography
34. A burn victim with ?ame burns of the IIIA-B and IV degrees on his face, neck, and anterior surface
of the thorax was brought into the admission room. The hairs in his nostrils are burnt, his labial and
glossal mucosa are gray-white. The voice is hoarse; respirations are frequent and shallow; the
patient has trumpet-like cough that produces soot-streaked sputum. The signs of respiratory failure
were progressing, while the patient was being transported into the intensive care unit. What
emergency care must be provided to this patient?
A. * Intubation of the trachea and mechanical ventilation
B. Tracheostomy
C. Administration of bronchial spasmolytics
D. Administration of respiratory analeptics
E. Inhalation of moisturized oxygen
35. The body of a 24-year-old woman with probable signs of poisoning has been found on the street.
Forensic medical examination was requested by an investigator during examination of the site and
the body. According to the Criminal Procedure Code currently in force in Ukraine, forensic medical
examination is required when it is necessary to determine the:
A. * Cause of death
B. Manner of death
C. Time of death
D. Mode of death
E. Mechanism of death
36. A 27-year-old man was hospitalized in severe condition 50 minutes after receiving a penetrating
wound to the left side of the chest. Objectively the patient is in a stupor, his skin is pale and
acrocyanotic. Pulse is 120/min., of poor volume, weak. Blood pressure is 80/40 mm Hg. Heart
sounds are muf?ed, cardiac borders are markedly expanded. In the III intercostal area along the
parasternal line on the left there is a stab-incised wound. Plain chest X-ray shows enlarged heart
shadow with smoothed out waist of the heart, there is haemothorax on the left to the 5th rib. What
contributes the most to the severity of the patient’s condition?
A. * Cardiac tamponade
B. Acute heart failure
C. Cardiac rhythm disturbance
D. Blood loss
E. Haemothorax and acute respiratory failure
37. During regular examination of a 2-yearold boy, he presents with enlarged left kidney, painless on
palpation. The right kidney was undetectable on palpation. Excretory urography shows no contrast
on the right. Cytoscopy detected hemiatrophy of the urinary bladder trigone, the right ureteral
ori?ce is not detected. What pathology is it?
A. * Agenesis of the right kidney
B. Dystopia of the right kidney
C. Hypoplasia of the right kidney
D. Agenesis of the right ureter
E. Ectopic right ureteral ori?ce
38. A 64-year-oldpatienthasbeenhospitalized with complaints of progressive jaundice that developed
over 3 weeks without pain syndrome and is accompanied by general weakness and loss of appetite.
Objectively: temperature is 36,8oC, heart rate is 78/min, abdomen is soft and painless, peritoneum
irritation symptoms are not detected, palpation reveals sharply enlarged tense gallbladder. What
disease can be characterised by these symptoms?
A. * Cancer of pancreas head
B. Duodenal ulcer
C. Acute cholecystitis
D. Chronic cholecystitis
E. Cholecystitis caused by lambliasis
39. A 37-year-old patient complains of pain in the spinal column, reduced mobility. The condition
persists for 7 years. ”Sway back” is observed, there is no movement in all spinal regions. On
X-ray: ”bamboo spine” is detected. What is the most likely diagnosis?
A. * Ankylosing spondylitis
B. Osteochondrosis
C. Spondylitis deformans
D. Tuberculous spondylitis
E. Spondylolisthesis
40. A 54-year-old patient complains of weakness, jaundice, itching skin. Disease onset was1.5 months
ago: fever up to 39oC appeared at ?rst, with progressive jaundice developed 2 weeks later. On
hospitalisation jaundice was severely progressed. Liver cannot be palpated. Gallbladder is enlarged
and painless. Blood bilirubin is 190 mcmol/L (accounting mainly for direct bilirubin). Stool is
acholic. What is the most likely reason for jaundice in this patient?
A. * Mechanicaljaundice
B. Hepatocellularjaundice
C. Hemolyticjaundice
D. Caroli syndrome
E. Gilbert’s syndrome

41. A 23-year-old man came to the surgeon with complaints of pain, redness of the skin, and swelling
in the area of his proximal interphalangeal joint of the III ?nger on the right hand. Six days ago he
pricked his ?nger with a wire. Objectively the III ?nger on the right hand is swollen, hyperaemic,
prominent in the projection of interphalangeal joint, sharply painful on touch and during
movements. Finger mobility is reduced. Fluctuation sign is present. What diagnosis corresponds to
the given clinical presentation?
A. * Articular panaritium
B. Bone panaritium
C. Subcutaneous panaritium
D. Pandactylitis
E. Finger furuncle
42. A 30-year-oldpatient was in a car accident. He is unconscious, pale, has thready pulse. In the middle
third of the right thigh there is an extensive laceration with ongoing profuse external arterial
bleeding. What urgent actions must be taken to save the life of the patient?
A. * Tourniquet above the wound of the right thigh
B. Tourniquet below the wound of the right thigh
C. Arti?cial lung ventilation
D. Precordial thump
E. Application of plaster bar
43. A 25-year-old patient has been admitted to the hospital with the following problems: weakness,
sweating, itching, weight loss, enlarged submandibular, cervical, axillary, inguinal lymph nodes.
Objectively: hepatomegaly. Lymph node biopsy revealed giant Berezovsky-Reed-Sternberg cells,
polymorphocellular granuloma composed of lymphocytes, reticular cells, neutrophils,
eosinophils, ?brous tissue, and plasma cells. What is the most likely diagnosis?
A. * Lymphogranulomatosis (Hodgkin’s lymphoma)
B. Lymph node tuberculosis
C. Lymphoreticulosarcoma
D. Cancer metastases to the lymph nodes
E. Macofollicularreticulosis
44. A 68-year-old man complains of inability to urinate for a day. On attempt of urinary bladder
catheterization there was detected a rough stricture in the membranous portion of the urethra.
What ?rst aid tactics should be applied in this case?
A. * Troacarcystostomy
B. Adenomectomy
C. Optical internal urethrotomy
D. ?-adrenoblockers
E. Urinary bladder catheterization
45. Heart X-ray of a 31-year-old man has revealed the following: with tightly ?lled opaci?ed
oesophagus there is a marginal ?lling defect in its middle third on the posterior wall; the defect is
1,8x1,3 cm in size with clear oval border. Mucosal folds are retained and envelop the defect; wall
peristalsis and elasticity are not affected. There are no complaints regarding the condition of the
patient’s alimentary canal. Make the provisional diagnosis:
A. * Oesophageal tumour
B. Achalasia cardiae
C. Oesophageal burns
D. Diverticulum
E. Barrett oesophagus

46. A 45-year-old woman underwent one year ago mastectomy followed by chemo- and radiation
therapy. She now complains of dyspnoea at rest and temperature up to 37.2oC. Her general
condition is severe, acrocyanosis is observed. The right side of her chest practically does not
participate in respiration. Percussion reveals a dull sound below the 3rd rib; auscultation detects
acute weakening of the respiratory sounds. Pleural puncture on the right has yielded a large amount
of haemorrhagic exudate. What complication has developed in the patient?
A. * Carcinomatous pleuritis
B. Acute pleural empyema
C. Acute right-sided pleuropneumonia
D. Right lung abscess
E. Pulmonary embolism
47. A patient with signs of general overexposure to cold presenting with local frostbites of ?ngers has
been delivered into an admission room. Objectively: conscious, inert, speech is slow, the skin of the
face is cold, body temperature is 34oC, heart rate is 68/min. What would be the actions of a doctor
on call?
A. * Hospitalize the patient to the surgical department
B. Hospitalize the patient to the therapeutics department
C. Hospitalize the patient to the traumatology department
D. Let the patient go home
E. Refer to a family doctor on the next day
48. A 24-year-old patient had been delivered to a thoracic department with a chest injury, fracture of the
IV, V, VI ribs on the right. Plan radiography showed the ?uid level in the pleural cavity to be
reaching the III rib on the right. Puncture contained blood clots. What is the optimal treatment
tactics?
A. * Emergency thoracotomy
B. Pleural puncture
C. Thoracentesis and thoracostomy
D. Haemostatic therapy
E. Medical thoracoscopy
49. A 74-year-old patient visited a urologist with complaints of pain above the pubis and inability to
urinate for 8 hours. At home he had taken antispasmodics and had a warm bath but no improvement
occurred. Objectively: abdomen is soft and painful above the pubis; dullness of percussion sound is
observed above the pubis. Murphy’s (Pasternatski’s) punch sign is negative on the both sides. What
condition does the patient have?
A. * Acute urinary retention
B. Paradoxalischuria
C. Chronic urinary retention
D. Anuria
E. Oliguria
50. During preventive examination a 58-yearold man on chest X-ray presents with multiple globular
pale shadows 3 cm in diameter within parenchyma of the both lungs. Examination in the oncologic
hospital: the primary focus is not found; transbronchial biopsy with cytologic investigation detected
cells of glandular neoplasm. What tactics should the physician choose?
A. * Polychemotherapy courses
B. Exploratory laparotomy
C. Exploratory thoracotomy
D. Laparoscopy
E. Symptomatic treatment at home

51. A 35-year-old man complains of persisting enlargement of his peripheral lymph nodes that cause
him no discomfort. The case history states that the ?rst lymph nodes to enlarge were cervical,
supraclavicular, and axillary; new groups of lymph nodes emerged. Objectively the lymph nodes
are soft and elastic on palpation, enlarged, painless, not ?xed to the surrounding tissue. What
examination method would be the most informative for early diagnostics of this disease?
A. * Needle biopsy
B. Magnetic resonance tomography
C. Radioisotope scanning of the skeleton
D. Ultrasound
E. X-ray
52. A woman in her early- to mid-thirties has lost her consciousness 3-5 minutes ago. On examination:
the skin is pale, no pulse over the carotid arteries, no spontaneous respiration, pupils are dilated; the
patient is nonresponsive, presents with atony. The patient’s condition can be determined as:
A. * Apparent death
B. Natural death
C. Syncope
D. Brain death
E. Coma
53. A 38-year-old man underwent surgical treatment of a wound with a suppuration focus. On the 8th
day after the procedure the wound cleared of purulo-necrotic discharge and granulations appeared.
However, against the background of antibacterial therapy, the patient’s body temperature persists as
high as 38.5-39.5oC; chills, excessive sweating, and euphoria are observed in the patient; heart rate
is 120/min. What complication of the local suppurative in?ammatory process can be suspected?
A. * Sepsis
B. Purulent-resorptive fever
C. Trombophlebitis
D. Meningitis
E. Pneumonia
54. A 28-year-old woman complains of girdle pain in her epigastric and left subcostal areas with
irradiation to the back, nausea, and vomiting without relief. On examination a surgeon observes
stomach distension and meteorism. There are positive Mondor’s, Mayo-Robson’s, and Cullen’s
symptoms. What is the most likely diagnosis?
A. * Acute pancreatitis
B. Acute cholecystitis
C. Acute intestinal obstruction
D. Aortic dissecting aneurysm
E. Splenic infarction
55. A man complains of high fever, pain in the area of his right mastoid bone, and purulent discharge
from the right ear. One week ago he had a case of URTI. Objectively the right auricle protrudes, the
skin behind the ear is hyperaemic and pastose; on palpation of the mastoid bone the pain intensi?es;
the auditory meatus is ?lled with thick pus, posterosuperior meatal wall sags; the tympanic
membrane is red and perforated. Make the diagnosis:
A. * Acute mastoiditis
B. Furuncle of the external auditory meatus
C. Acute otitis media
D. Acute otitis externa diffusa
E. Exacerbation of chronic mesotympanitis

56. A 28-year-old man after car accident received a wound to the right side of his chest in?icted by a
sharp metal object. A foamy liquid ?ows out from the wound, there are tympanitis and acutely
weakened respirations in the right. Blood pressure is 70/30 mm Hg, pulse is 120/min., Hb is 28 g/L.
X-ray shows collapsed right lung, horizontal ?uid level is at the 3rd rib. What treatment tactics
should be chosen?
A. * Urgent thoracotomy
B. Delayed thoracotomy
C. Drain the right pleural cavity
D. Apply occlusive dressing to the wound
E. Conservative therapy
57. When her car collided with a tree, a 37year-old woman felt sharp pain in her left hip joint. She was
unable to get out of the car. Her position is forced, the hip is pressed to the abdomen, ?xed, and
rotated inwards; the limb is ?exed in the knee, any attempt to change the position results in sharp
pain. Make the diagnosis:
A. * Closed dislocation of the left hip
B. Contusion of the left hip joint
C. Hemarthrosis of the left hip joint
D. Arthritis of the left hip joint
E. Closed cervical fracture of the left hip
58. A 22-year-old man at 18:00 developed persisting dull pain in the epigastrium. Three hours later
nausea appeared, he vomited once. By the morning the pain shifted to the right iliac area. Body
temperature rose to 38.6oC, developed tachycardia of 110/min. On examination there are muscle
rigidity and Bloomberg’s sign (rebound tenderness) in the right iliac area of the anterior abdominal
wall. Plain x-ray of the abdomen shows no ?uid levels, free air under the diaphragm on the right.
Make the diagnosis:
A. * Perforation of a gastric ulcer
B. Renal colic
C. Acute appendicitis
D. Acute pancreatitis
E. Acute cholecystitis
59. On ultrasound of the thyroid gland, a 47year-old woman presents with a hypoechoic node 1.6 cm in
diameter with blurred margins and intranodular hypervascularization. The doctor suspects thyroid
carcinoma. What method should be used to verify the diagnosis?
A. * Fine-needle aspiration biopsy
B. Thyroid scintigraphy
C. Case monitoring
D. Determine TSH level in the blood
E. Positron emission tomography(PET)
60. A 19-year-old young man complains of cough with expectoration of purulent sputum in amount of
100 ml per day, haemoptysis, dyspnoea, increased body temperature up to 37.8oC, general
weakness, weight loss. The patient’s condition has been persisting for 4 years. Exacerbations occur
2-3 times per year. The patient presents with malnutrition, pale skin, cyanosis of the lips, drumstick
(clubbed) ?ngers. Tympanic percussion sound in the lungs, weakened respiration, various numerous
moist crackles in the lower pulmonary segments on the left can be observed. In blood: erythrocytes
- 3.2 • 1012/L, leukocytes -8.4•109/L, ESR-56 mm/hour. On X-ray: lung ?elds are emphysematous,
the left pulmonary root is deformed and dilated. What is the most likely diagnosis?
A. * Multiple bronchiectasis of the left lung
B. Chronic left-sided pneumonia
C. Chronic abscess of the left lung
D. Left-sided pulmonary cystic dysplasia

E. Suppuration of the cyst in the left lung


61. An 8-year-old child presents with blood pressure up to 180/100 mm Hg in the upper limbs
accompanied by headaches, tinnitus, occasional nosebleeds, and high fatigability. On examination
there is no pulse over the leg arteries. ECG shows left ventricular hypertrophy. MRI-scan shows
aortic narrowing to 5 mm in the typical place. Coarctation of aorta is diagnosed. What kind of
treatment should be prescribed in this case?
A. * Surgical
B. Conservative
C. Physical therapy
D. Case monitoring
E. Abstain from surgery in favour of complex conservative therapy
62. A 6-year-old girl arrived to the in-patient unit with complaints of enlargement of the lower third of
her right thigh. According to the case history, she has been stepping carefully on her right leg and
limping for 6 months. Blood test detected anaemia. X-ray of the right thigh shows a round bone
defect with clear margins resembling melting sugar in the distal metaphysis. What provisional
diagnosis can be made in this case?
A. * Osteogenic sarcoma of the right femur
B. Rheumatoid arthritis of the right knee joint
C. Acute haematogenous osteomyelitis of the distal femoral metaphysis on the right
D. Tuberculous osteitis of the distal femoral metaphysis on the right
E. Giant cell tumour of the right femur
63. A 46-year-old man came to the surgeon’s of?ce. He complains of twitching sensation in the wound
on his left foot, insomnia, and anxiety. According to the patient, he received this wound 5 days ago,
when he accidentally stepped on a glass shard, while on the beach. He requested no medical
assistance. Objectively the patient’s general condition is satisfactory, pulse is 75/min., blood
pressure is 130/80 mm Hg, temperature is 36.9oC. On the plantar surface of his foot there is a
wound 1.5 cm long and up to 3 cm deep. The wound edges are moderately hyperaemic, no
discharge from the wound is observed. What disease can be suspected in this patient?
A. * Tetanus
B. Diphtheria
C. Anthrax
D. Fasciitis
E. Phlegmon
64. A 20-year-old student was brought to the ?rst-aid centre. He has a closed fracture of the left forearm
and a contused lacerated wound on his left shin. After the patient received initial wound
management, he presented the documents con?rming that he has received all the necessary
preventive vaccination as scheduled. What should the doctor do to prevent tetanus in this patient?
A. * Dynamic case monitoring
B. Administration of tetanus immunoglobulin
C. Administration of anti-tetanus serum
D. Antibiotic therapy
E. Administration of tetanus toxoid
65. ?During medical examination a cadet in the naval college was detected to have a painless dense
ulcer 1.5x0.5 in size in his perianal area at the 2 o’clock position. The ulcer ?oor resembles ”old
fat”. What is the provisional diagnosis?
A. * Hard syphilitic chancre of the rectum
B. Rectal ?ssure
C. Rectal ?stula
D. Anal cancer

E. Anal crypt suppuration


66. A 32-year-old woman complains of tumourlike formation on the anterior surface of her neck that
appeared 2 years ago. Within the last 3 months the tumour has been rapidly growing. It hinders
swallowing and impairs speech; the tumour causes a sensation of pressure. Objectively the skin
moisture is normal, pulse is 80/min., rhythmic, blood pressureis130/80mmHg.Intherightlobeof the
thyroid gland there is a dense lumpy node 3.0x3.5 cm that moves during swallowing. Scanning
image shows a ”cold nodule” in the thyroid gland. Make the provisional diagnosis:
A. * Thyroid cancer
B. Thyroid adenoma
C. Thyroid cyst
D. Nodular goitre
E. Autoimmune thyroiditis
67. After a surgery for a left thigh phlegmon the disease progression was complicated by sepsis. On the
7th day after the surgery there are marked signs of a generalized in?ammatory reaction, in blood
there are signs of toxic anaemia and progressing hypoproteinemia, bilirubin levels are 40 mcmol/L,
AST and ALT exceed the norm by 2.5 times. Oliguria persists (700 mL of urine per day). Name the
phase of sepsis progression:
A. * Catabolic phase
B. Stress phase
C. Anabolic phase
D. Recovery phase
E. Mixed phase
68. A 10-year-old boy, who was outdoors in windy and cold weather, developed moderate pain and
tingling in his ?ngers and toes. When he returned home, his parents noticed that the tips of
his ?ngers and toes were white and their sensitivity was lost. As the affected areas were warming
up, the ?ngers and toes developed tingling and painful sensations. Skin pallor changed into redness,
tingling stopped, mild itching and swelling of the ?ngers appeared. Determine the frostbite degree
in this child:
A. * Frostbite of the I degree
B. Perniosis
C. Frostbite of the II degree
D. Frostbite of the III degree
E. Frostbite of the IV degree
69. A 16-year-old patient has made an appointment with an otolaryngologist. He complains of elevated
body temperature and sore throat. Disease onset was 2 days ago, after the patient ate two portions
of ice-cream. Pharyngoscopy shows hyperaemic mucosa of the palatine tonsils, with purulent
exudate in the lacunae. Make the provisional diagnosis:
A. * Lacunar tonsillitis
B. Follicular tonsillitis
C. Diphtheria
D. Acute pharyngitis
E. Pseudomembranous (Vincent’s) tonsillitis
70. A 35-year-old woman complains of high body temperature and pain in the upper outer quadrant of
her right buttock, which developed after an injection. She has been presenting with this condition
for 3 days. At the site of injection, the skin is hyperaemic; there is a painful in?ltrate with an area of
softening in its centre. The woman is diagnosed with a postinjection abscess of the right buttock.
What tactics should the surgeon choose in this case?
A. * Abscess incision, sanation and drainage of the cavity
B. Hospitalization, prescription of antibiotics, UHF
C. Abscess puncture, pus removal followed by application of antiseptics

D. 10-15 minutes of low-intensity laser radiation directed at the right buttock


E. Antipyretic agents, massage, and application of dry heat to the right buttock
71. A 65-year-old woman on abdominal palpation presents with a tumour in the umbilical region and
above it; the tumour is 13x8 cm in size, moderately painful, nonmobile, pulsing. On auscultation
systolic murmur can be observed. What is the most likely diagnosis?
A. * Abdominal aortic aneurysm
B. Gastric tumour
C. Arteriovenous aneurysm
D. Tricuspid insuf?ciency
E. Bicuspid insuf?ciency
72. A 32-year-old man complains of pain in his legs that intensi?es during walking, intermittent
claudication, numbness of his toes, extremity coldness, and inability to walk more than 100 meters.
When he sleeps, his leg usually hangs down. The patient has been smoking since he was 16. He
drinks alcohol in excess. The left leg is colder than the right one; the skin of the extremities is dry.
No pulse can be detected on the pedal arteries, while pulsation of the femoral arteries is retained.
What is the most likely diagnosis?
A. * Obliterating endarteritis
B. Diabetic angiopathy
C. Leriche syndrome (aortoiliac occlusive disease)
D. Raynaud disease
E. Deep thrombophlebitis
73. A 50-year-old patient was brought to a hospital with complaints of blood in urine. Urination is
painless and undisturbed. Macrohematuria had been observed for 3 days. Objectively: kidneys
cannot be palpated, suprapubic area is without alterations, external genitalia are nonpathologic. On
rectal investigation: prostate is not enlarged, painless, has normal structure. Cystoscopy revealed no
changes. What is the most likely diagnosis?
A. * Renal carcinoma
B. Bladder tuberculosis
C. Varicocele
D. Dystopic kidney
E. Necrotic papillitis
74. A 59-year-old man complains of pain in his left eye and left side of his head, signi?cant vision
impairment of the left eye, nausea, and vomiting. Visual acuity of the right eye is 1.0. Visual acuity
of the left eye is 0.03, attempts at correction bring no improvement. Right eye intraocular pressure -
21 mm Hg, left eye intraocular pressure 65 mm Hg. Congestive injection is observed on the sclera
of the left eye. The cornea is thick and swollen. The anterior chamber is shallow, moist, and clear.
The pupil is dilated and unresponsive to the light, the fundus of the eye is not visible. What is the
most likely diagnosis?
A. * Acute attack of glaucoma of the left eye
B. Acute iridocyclitis of the left eye
C. Stage II intraocular tumour of the left eye
D. Endophthalmitis of the left eye
E. Panophthalmitis of the left eye
75. On the 15th day after a small trauma of the right foot, the patient developed indisposition,
fatigability, irritability, headache, elevated body temperature, and sensation of constriction, tension,
and twitching in the muscles of the right shin. What disease can be suspected?
A. * Tetanus
B. Anaerobic gas gangrene
C. Erysipelas

D. Acute thrombophlebitis
E. Thrombophlebitis of the popliteal artery
76. A patient has the second and third degree burns of the15% of the body surface. On the 20th day
after the trauma the patient presents with sharp increase of body temperature, general weakness,
rapid vesicular respiration; facial features are sharpened, BP is 90/50mm Hg, heart rate is 112/min.
What complication is it?
A. * Sepsis
B. Pneumonia
C. Acute intoxication
D. Purulent bronchitis
E. Anaerobic infection
77. 2 hours after a traf?c accident a 28-yearold man in a grave condition was brought to a hospital. The
patient complains of abdominal pain. He received a blow to the abdomen with the steering wheel.
Objective examination revealed the following: the abdomen does not participate in respiration, is
tense and acutely painful on palpation; the abdominal muscles are defensively tense, peritoneal
irritation signs are positive, hepatic dullness is absent. BP is 90/60 mm Hg, heart rate is 120/min.
What further treatment tactics should be chosen?
A. * Laparotomy
B. Laparoscopy
C. Cold to the abdomen
D. Ultrasound investigation
E. Laparocentesis
78. A 48-year-old woman has arrived to the surgical unit with wounds in her thigh. On examination the
wound surface has dirty-grey coating with unpleasant sweet smell. Wound content resembles
raspberry jelly. Skin tissues around the wound are glossy and turgid. Palpation reveals moderate
crepitation in the tissues. What micro?ora is the most likely to cause such in?ammation?
A. * Anaerobic clostridial
B. Anaerobic non-clostridial
C. Streptococci
D. Staphylococci
E. Blue pus bacillus
79. After a pain attack in the right subcostal area, a 58-year-oldwomanwithovernutrition developed
icteric skin and sclera, light-coloured faeces, and dark urine. Her abdomen is distended and painful
on palpation in the right subcostal area. Palpation detects liver enlargement by 2-3 cm. Blood test:
total bilirubin - 90 mcmol/L, conjugated bilirubin - 60 mcmol/L. What method of examination will
be the most informative for diagnosis clari?cation?
A. * Retrograde cholangiopancreatography
B. Intravenous cholegraphy
C. Infusion cholegraphy
D. Percutaneous transhepatic cholegraphy
E. US of the hepatopancreatobiliary zone
80. An 11-year-old boy for a month has been presenting with increasing pain in the right femur. In the
painful area there is a nonmobile painful tumour with unclear margins. The child complains of
general indisposition, weakness, increased body temperature up to 39oC.X-ray shows widened
medullary cavity, small foci of cancellous bone destruction, and onion-like lamellar exfoliation of
the cortical layer. What is the most likely pathology resulting in such clinical presentation?
A. * Ewing sarcoma
B. Osteogenic sarcoma
C. Fibrosarcoma

D. Chondrosarcoma
E. Juxtacortical sarcoma
81. A 43-year-old man complains of a protrusion in the right inguinal region, that enlarges due to strain.
He has been presenting with this condition for 6 months. Within this period the protrusion has
grown. Objectively in the right inguinal region an elastic protrusion 8x5 cm is visible. On palpation
it disappears, leaving an empty space 4x4 cm between the pedicles of the Poupart ligament. ”Cough
push” sign is positive over this opening. Make the diagnosis:
A. * Right-sided reducible inguinal hernia
B. Right-sided reducible femoral hernia
C. Cyst of the right spermatic cord
D. Right-sided inguinal lymphadenitis
E. Right-sided reducible arcuate line hernia
82. A 78-year-old man with a prostate adenoma underwent a herniotomy for a direct inguinal hernia.
After the surgery he presents with absent urination. Enlarged urinary bladder is detectable above
the patient’s pubis. What measures should be taken in this case?
A. * Bladder catheterization
B. Apply cold to the urinary bladder area
C. Prescribe processing of the postoperative wound with UHF ?eld
D. Prescribe proserin (neostigmine) intramuscularly
E. Prescribe antispasmodics subcutaneously
83. A 38-year-old patient has been brought by an ambulance to the surgical department with complaints
of general weakness, indisposition, black stool. On examination the patient is pale, there are dotted
haemorrhages on the skin of his torso and extremities. On digital investigation there are black
faeces on the glove. Blood test: Hb- 108
A. Thrombocytopenic purpura
B. Haemophilia
C. Ulcerative bleeding
D. Rectal tumour
E. Nonspeci?c ulcerative colitis
84. A 30-year-old man came to the family physician. 2 months ago he underwent a surgery for open
fracture of the humerus. On examination the patient’s condition is satisfactory; in the area of the
postoperative wound there is a ?stula that discharges a small amount of pus; the area itself is
red; ?uctuation is detected. X-ray shows destruction of the humerus with sequestra. What
complication did the patient develop during the postoperative period?
A. * Posttraumatic osteomyelitis
B. Haematogenous osteomyelitis
C. Wound suppuration
D. Posttraumatic phlegmon
E. Suture sinus
85. 3 hours after a trauma, a young man developed bradycardia of 46/min., anisocoria D>S,
hemi-hyperre?exia S>D, hemihypesthesia on the left, and a convulsive disorder. The character of
this process needs to be clari?ed. What method of examination will be the most accurate for this
purpose?
A. * Brain CT
B. Skull X-ray
C. Electroencephalography
D. Echoencephalography
E. Lumbar puncture

86. The body of a 24-year-old woman with suspected poisoning has been found on the street. Forensic
medical examination was requested by an investigator during examination of the site and the body.
According to the Criminal Procedure Code currently in force in Ukraine, forensic medical
examination is required when it is necessary to determine the:
A. * Cause of death
B. Manner of death
C. Time of death
D. Mode of death
E. Mechanism of death
87. A 37-year-old patient complains of pain in the spinal column, reduced mobility. The condition
persists for 7 years. ”Sway back” is observed, there is no movement in all spinal regions. X-ray
shows ”bamboo spine” vertebral column. What is the most likely diagnosis?
A. * Ankylosing spondyloarthritis
B. Osteochondrosis
C. Spondylitisdeformans
D. Tuberculous spondylitis
E. Spondylolisthesis
88. A surgery unit received a person with an incised stab wound on the upper third of the right thigh.
Examination detects an incised stab wound 3.0x0.5x2.0 cm in size on the inner surface of the upper
third of the right thigh. Bright-red blood ?ows from deep within the wound in a pulsing stream.
Characterize this type of bleeding:
A. * Arterial
B. Venous
C. Parenchimatous
D. Capillary
E. Mixed
89. A 47-year-old man developed the signs of decompensated laryngeal stenosis against the background
of acute phlegmonous laryngitis. He presents with inspiratory dyspnea at rest, forced position,
cyanotic skin covered in cold sweat, tachycardia, de?cient pulse, and low blood pressure. What
urgent treatment tactics should be chosen?
A. * Tracheostomy
B. Oral administration of hyposensitization substances and broncholytics
C. Intravenous administration of dehydrating agents
D. Administration of glucocorticoid hormones
E. Oxygen therapy
90. Heart X-ray of a 31-year-old man has revealed the following: with tightly ?lled opaci?ed
oesophagus there is a marginal ?lling defect in its middle third on the posterior wall; the defect is
1.8x1.3 cm in size with clear oval border. Mucosal folds are retained and envelop the defect; wall
peristalsis and elasticity are not affected. There are no complaints regarding the condition of the
patient’s alimentary canal. Make the provisional diagnosis:
A. * Oesophageal tumour
B. Achalasia cardiae
C. Oesophageal burns
D. Diverticulum
E. Barrett oesophagus

91. A 25-year-old man was hospitalized with complaints of pain in his lower abdomen and right lumbar
area that appeared one hour ago. Patient’s general state is moderately severe. Body temperature –
38.2oC, heartrate – 102/min. The tongue is dry. The abdomen is painful on deep palpation in the
right iliac area and in the Petit triangle. Aure-Rozanov and Gabay signs are positive. Make the
provisional diagnosis:
A. * Acute appendicitis
B. Right-sided renal colic
C. Cecal tumour
D. Intestinal obstruction
E. Acute cholecystitis
92. A 45-year-old man diagnosed with acute pulmonary abscess suddenly developed sharp pain in his
chest on the right and dyspnoea up to 30/min. Examination detects facial cyanosis and shallow
rapid respirations. Auscultation reveals acutely weakened respiration throughout the whole right
lung; percussion reveals a vesiculotympanitic (bandbox) resonance at the lung apex and dullness in
the lower lobe. What complication developed in this patient?
A. * Pyopneumothorax
B. Pleuropneumonia
C. Pneumothorax
D. Acute mediastinitis
E. Oesophageal perforation
93. A 5-year-old child was brought to the ENT department by an ambulance. The child presents with
cough and dif?cult respiration. From the patient’s history it is known that the child was playing
with a toy construction set, when suddenly started coughing and developed laboured breathing.
Examination detects periodical cough, laboured expiration, and respiratory lag in the left side of the
child’s thorax. Auscultation: diminished respiration on the left. Percussion: tympanitis. X-ray
shows a displacement of the mediastinal organs to the right. Make the diagnosis:
A. * A foreign body in the left bronchus, valvular bronchostenosis
B. A foreign body in the right bronchus, valvular bronchostenosis
C. A foreign body in the trachea
D. A foreign body in the left bronchus, complete bronchostenosis
E. A foreign body in the right bronchus, partial bronchostenosis
94. A 30-year-old man was brought to the neurosurgical department with complaints of constant
headaches, nausea, vomiting, fever, and weakness of the right-side limbs. Anamnesis states that one
month ago the patient had a surgery for left-sided suppurative otitis and mastoiditis. He has been
undergoing treatment in an ENT department. Approximately 2 weeks ago the temperature
increased, and the patient developed headaches. Objectively: heart rate - 98/min., BP- 140/90 mm
Hg, temperature 38.3oC. Neurologically manifested stiff neck: bilateral Kernig’s symptom,
unsteadiness during the Romberg’s maneuver. Computer tomography of the brain revealed a three
dimensional growth with a capsule in the left hemisphere. Make the diagnosis:
A. * Cerebral abscess
B. Echinococcus
C. Haemorrhage
D. Hydrocephalus
E. Arnold-Chiari malformation
95. The burns unit received a patient, who 6 hours ago during a ?re received ?ame burns. On the
patient’s body there is graybrown area of necrosis that covers 3/4 of the body perimeter.
Occasionally there are small blisters with haemorrhagic contents and patches of shredded
epidermis. What local therapy is necessary in this case?
A. * Decompression necrectomy
B. Chemical necrolysis

C. Blister puncture
D. Necrectomy with xenotransplantation
E. Necrectomy with dermal autograft
96. A woman in her early- to mid-thirties has lost her consciousness 3-5 minutes ago. On examination:
the skin is pale, no pulse over the carotid arteries, no spontaneous respiration, pupils are dilated; the
patient is nonresponsive, presents with atony. The patient’s condition can be determined as:
A. * Clinical death
B. Natural death
C. Syncope
D. Brain death
E. Comatose state
97. A boy had a foreign body removed from under his nail plate. 3 days later he developed a sharp
throbbing pain at the end of his distal phalanx, which intensi?es when the phalanx is pressed,
hyperaemia of the nail fold, elevated body temperature up to 38.5oC, and nail plate discoloration.
Make the diagnosis:
A. * Subungual panaritium
B. Erysipelas
C. Paronychia
D. Erysipeloid
E. Abscess
98. A 32-year-old woman complains of body weight loss despite her increased appetite, nervousness,
and tremor of the extremities. Objectively: the skin is moist; the thyroid gland is diffusely enlarged,
painless, soft, and mobile. Blood test: increased level of T3, T4, and thyroid-stimulating hormone
(THS). What is the most likely diagnosis?
A. * Diffuse toxic goitre
B. Thyroid carcinoma
C. Autoimmune (Hashimoto’s) thyroiditis
D. Thyroid adenoma
E. Diffuse nontoxic goitre
99. A 19-year-old young man complains of cough with expectoration of purulent sputum in the amount
of 100 mL per day, haemoptysis, dyspnoea, increased body temperature up to 37.8oC, general
weakness, weight loss. The patient’s condition lasts for 4 years. Exacerbations occur 2-3 times a
year. The patient presents with malnutrition, pale skin, cyanosis of the lips, drumstick
(clubbed) ?ngers. Tympanic percussion sound in the lungs, weakened respiration, numerous various
moist crackles in the lower pulmonary segments on the left can be observed. In blood: erythrocytes
-3.2•1012/L, leukocytes-8.4•109/L,ESR-56mm/hour. On X-ray: lung ?elds are emphysematous, the
left pulmonary root is deformed and dilated. What is the most likely diagnosis?
A. * Multiple bronchiectasis of the left lung
B. Chronic left-sided pneumonia
C. Chronic abscess of the left lung
D. Left-sided pulmonary cystic dysplasia
E. Suppuration of the cyst in the left lung
100. A 57-year-old woman during a regular ultrasound examination presented with a space-occupying
heterogeneous lesion in the right kidney. What is the most informative method of renal tumour
diagnostics?
A. * Spiral computed tomography
B. Excretory urography
C. Retrograde pyelography
D. Radioisotope renography

E. Three glass urine test


101. A 40-year-old victim of a traf?c accident sustained the following injuries: closed diaphyseal femur
fracture, brain concussion, multiple rib fractures, hemopneumothorax, degloving shin injuries.
What injuries require the most urgent attention?
A. * Multiple rib fractures, hemopneumothorax
B. Closed diaphyseal femur fracture
C. Brain concussion
D. Degloving shin injuries
E. All injuries are equivalent
102. At the railroad crossing a passenger train collided with a bus. In this collision 26 bus passengers
died, another 18 passengers received mechanical injuries of varying severity. Where will be
professional medical aid provided for the victims of this accident? Who will provide this aid?
A. * In medico-prophylactic institutions; general physicians and surgeons
B. At the site of the accident; ?rst-response emergency teams
C. At the site of the accident; specialized second-response emergency teams
D. In medico-prophylactic institutions; specialized second-response emergency teams E. In
medical institutions; all listed types of healthcare workers
103. A 45-year-old man underwent a cardiac surgery one week ago. His general state has been
deteriorating since then: dyspnoea at rest, retrosternal pain that irradiates to the neck, marked
weakness. Objectively his body temperature is hectic. His cardiac borders are expanded; apical beat
is weakened. Auscultation detects pericardial friction rub. What is the most likely diagnosis?
A. * Acute pericarditis
B. Acute cardiac aneurysm
C. Myocardial infarction
D. Acute myogenic dilatation of the heart
E. Pulmonary embolism
104. A 45-year-old man was brought by an ambulance into the emergency hospital. He complains of
sudden pain in the lumbar area, frequent painful urination, and vomiting. Examination detects pain
in the lumbar area, costovertebral angle tenderness, pain on palpation of kidneys and along the
ureter on the right. Urine test: proteins, fresh erythrocytes, leukocytes. Make the provisional
A. * diagnosis:
Urolithiasis, renal colic
B. Acute pyelonephritis
C. Acute glomerulonephritis
D. Acute renal failure
E. Polycystic kidney disease
105. ?A 40-year-old man was brought into a hospital wiyh a closed chest trauma. Examination shows
that the right side of his chest lags behind during breathing. Palpation detects a sharp pain in the
projection of ribs V, VI, and VII on the anterior axillary line and subcutaneous emphysema on the
right side of the torso. Percussion reveals tympanitis over the right-sided segments of the chest.
Auscultation detects no breathing on the right and vesicular breathing on the left. What surgical
procedure is necessary for this patient ?
A. * Immediate thoracotomy
B. Tight bandaging of the chest
C. Drainage of subcutaneous emphysema
D. Splint stabilization of the rib facture
E. Drainage of the right pleural cavity
106. Two weeks after an antibacterial therapy for a febrile illness, a 25-year-old woman developed
severe watery diarrhea, colicky abdominal pain, and elevated body temperature. Proctosigmoscopy
detects focal mucosal lesions with a pale yellow coating. What is the most likely diagnosis in this
case?

A. * Gastroenteritis
B. Crohn’s disease
C. Ulcerative colitis
D. Pseudomembranous colitis
E. Ischemic colitis
107. A 75-year-old man in severe condition suffers from dyspnea at rest, marked weakness, and
arrhythmia. Abdominal aortic pulsation is observed, further on there is a systolic murmur
detected. Palpation reveals a volumetric formation in the mesogastrium. Blood pressure is 70/40
mm Hg. There is no pulsation over the femoral arteries. Oliguria is detected. Which diagnosis is the
correct one ?
A. * Dissecting aortic aneurysm
B. Pancreatic cyst
C. Cardiosclerotic aneurysm
D. Acute pericarditis
E. Acute cardial aneurysm
108. A man complains of a heaviness behind his sternum, periodical sensation of food retention, and
dysphagia. Durig X-ray the barium contrast reveals a single pouch-like protrusion in the right
anterior wall of the esophagus. The protrusion has clear margins and a clearly defined neck. What is
most likely diagnosis in this case?
A. * Hiatal hernia
B. Varicose veins of the esophagus
C. Esophageal carcinoma
D. Esophageal polyp
E. Esophageal diverticulum
109. A 48-year-old woman has arrived to the surgical unit with wounds in her thigh. On examination the
wound surface has dirty-gray coating with unpleasant sweet smell. Wound content resembles
raspberry jelly. Skin tissues around the wound are glossy and turgid. Palpation reveals moderate
crepitation in the tissues. What microflora is the most likely to cause such inflammation?
A. * Anaerobic clostridial
B. Anaerobic non-clostridial
C. Streptococci
D. Staphylococci
E. Blue pus bacillus
110. A man suddenly developed a sharp pain in the right side of his chest. Dyspnea has rapidly
progressed. Objectively, the patient has marked acrocyanosis and is in a severe condition.
Subcutaneous emphysema is observed in the area of the patient’s neck and upper chest. Over the
right lung a bandbox resonance can be heard, respiration is absent there. The heart borders are
displaced to the left. The patient’s heart rate is 110/min., blood pressure - 110/60 mm Hg. What is
the most likely disease in this case?
A. * Spontaneous pneumothorax
B. Exudative pleurisy
C. Myocardial infarction
D. Community-acquired pneumonia
E. Lung infarction
111. A 17-year-old girl complains of a pain and swelling of her second finger on the right hand. Three
days ago she made a manicure. The pain developed on the second day after that. Objectively, her
nail fold is swolen, hyperemic, overhangs the nail plate, and is painful on palpation. What is most
likely diagnosis in this case?
A. * Paronychia

B. Subcutaneous panaritium
C. Erysipeloid
D. Subungual panaritium
E. Cutaneous panaritium
112. A 47-year-old man has been ill for 3 days already. Palpation detects a painful inflamed infiltration
in his right subcostal region. His body temperature is 38,9oC. Sonography allowed diagnosing him
with calculous destructive cholecystitis. Clinical and laboratory data are not indicative of chole
docholithiasis. What tractics should be chosen for the treatment of this man?
A. * Surgical treatment - cholecystectomy
B. Choleretics, hepatoprotectors, corticosteroids
C. Complex anti-inflammatory therapy
D. Laparocentesis, abdominal drainage
E. Monitoring, cholecystectomy if peritonitis starts developing
113. A 36-year-old woman complains of nausea, belching, liquid stool, and a pain in the epigastrium
after meals. For the last 2 years the disease has been slowly progressing. Objectively, her skin is
pale and dry, her tongue is coated, moist, and has imprints of the teeth on its edges. Abdominal
palpation detects a diffuse pain in the epigastrium. What test will be the most informative in this
case and should be conducted next?
A. * Fibrogastroscopy with biopsy of the gastric mucosa
B. Gastrointestinal X-ray
C. Comprehensive complete blood count
D. Abdominal CT scan
E. Fractional analysis of gastric secretion
114. For the last 2 years, a 32-yesr-old woman has been observing periodical pain attacks in her right
subcostal area that could be removed with no-spa (drotaverine). The pain is not always associated
with meals, sometimes it's caused by anxiety and accompanied by cardiac pain and palpitations.
Objectively, the woman is emotionally labile. Abdominal palpation detects a slight pain in the area
of the gallbladder. What pathology is the most likely to cause such clinical presentation?
A. * Chronic cholecystitis
B. Duodenitis
C. Chronic cholangitis
D. Chronic pancreatitis
E. Biliary dyskinesia
115. A 47-year-old man has received a polytrauma in a car accident: closed displaced fractures on his
right humerus and the bones of his left forearm and a closed blunt abdominal trauma. He was
brought into the admmision room 30 minutes after the trauma. His skin is pale. His blood pressure
is 90/20 mm Hq, the fracture sites are deformed and painful. The abdomen is rigid and its palpation
causes sharp pain. The Bloomberg's sign is positive. What medical procedures must be performed
first in this case?
A. * Urgent laparotomy
B. Additional examination to determine the exact diagnosis
C. Fracture blockade with a topical anesthetic
D. Infusion therapy to stabilize the blood pressure
E. Immobilization of the fractures, pain relief
116. A 65-year-old man complains of cough attacts that ocuur when he eats liquid foods. Three months
ago he was diagnosed with a carcinoma in the upper third of the esophagus. He underwent radiation
therapy. What complication developed in this man?
A. * Tracheoesophageal fistula
B. Tracheal stenosis

C. Lung abscess rupture into the pleural cavity


D. Spontaneous pneumothorax
E. Perforation of a gastric cardia ulcer
117. A 74-yesr-old woman came to a doctor complaining of a pain in her right inguinal region. The signs
appeared suddenly, approximately 2 hours ago. The woman notes that she already had these sins 3
weeks ago, but back then they disappeared on their own after she lay down. Objectively, palpation
detects below the Poupart's ligament a sharply painful, dense, and tense formation 3.5 cm in
diameter. The Dejerine sign (aggravation on coughing) is negative. What is the most likely
diagnosis in this case?
A. * Acquired incarcerated inguinal hernia
B. Acquired incarcerated femoral hernia
C. Acquired strangulated inguinal hernia
D. Acquired strangulated femoral hernia
E. Inguinal lymphadenitis
118. A 39-yesr-old man came to a doctor complaining of a pain in his left leg. The disease onset was 2
days ago. Objectively, his body temperature is 37.8C and he has subcutaneus varicose veins on the
inner surface of his left thigh and shin. The skin over the varicose veins is hot and red. The Moses
and Homans signs are negative. What is the most likely diagnosis in this case?
A. * Acute ascending thrombophlebitis of the saphenous veins in the left leg
B. Varicose saphenous veins in the left leg
C. Thrombosis of the tibial arteries on the leg
D. Postthrombotic syndrome of the left leg
E. Acute deep vein thrombosis in the left leg
119. A 74-yesr-old man has been brought into a vascular surgery department with complaints of pain and
chills in his leg. Ultrasound of his leg arteries dhows atherosclerotic method is necessary t
odetermine the localization and the extent of the pathologic process in this case?
A. * Angiography
B. Chest X-ray
C. X-ray of the extremities
D. Thermometry
E. ECG
120. A 72-yesr-old man on the 7th day after a surgical reposition of an intertrochanteric hip fracture has
suddenly developed dyspnea and an intense pain in the left side of his chest. Examination reveals
distended cervical veins and cyanosis. His respiration rate is 26/min. Auscultation detects weakened
breathing over the left lung. Heart rate - 98/min. Blood preasure - 120/70 mm Hq. CT scan shows
significant disappearance of the lung pattern on the left. Echocardiography shows no signs of right
ventricle overload. What next step will be the most advisable in this case?
A. * Surgical embolectomy
B. Installing a vena cava filter
C. Prescribing aspirin (acetylsalicylic acid)
D. Trombolytic injection into the left pulmonary artery
E. Prescribing low molecular weight heparin
121. A 43-yesr-old woman complains of persistent stomachache with reccurent pain attacks, nausea?
repeated vomiting with sragnant bowel content, abdominal distension, and flatulence. She has been
presenting wit these signs for 7 hours. Pulse is 116/min. The tongue is dry and brown. The abdomen
is symmetrically distended, soft, painful. Percussion reveals tympanitis. On auscultation there ere
bowel sounds with metallic overtone, splashing, and dripping. make the diagnosis:
A. * Acute intestinal obstruction
B. Acute destructive cholecystitis

C. Acute nonspecific colitis


D. Acute necrotizing pancreatitis
E. Acute erosive gastritis
122. A 35-yesr-old man complains of rapidly incresing fatigue, palpitations, "visual snow", dizziness. he
has a history of peptic ulcer of the stomach. Objectively the skin is pale. Vesicular respiration is
observed in the lungs. Systolic murmur is detected over the cardiac apex, heart rate is 100/min,
blood preasure is 100/70 mm Hq. The epigastrium is slightly tender on palpation. Blood test:
erythrocytes - 3.2 * 10 12/L, color index- 0.95. What type of anemia is the most likely present in this
A. * case?
Posthemorrhagic anemia
B. Hemolytic anemia
C. Chronic iron-deficiency anemia
D. Syderoblastic anemia
E. Hypoplastic anemia
Назва наукового напрямку (модуля): Семестр: 11
SRS
Опис:
6 course
Перелік питань:
1. The site of crossing with left bronchus is:
A. * The second anatomical narrowing of esophagus
B. The first anatomical narrowing of esophagus
C. The third anatomical narrowing of esophagus
D. The first physiological narrowing of esophagus
E. The second physiological narrowing of esophagus
2. What is the main objective manifestation of Zenker's diverticula?
A. * Compressible mass on the left side of the neck
B. Signs of achalasia
C. Cyanosis of the upper part of body
D. Esophago-bronchial fistula with aspiration pneumonia
E. Lung atelectasis
3. For the clinical manifestation of esophageal diverticulum is typical:
A. * The sign "of a wet pillow"
B. Dyspnea
C. Cyanosis of the upper part of body
D. Retention of stool and gases
E. Vomiting by "coffee masses"
4. What is the main method of diagnostic of esophageal diverticula?
A. * X-ray examination with barium swallow
B. Pleural punctere
C. Ultrasound examination
D. Plain X-ray examination of the chest
E. Irrigoscopy
5. The failure of the lower esophageal sphincter to relax is called:
A. Achalasia
B. * Chalasia
C. Esophageal diverticulum
D. Pilorostenosis
E. Intestinal obstruction
6. What is the cause of achalasia?
A. * Disturbance of innervation of esophagus
B. Ischemia of esophagus
C. Tumour growth of esophagus
D. Diverticula of esophagus
E. Cicatrical changes after the burn of esophagus
7. What is the cause of achalasia?
A. * Psycho-emotional trauma
B. Ischemia of esophagus
C. Tumour growth of esophagus
D. Diverticula of esophagus
E. Cicatrical changes after the burn of esophagus
8. What is the cause of achalasia?
A. * Influence of vegetotrophic substances on muscular fibers
B. Ischemia of esophagus
C. Tumour growth of esophagus
D. Diverticula of esophagus
E. Cicatrical changes after the burn of esophagus
9. What is the main treatment of the II stage of achalasia?
A. * Cardiodilatation
B. Diet, conservative treatment
C. Esophagomyotomy (Heller's operation)
D. Esophagogastroanastomosis (Helerovsky's operation)
E. Esophageal plastics by intestine
10. What is the main treatment of the III stage of achalasia?
A. * Esophagomyotomy (Heller's operation)
B. Diet, conservative treatment
C. Cardiodilatation
D. Esophagogastroanastomosis (Helerovsky's operation)
E. Esophageal plastics by intestine
11. What is the main treatment of the IV stage of achalasia?
A. * Esophagogastroanastomosis (Helerovsky's operation)
B. Diet, conservative treatment
C. Cardiodilatation
D. Esophagomyotomy (Heller's operation)
E. Esophageal plastics by intestine
12. Helerovsky's operation is used for the treatment of:
A. * Esophageal achalasia
B. Esophageal ulcer
C. Esophageal diverticulum
D. Pilorostenosis
E. Intestinal obstruction
13. What is the main prophylaxis of esophageal stricture after the chemical burn?
A. * Esophageal bougienage
B. Spasmolytics
C. Parenteral feeding
D. Pneumocompression
E. Gastrostomy
14. What complication is typical for esophageal burn?
A. * Disturbances of epiglottic valve
B. Esophageal diverticulum
C. Obstructive jaundice
D. Intestinal obstruction
E. Lerishe's syndrome
15. What is the chief clinical manifestations of reflux-esophagitis?
A. * Heartburn
B. Achalasia
C. Dysphagia

D. Vomiting
E. Coughing
16. For the clinical manifestation of sliding diaphragmatic hernia is typical:
A. * Pain behind breastbone
B. Dyspnea
C. Cyanosis of the upper part of body
D. Retention of stool and gases
E. Vomiting by "coffee masses"
17. What is the X-ray sign of diaphragmatic relaxation?
A. * Inflection of abdominal part of esophagus
B. Filling defect
C. "Rat tail" sign
D. Sign of "nishe"
E. Pneumoperitoneum
18. What disease should be the diaphragmatic relaxation differentiated from?
A. * Diaphragmatic elevation
B. Pancreatitis
C. Intestinal obstruction
D. Cholecystitis
E. Bronchial asthma
19. Treatment of patients with acute intestinal obstruction in the stage of decompensation must be:
A. * 2-4 hours of conservative, then operative
B. To 24 hours of conservative, then operative
C. Immediately operative
D. During the first days conservative treatment with the gradual increase of volume of infusion
E. Conservative in ambulatory conditions
20. The treatment of patients with strangulation acute intestinal obstruction which accompanied by the
manifestations of peritonitis must include:
A. * 2 hours of conservative treatment, then operative
B. To 12 hours conservative treatment, then operative
C. Immediately operative without conservative
D. Conservative in ambulatory conditions
E. During the first days conservative with the gradual increase of volume infusion
21. Arterial mesenteric acute intestinal obstruction belongs to:
A. * Obturation
B. Strangulation
C. Paralytic
D. Spastic
E. Mixed
22. What is the essence of arterial mesenteric intestinal obstruction?
A. * Superior mesenteric artery compresses the duodenum
B. Duodenum compresses the superior mesenteric artery
C. Acute intestinal obstruction on the background of mesenteric thrombosis
D. Mesenteric thrombosis caused by obstruction
E. Duodenum compresses inferior mesenteric artery
23. What sign is typical for phlegmonous appendicitis in contrast to simple appendicitis?

A. * Voskresenky's sign
B. Sitkovsky’s sign
C. Bartomier’s sign
D. Kocher’s sign
E. Dunphy's sign
24. What does the Voskresenky's sign mean?
A. * Increase of pain during quick sliding movements by the tips of fingers from epigastric to right iliac
area
B. Pain in right lower quadrant during palpation of left lower quadrant
C. Increase of pain in a right iliac area when the patient lies on the left side
D. Increased pain with coughing
E. Migration of pain to the right iliac area from epigastric
25. What does the Pasternatsky’s sign mean?
A. * Tapping of lumbar region cause the pain
B. Pain in right lower quadrant during palpation of left lower quadrant
C. Increase of pain in a right iliac area when the patient lies on the left side
D. Increased pain with coughing
E. Migration of pain to the right iliac area from epigastric
26. What does the Yaure-Rozanov sign mean?
A. * Painfulness during palpation of Petit triangle
B. Pain in right lower quadrant during palpation of left lower quadrant
C. Migration of pain to the right iliac area from epigastric
D. Tapping of lumbar region cause the pain
E. Increase of pain in a right iliac area when the patient lies on the left side
27. What does the Bartomier-Mikhelson's sign mean?
A. * The increase of pain intensity during the palpation of right iliac area when the patient lies on the
left side.
B. Increased pain with coughing
C. Pain in right lower quadrant during palpation of left lower quadrant
D. Increase of pain in a right iliac area when the patient lies on the left side
E. Migration of pain to the right iliac area from epigastric
28. What does the Blumberg's sign mean?
A. * The sharp increase of pain quick taking off the hand during palpation of anterior abdominal wall.
B. Increased pain with coughing
C. Pain in right lower quadrant during palpation of left lower quadrant
D. Increase of pain in a right iliac area when the patient lies on the left side
E. Migration of pain to the right iliac area from epigastric
29. The most informing method of instrumental diagnostics of acute appendicitis is:
A. * ultrasound examination
B. contrasting roentgenoscopy
C. gastroscopy
D. esophagogastroscopy
E. colonoscopy
30. Acute appendicitis in the 1st phase is necessary to differentiate from:
A. * gastric ulcer
B. pancreatitis
C. cholecystitis

D. intestinal obstruction
E. strangulated hernia
31. For retrocecal appendicitis is characteristic the sign:
A. * Gabay's sign
B. Sitkovsky's sign
C. Obrastsow's sign
D. Voskresensky's sign
E. Kulenkampf's sign
32. For retroperitoneal appendicitis is characteristic the sign:
A. * Pasternatsky's
B. Sitkovsky's
C. Yaure-Rozanov
D. Rovzing's
E. Koer's
33. Causes of the appendicular infiltrate development:
A. * late hospitalisation, misdiagnosed appendicitis
B. aggressive infection, impaired immunity
C. adhesions, increased immunity
D. peritonitis, abscessing
E. surgical trauma, infection
34. The clinical manifestation of appendicular infiltrate is:
A. * swelling
B. the signs of peritoneal irritation
C. muscular tension
D. high temperature
E. leucocytosis
35. The most informative for differentiation of appendicitis with an epigastric form of myocardial
infarction are the changes in:
A. * ECG
B. hemodynamic disturbances
C. expressed shortness of breath
D. auscultation
E. tachycardia
36. The most informative for differentiation of appendicitis with intercostal neuralgia is:
A. * paravertebral blockade
B. laparoscopy
C. microlaparotomy
D. laparocentesis
E. peridural blockades
37. Chronic residual appendicitis arises up after:
A. * acute appendicitis
B. chronic appendicitis
C. colicks
D. recurrent appendicitis
E. primary chronic appendicitis
38. Chronic residual appendicitis arises up after:

A. * appendicular infiltrate
B. chronic appendicitis
C. recurrent appendicitis
D. colicks
E. primary chronic appendicitis
39. Hyperaemia, thickening, oedema of appendix are the signs of:
A. * catarrhal appendicitis
B. phlegmonous appendicitis
C. gangrenous appendicitis
D. gangreno-perforating appendicitis
E. dystrophic appendicitis
40. Hyperemia, fibrino-purulent fur, pus the lumen are the signs of:
A. * phlegmonous appendicitis
B. catarrhal appendicitis
C. gangrenous appendicitis
D. gangreno-perforative appendicitis
E. dystrophic appendicitis
41. The peculiarities of the clinical course of appendicitis in children are caused:
A. * by the bailer form of appendix
B. by the tubular form of appendix
C. by hypertrophy of appendix
D. by atrophy of appendix
E. by the spherical form of appendix
42. For the acute appendicitis, complicated by appendicular infiltrate, in contrast to the tumour of
caecum, is characteristic:
A. * Tendency to diminishing of the tumour in the process of supervision
B. Long-term anamnesis
C. Excretion of blood from rectum
D. Curvuasier's sign
E. Frequent partial intestinal obstruction in anamnesis
43. In preparations for parenteral nutrition do not include:
A. * salt
B. hidrolizyn;
C. casein hydrolysates;
D. 10% glucose;
E. lipofundin
44. Frequently the cause of early complications after surgery using the central analgesia:
A. * depressed respiration
B. bronhiolospazm;
C. hypotension;
D. cardiac arrest;
E. acute liver failure
45. What is the lung abscess characterized by?
A. * Purulent destruction of pulmonary tissue within 1 segment with formation of cavity, filled by pus
B. Multiple destructive foci 0,3-0,5 cm in size within 1-2 segments of lungs
C. Purulent, necrosis of a pulmonary tissue within 2-3 segments, detached from adjacent pulmonary
parenchyma

D. Diffuse purulent, ichorous necrosis more than lobe without the tendency to defined demarcation
E. Accumulation of pus in a pleural cavity
46. What is the lung gangrenous abscess characterized by?
A. * Purulent, necrosis of a pulmonary tissue within 2-3 segments, detached from adjacent pulmonary
parenchyma
B. Multiple destructive foci 0,3-0,5 cm in size within 1-2 segments of lungs
C. Purulent destruction of pulmonary tissue within 1 segment with formation of cavity, filled by pus
D. Diffuse purulent, ichorous necrosis more than lobe without the tendency to defined demarcation
E. Accumulation of pus in a pleural cavity
47. Homogeneous spherical shadow with regular edge on the background of intact pulmonary tissue on
X-ray is typical for:
A. * Lung cyst
B. Tuberculoma
C. Peripheral lung cancer
D. Tubercular cavern
E. Lung emphysema
48. Heterogeneous shadow with calcifications and regular edge on X-ray is typical for:
A. * Tuberculoma
B. Lung cyst
C. Peripheral lung cancer
D. Tubercular cavern
E. Lung emphysema
49. What is the typical method of treatment of pyopneumothorax?
A. * Drainage of pleural space
B. Pleural puncture
C. Thoracotomy
D. Pneumonectomy, bilobectomy, lobectomy
E. Conservative treatment
50. Where is performed the drainage of pleural space in pyopneumothorax?
A. * II intercostal space, midclavicular line and VII intercostal space, scapular line simultaneously
B. II intercostal space, midclavicular line
C. II intercostal space, scapular line
D. VII intercostal space, midclavicular line
E. VII intercostal space, scapular line
51. What is the typical treatment of noncomplicated rib fracture?
A. * Novocaine block
B. External fixation of ribs
C. Intrmedullary costal osteosynthesis;
D. Mechanical ventilation with positive end-expiratory pressure
E. Thoracotomy
52. What is the first aid of floating rib fracture?
A. * Fixation of floating segment
B. Analgesics
C. Spasmolytics
D. Hemostatic drugs
E. Antibiotics
53. What type of Novocaine block is used for the treatment of floating rib fracture?

A. * Paravertebral block
B. Paranephral block
C. Spinal block
D. Epidural anesthesia
E. Lung root dlock
54. Partial pneumothorax means:
A. * Collapse of lung to 1/3 of its volume
B. No collapse of lung
C. Collapse of lung to 2/3 of its volume
D. Collapse of lung more than 2/3 of its volume
E. Total collapse of lung
55. What is revealed in pneumothorax by auscultation?
A. * The breathing isn't auscultated
B. Vesicular breathing
C. Amphoric breathing with moist rales
D. Bronchial breathing with moist rales
E. Harsh breathing with dry rales
56. The absence of breathing sound by auscultation is typical for:
A. * Pneumothorax
B. Chronic bronchitis
C. Pneumonia
D. Lung emphyzema
E. Lung abscess
57. What is the first aid in closed pneumothorax?
A. * It doesn't require first aid measures
B. Pleural drainage
C. Compression bandage with closure of the wound
D. Artificial respiration
E. Intubation
58. What is revealed in hemothorax by auscultation?
A. * The breathing isn't auscultated
B. Vesicular breathing
C. Amphoric breathing with moist rales
D. Bronchial breathing with moist rales
E. Harsh breathing with dry rales
59. What method is the most informative in differential diagnostic of hemothorax with pleurisy?
A. * Pleural puncture
B. Clinical manifestation
C. Sputum analysis
D. Auscultation
E. X-ray examination
60. What method is the most informative in the diagnostic of hemothorax?
A. * Pleural puncture
B. General blood analysis
C. Sputum analysis
D. Auscultation

E. X-ray examination
61. Indication to early operative interference at acute pancreatitis is:
A. Acute oedematous pancreatitis
B. Acute pancreatolysis
C. * Acute traumatic pancreatitis at the „fresh” break the gland
D. Acute fatty pancreatitis
E. A faithful answer is not present
62. Indication to early operative interference at acute pancreatitis is:
A. Acute pancreatolysis
B. Acute oedematous pancreatitis
C. * Progressive multiple organ failure what not added conservative therapy during 48-72 hours
D. Acute fatty pancreatitis
E. Forming of pseudocyst
63. What is pancreatectomy:
A. Delete the necrotic area within the limits of nonviable fabrics
B. Delete the necrotic area within the limits of healthy fabrics
C. Delete part of organ with his transversal cutting within the limits of the changed fabrics
D. * Total delete of organ
E. There is not a faithful answer
64. What is pancreas located in relation to a peritoneum:
A. * Retroperitoneal
B. Mesoperitoneal
C. Intraperitoneal
D. All answers are incorrect
E. Variously
65. What is blood supply the body and tail pancreas:
A. * Splenic artery
B. A.gastroduodenalis
C. A.gastrica sinistra
D. A.cystica
E. Variously
66. Mondor’s sing at acute pancreatitis:
A. * Violet spots on face and trunk
B. Cyanosis sides of stomach and trunk
C. Cyanosis skin of stomach
D. Icteritiousness round a belly-button
E. Cyanosys of hands
67. Holsted’s sing at acute pancreatitis:
A. Violet spots on face and trunk
B. Cyanosis sides of stomach and trunk
C. * Cyanosis skin of stomach
D. Icteritiousness round a belly-button
E. Cyanosys of hands
68. Pulsating tumor-like formation with a reddish tinge of the skin over his right or left from the front
of the sternum is characteristic:
A. * Aneurism

B. Aneurysms of the descending aorta


C. Abdominal aortic aneurysms
D. Embolism of aorta
E. Thrombosis of the aorta
69. Pulsating tumor-like formation with a reddish tinge of the skin over his right or left from the front
of the sternum is characteristic:
A. * No right answer
B. Aneurysms of the descending aorta
C. Aneurysms of the abdominal aorta
D. Embolism of aorta
E. Thrombosis of the aorta
70. For coarctation of the aorta is characterized
A. * Well muscled shoulder girdle
B. Poor development of the musculature of the shoulder girdle
C. Well muscled legs
D. Good development of the pelvic belt
E. All answers are correct
71. For coarctation of the aorta is characterized
A. * Poor development of the pelvic girdle muscles
B. Poor development of the musculature of the shoulder girdle
C. Well muscled legs
D. Good development of the pelvic belt
E. All answers are correct
72. For diagnostic coarctation of the aorta used:
A. * Contrast aortography
B. Pulse Oximetry
C. Radiography limbs
D. Radiography abdominal
E. All answers are correct
73. For diagnostic coarctation of the aorta used:
A. * No right answer
B. Pulse Oximetry
C. Radiography limbs
D. Radiography abdominal
E. All answers are correct
74. Reducing pulsations on the left radial artery characteristic for:
A. * Lesion of the left subclavian artery
B. Lesions of the aortic arch
C. Occlusion of the brachial artery
D. Lesions of the abdominal aorta
E. Lesions of the superior vena cava
75. In the diagnosis pathology of the subclavian artery leading place is:
A. * No right answer
B. Thermometry
C. Radiography of the neck
D. Radiography of the chest cavity

E. Ultrasonography of the abdomen


76. In the diagnosis of atherosclerotic lesions an important role plays:
A. * Nuclear Magnetic Resonance
B. Thermometry
C. Rheovasography
D. Radiological examination of the neck
E. Ultrasonography of the abdomen
77. Indications for carotid endarterctomy based on
A. * Degree of stenosis of internal carotid artery
B. Limitation of atherosclerosis
C. Prescription treatment of atherosclerosis
D. Patient's wishes
E. Want doctor
78. Carotid endarterctomy contraindicated
A. * At liver failure
B. 4-5 months. myocardial infarction
C. Up to 10 weeks after stroke
D. Contraindications No
E. All true
79. For aneurysms the thoracic aorta is characterized
A. * No right answer
B. Reduced breast pressure
C. Pressure changes do not
D. All answers are correct
E. The lower abdominal pressure
80. For the initial part of the aortic arch aneurysm is characterized by
A. * No right answer
B. The decrease and ¬ pazdyvanie pulse on the carotid artery on the left
C. The increase in size and acceleration of the pulse at the radial artery on the right
D. The increase in size and acceleration of the pulse at the radial artery on the left
E. All answers are correct
81. For diagnosis of aneurysms of the thoracic aorta is used:
A. * No right answer
B. Pulse oximetry
C. Peripheral vascular ultrasound
D. Ultrasonography of the abdomen
E. Thermometry
82. For coarctation of the aorta is characterized
A. * Well muscled shoulder girdle
B. Poor development of the musculature of the shoulder girdle
C. Well muscled legs
D. Good development of the pelvic belt
E. All answers are correct
83. For coarctation of the aorta is characterized
A. * No right answer
B. Pulse on the femoral artery is absent

C. Pulse on the femoral artery is defined clearly


D. Pulse on the femoral artery satisfactory
E. All answers are correct
84. For diagnostic coarctation of the aorta used:
A. * ECG
B. Pulse Oximetry
C. Radiography limbs
D. Radiography abdominal
E. All answers are correct
85. Syndrome of vertebrobasilar insufficiency characteristic:
A. * Lesions of vertebral arteries
B. Lesions of the internal carotid arteries
C. Lost external carotid arteries
D. Lesions of brachial artery
E. Lesions of the aortic arch
86. For lesions the subclavian artery is characterized by:
A. * Weak hands
B. Dermahemia hands
C. Increased filling of subcutaneous veins of the upper extremities
D. The pulsation of the arteries of the upper extremities is not broken
E. All true
87. For atherosclerotic carotid arteries is characterized by:
A. * The pulsation of the carotid arteries is not determined
B. Dermahemia neck
C. Increased filling saphenous veins neck
D. The pulsation of the carotid artery is not broken
E. All true
88. Indications for carotid endarterctomy based on
A. * Structural characteristics of atherosclerotic plaque
B. Limitation of atherosclerosis
C. Prescription treatment of atherosclerosis
D. Patient's wishes
E. No right answer
89. Localization relapsing great saphenous vein are
A. * 2-3 cm below the inguinal ligament
B. In the upper third of the lower extremity
C. In the popliteal fossa
D. In the lower third of the thigh
E. In the lumbar region
90. Transient edema of lower limbs is characteristic for:
A. * Varicose
B. Atherosclerotic lesions
C. Obliterative endarteritis
D. Leriche
E. Femoral artery embolism
Назва наукового напрямку (модуля): Семестр: 12
SRS2018
Опис:
MIX
Перелік питань:
1. ?A patient with uterine fibromyoma sized up to 8-9 weeks of pregnancy consulted a gynaecologist
about acute pain in the lower abdomen. Examination revealed pronounced positive symptoms of
peritoneal irritation, high leukocytosis. Vaginal examination revealed that the uterus was enlarged
up to 9 weeks of pregnancy due to the fibromatous nodes, one of which was mobile and extremely
painful. Appendages were not palpable. Discharges were mucous, coming in moderate amounts.
What is the treatment tactics?
A. * Urgent surgery (laparotomy)
B. Surveillance and spasmolytic therapy
C. Fractional diagnostic curettage of the uterine cavity
D. Surgical laparoscopy
E. Surveillance and antibacterial therapy
2. To perform a diagnostic laparoscopy on suspicion of cyst of the right lobe of the liver, staging port
for laparoscope is typically placed
A. * In the periumbilical area
B. On the midline of the abdomen in the epigastrium
C. On the midline of the abdomen in hypogastrium
D. At McBurney point
E. At Volkovych-Kocher point
3. The maximum flow rate of gas through the needle of Veresh?
A. 1 L / min
B. 5 L / min
C. 7 L/ min
D. * 3 L / min
E. 9 L / min
4. The parts of tools for connecting tissues are
A. Handless and inserts without cremaliers
B. Cremaliers without handles, inserts and tube
C. With handles and inserts with cremaliers
D. Tools are solid
E. * Cremalier handles, inserts and tube
5. To perform a diagnostic laparoscopy on suspicion of cyst of the right lobe of the liver, staging
second port typically is placed
A. * On the midline of the abdomen in the epigastrium
B. In the periumbilical area
C. On the midline of the abdomen in hypogastrium
D. At McBurney point
E. At Volkovych-Kocher point
6. The reason why surgeons use fiber optic cable is
A. * To transfer the "cold" light
B. For gas supply
C. For the signal from the camera
D. To apply liquid
E. For coagulation
7. The solution used in the suction-irrigator is
A. Glucose
B. Furacillinum
C. * Physiological
D. Solution with antibiotic
E. Chlorhexidine
8. The three main groups of instruments for laparoscopy are
A. For access, basic and supportable
B. For access, general and special purpose
C. General, special, utility
D. * For separation of tissues, specific, for connection of tissues
E. Tools for laparoscopy are not divided into groups
9. The three main indicators of apparatus for insufflation important for the surgeon during surgery are
A. The pressure in the abdomen, gas temperature and carbon dioxide level in the tank
B. Pressure in the abdomen, the rate of gas supply and gas temperature
C. The pressure in the abdominal cavity, the gas flow rate and the level of carbon dioxide in the tank
D. * The pressure in the abdominal cavity, the gas flow rate and the amount of gas spent
E. The rate of gas supply, the amount of spent gas and gas temperature
10. The three main types of laparoscopes by angle?
A. 0 °, 15 °, 45 °
B. 0 °, 45 °, 60 °
C. 10 °, 25 °, 50 °
D. * 0 °, 30 °, 45 °
E. 0 °, 5 °, 10 °
11. The tools that are used in preparing and applying pneumoperitoneum include all of these except:
A. Janet Syringe
B. Scalpel
C. Puncture needle
D. Clamp
E. * Needle clamp
12. The tools that are used when preparing and applying the pneumoperitoneum include all of these,
except:
A. Janet Syringe
B. Scalpel
C. Puncture needle
D. Clamp
E. * Dressing
13. The tools used in the performance of laparoscopic cholecystocholangiography include all of these
except:
A. "Record"Syringe
B. Iversen - Roholm Needle
C. Scalpel
D. * Soft clip
E. Metal palpator
14. Thoracoscopy - is:
A. Overview of the abdominal cavity using a special endoscope
B. * Overview of the chest cavity using a special endoscope

C. Review of the mediastinum using a special endoscope


D. Review the joint cavity using a special endoscope
E. Pelvic exam using a special endoscope
15. To dissect cancer node surgeon should use this tool:
A. monopolar electrode
B. * surgical "crocodile" clamp
C. anatomical clip
D. bipolar forceps
E. "universal" clamp
16. To isolate gallbladder surgeon can use this tool:
A. * monopolar "scoop" electrode
B. surgical clamp
C. anatomical clip
D. bipolar forceps
E. monopolar electrode
17. To isolate the cystic artery surgeon should use this tool:
A. * dissector
B. surgical clamp
C. anatomical clip
D. bipolar forceps
E. "universal" clamp
18. To isolate the cystic duct surgeon should use this tool:
A. * dissector
B. surgical clamp
C. anatomical clip
D. bipolar forceps
E. "universal" clamp
19. When carrying out a diagnostic laparoscopy on suspicion of liver echinococcosis, optical port input
is typically carried out
A. * In the periumbilical area
B. On the midline of the abdomen in the epigastrium
C. On the midline of the abdomen in hypogastrium
D. At McBurney point
E. At Volkovych-Kocher point
20. To perform arthroscopy there is necessity of:
A. Introduction of air into the abdominal cavity
B. Intubation of main bronchi
C. The introduction of air into the chest cavity
D. The introduction of air into the joint cavity
E. * The introduction of fluid into the joint cavity
21. When carrying out a diagnostic laparoscopy on suspicion of acute pancreatitis, port for laparoscope
typically is placed
A. * In the periumbilical area
B. On the midline of the abdomen in the epigastrium
C. On the midline of the abdomen in hypogastrium
D. At McBurney point
E. At Volkovych-Kocher point

22. To perform laparoscopy surgeon needs:


A. * Introduction of air into the abdominal cavity
B. Introduction of fluid in the abdominal cavity
C. Introduction of air into the chest cavity
D. Introduction of fluid in the chest cavity
E. The introduction of fluid into the joint cavity
23. To perform laparoscopy there is necessity of:
A. * Introduction of air into the abdominal cavity
B. Introduction of fluid in the abdominal cavity
C. The introduction of air into the chest cavity
D. Introduction of fluid in the chest cavity
E. The introduction of fluid into the joint cavity
24. To perform thoracoscopy there is necessity of:
A. Introduction of air into the abdominal cavity
B. * Intubation of main bronchi
C. The introduction of air into the chest cavity
D. Introduction of fluid in the chest cavity
E. The introduction of fluid into the joint cavity
25. When carrying out a diagnostic laparoscopy on suspicion of abscess of Douglas space, the first port
is typically placed
A. * In the periumbilical area
B. On the midline of the abdomen in the epigastrium
C. On the midline of the abdomen in hypogastrium
D. At McBurney point
E. At Volkovych-Kocher point
26. To stop the bleeding from the liver surgeon can use this tool:
A. * monopolar electrode "scoop"
B. surgical clamp
C. anatomical clip
D. bipolar forceps
E. "universal" clamp
27. Trocar used in laparoscopy differs from the classic by:
A. No valve
B. Always disposable
C. * The presence of a special valve
D. Has a length of 10 cm
E. Similar to other trocars
28. Trocar used in performance of laparoscopic thoracoscopy is special because of the next reason:
A. * No valve
B. Always disposable
C. The presence of a special valve
D. Has a length of 10 cm
E. Similar to other trocars
29. Videocomplex for laparoscopy has the following components:
A. The monitor
B. Camcorder

C. Light
D. Laparoscope
E. * All answers are correct
30. VideoRS is:
A. Apparatus for coagulation
B. * Apparatus for recording video
C. Tools for fixation of the abdominal cavity
D. Apparatus for recording image
E. The device for the aspiration of fluid from the abdominal cavity
31. Visual laparoscopy was first applied by:
A. * Kelling in 1901
B. Ott in 1901
C. Caroli in 1909
D. Bergman in 1926
E. Rumann in 1932
32. What are the parts of tools for connecting tissues?
A. Handless and inserts without cremaliers
B. Cremaliers without handles, inserts and tube
C. With handles and inserts with cremaliers
D. Tools are solid
E. * Cremalier handles, inserts and tube
33. What are the parts of tools for tissue separation?
A. Handles and inserts without cremaliers
B. cremaliers without handles, inserts and tube
C. with handles and inserts with cremaliers
D. tools for dissection are solid
E. * Handles with cremaliers, inserts and tube
34. When carrying out a diagnostic laparoscopy on suspicion of cyst of the left lobe of the liver,
entering the first port is typically carried out
A. * In the periumbilical area
B. On the midline of the abdomen in the epigastrium
C. On the midline of the abdomen in hypogastrium
D. At McBurney point
E. At Volkovych-Kocher point
35. What are the three main indicators of apparatus for insufflation important for the surgeon during
surgery?
A. The pressure in the abdomen, gas temperature and carbon dioxide level in the tank
B. Pressure in the abdomen, the rate of gas supply and gas temperature
C. The pressure in the abdominal cavity, the gas flow rate and the level of carbon dioxide in the tank
D. * The pressure in the abdominal cavity, the gas flow rate and the amount of gas spent
E. The rate of gas supply, the amount of spent gas and gas temperature
36. What are the three main types of laparoscopes in the classification of angle?
A. 0 °, 15 °, 45 °
B. 0 °, 45 °, 60 °
C. 10 °, 25 °, 50 °
D. * 0 °, 30 °, 45 °
E. 0 °, 5 °, 10 °
Назва наукового напрямку (модуля): Семестр: 12
KROK2018
Опис:
MIX
Перелік питань:
1. ?"Blind" trocar is:
A. first (optical) Trocar
B. Surgical Trocar
C. Assistant Trocar
D. * Trocar type «Visiport»
E. all answers are correct
2. "Minimally invasive surgery" means:
A. * perform operations at minimum access
B. perform the operation as quickly as possible
C. perform operations using laser
D. executing operation in only one anatomic site
E. perform operations after prior chemotherapy
3. Arthroscopy is:
A. Overview of the abdominal cavity using a special endoscope
B. Overview of the chest cavity using a special endoscope
C. Review of the mediastinum using a special endoscope
D. * Review the joint cavity using a special endoscope
E. Pelvic exam using a special endoscope
4. Aspirator-irrigator is:
A. The device that provides a supply of fluid into the abdominal cavity
B. The device that ensures removal of fluid from the abdominal cavity
C. The device that maintains pressure in the abdomen
D. There is no right answer
E. * Answers A, B are right
5. At laparoscopy one can figure out all of the listed except:
A. Color of tumor
B. * The morphological structure of the tumor
C. The presence of metastases
D. The consistency of the tumor
E. The mobility of tumor
6. At what temperature presterilization of laparoscopic instruments is performed?
A. * 50 ° C
B. 20 ° C
C. 30 ° C.
D. 90 ° C
E. 120 ° C
7. At what temperature sterilization of laparoscopic instruments in dry-air camera is performed?
A. 100-120 ° C
B. 140-160 ° C
C. * 170-180 ° C
D. 190-200 ° C
E. 80-90 ° C
8. Before the needle of Veresh puncture one must:
A. * Fix the abdominal wall
B. Press the epigastrium
C. Press the left and right sides
D. Add the liquid into the urinary catheter
E. Ask permission from the anesthesiologist
9. For of laparoscopy on perforated ulcer, a second port input is carried out
A. * In mesogastrium 1 cm above or below the navel on the median line
B. On the midline of the abdomen in the epigastrium 10 cm above the navel
C. At Volkovych-Kocher point
D. At McBurney point
E. At Kerr point
10. Benefits of operations performed by laparoscopic method over traditional:
A. More quick
B. More effective
C. Short postoperative period
D. Cosmetic effect
E. * Answers C and D are correct
11. Culdoscopy is:
A. Overview of the abdominal cavity using a special endoscope
B. Overview of the chest cavity using a special endoscope
C. Review of the mediastinum using a special endoscope
D. Review the joint cavity using a special endoscope
E. * Pelvic exam using a special endoscope
12. In the implementation of laparoscopy for suspected salpingal disorder, optical port input is done
A. * In mesogastrium 1 cm above or below the navel the median line
B. On the midline of the abdomen in the epigastrium 10 cm above the navel
C. On the midline of the abdomen in hypogastrium 10 cm below the navel
D. At McBurney point
E. At Kerr point
13. Diagnostic laparoscopy is informative at:
A. Vesico-ureteric reflux
B. * Acute appendicitis
C. Adrenal tumor
D. Hemophilia
E. Femoral hernia
14. Diagnostic laparoscopy is performed for:
A. * Confirmation of the diagnosis
B. Removal of the gallbladder
C. Removal of fluid at ascites
D. Appendectomy
E. There is no right answer
15. Directly into the abdominal cavity carbon dioxide is supplied through:
A. Laparoscope
B. Insuflator
C. Coagulator

D. * Trocar
E. Light
16. In the implementation of laparoscopy on suspicion of pelvic tumor, staging port for laparoscope is
put
A. * In mesogastrium 1 cm above or below the navel the median line
B. On the midline of the abdomen in the epigastrium 10 cm above the navel
C. On the midline of the abdomen in hypogastrium 10 cm below the navel
D. At Volkovych-Kocher point
E. At Kerr point
17. During laparoscopy abdominal pressure is maintained within:
A. 20 mm Hg.
B. 5 mm Hg.
C. * 12 mm Hg.
D. 25 mm Hg.
E. 8 mm Hg.
18. During laparoscopy abdominal pressure supports device called:
A. Duomat
B. Infusion pump
C. * Insuflator
D. Aspirator
E. Respirator
19. Endosurgical complex consists of:
A. Videocomplex
B. Systems of aspiration and irrigation
C. Systems of insufflation
D. Coagulation Systems
E. * All answers are correct
20. Endovideosurgery complex consists of:
A. Videocomplex
B. Systems of aspiration and irrigation
C. Systems of insufflation
D. Coagulation system
E. * All answers are correct
21. For aspiration biopsy instruments are sterilized by:
A. boiling
B. autoclaving
C. * dry high temperature camera
D. In formaline camera
E. In the antiseptic solution
22. For imposing pneumoperitoneum by needle of Veresh into the abdominal cavity one must not enter:
A. * Above the pubis of the white line of the abdomen
B. In the right iliac region
C. In the left iliac region
D. Below the navel on the white line of the abdomen
E. Above the navel on the white line of the abdomen
23. For imposing pneumoperitoneum needle puncture into the abdominal cavity can be made through:

A. * above the pubis in linea alba


B. In the right iliac region
C. In the left iliac region
D. below the navel on linea alba
E. above the navel on linea alba
24. For the creation of pneumoperitoneum in modern laparoscopy mostly is used:
A. Air
B. Nitrous Oxide
C. * Carbon dioxide
D. Oxygen
E. Saline
25. For traction of gallbladder surgeon should use this tool:
A. dissector
B. surgical clamp
C. * anatomical clip
D. bipolar forceps
E. monopolar electrode
26. Furrier needle is used to:
A. continuous suture of the intestine
B. knot suture of the intestine
C. for the closure of parenchymal organs
D. * for stitching wounds
E. for suturing the stomach wall
27. Hasson Trocar is used for:
A. * open method of setting the optical trocar
B. after a needle of Veresh for carboxyperitoneum
C. as a surgical port for working tools
D. as an additional assistant trocar
E. in all these cases
28. In the implementation of laparoscopy on suspicion of pelvic tumor, setting the working port is
carried out
A. * On the midline of the abdomen in hypogastrium 10 cm below the navel
B. In mesogastrium 1 cm above or below the navel the median line
C. On the midline of the abdomen in the epigastrium 10 cm above the navel
D. At Volkovych-Kocher point
E. At Kerr point
29. How much time is spent for sterilization of laparoscopic instruments in dry-air closet?
A. 30 minutes
B. 2 hours
C. 2 h 30 min
D. 3 hours
E. * 1 hour
30. How much time is spent on presterilization processing of laparoscopic instruments?
A. 10 minutes
B. 20 minutes
C. 30 minutes

D. * 15 minutes
E. 1 hour
31. In some of these cases using monopolar coagulation is prohibited?
A. At the presence titanium plates
B. At the presence of adrenal lesions
C. * At the presence of pacemaker
D. At the presence of echinococcal liver disease
E. At the presence of bladder damage
32. In the medial fold there is:
A. Hepatic artery
B. portal vein
C. choledochus
D. Inferior vena cava
E. * Obliterated umbilical artery
33. To perform a diagnostic laparoscopy on intra-tumor staging, the second port typically is placed
A. * Depending on the tumor site
B. On the midline of the abdomen in the epigastrium
C. At Kerr point
D. At McBurney point
E. At Volkovych-Kocher point
34. In the round ligament of liver there is:
A. Hepatic artery
B. portal vein
C. choledochus
D. Inferior vena cava
E. * Obliterated hepatic vein
35. Insuflator is:
A. The device that provides air flow in the abdominal cavity
B. The device, which provides creating space in the abdominal cavity
C. The device that maintains pressure in the abdomen
D. There is no right answer
E. * Listed answers are right
36. Into clinical practice laparoscopic method of examination was introduced by:
A. Kelling in 1901
B. * Yakobeus in 1910
C. Schmidt in 1927.
D. Tracking in 1929
E. Raddok in 1929

You might also like